You are on page 1of 174

@civilminds

@civilminds
@civilminds

e 2 1 s t c entury
h e i l l i t e rate of th
"T cannot
s e w h o
e tho
will not b o s e who
b u t t h
editorial write,
read and
l e a r n, and
note cannot le
arn , u n

relearn" offler
- Alvin T

LEARN, UNLEARN, RELEARN – KEY TO SUCCESS

Learning appears to be quite hard. Most of us spend our attuned with what's contemporary and thus succeeding
precious time in accumulating a body of knowledge which in an ambiguous, unpredictable and constantly evolving
we expect to use for the rest of our lives. But, it's not environment.
learning that is hard; it's unlearning that we struggle with. People who find opportunities in a changing
environment are those who are actively looking for them.

The most difficult phase for a person certainly is to unlearn, The choice is simple: act or be acted upon. Since change is

because he/she is completely attached to some previous the only constant you can truly rely upon, learning to

experiences, which may be undesirable for continued navigate and adapt to it is not just important to your

learning and future evolution. Thus, it really becomes survival, it's essential for you to thrive in the bigger

difficult to forget your past, and re-evalute your current games of life.

standing and capabilities to start afresh.

The need of the hour is to constantly rejuvenate for

Adaptability is the key to UNLEARN and RELEARN phase. enduring strategic advantage over competitors and this
mandates an out-and-out adoption of the learn-unlearn-
To succeed today you must be in a constant state of
relearn relevant techniques symbolized by flexibility,
adaptation – continually unlearning old 'rules' and
agility, steadfastness and tactfulness. It requires a
relearning new ones. That requires continually questioning
gradual and steady process of transformation and
assumptions about how things work, challenging old
overhauling your thinking process. It requires
paradigms, and 'relearning' what is now relevant.
challenging old assumptions and creating newer layers
of assumptions. Are you up for the change and ready to
Considering the above, agility to learn and grasping new scale new heights?
things is the new normal. When the rules of the game are
changing fast, swiftness and the attitude to let go of the old
”It is neither the strongest of the species that survives, nor the
rules and learning new ones is no more a matter of choice
most intelligent. It is the one that is the most adaptable to
but a critical ingredient to sustaining in this highly
change.”- Charles Darwin
competitive environment. Constant learning and upgrading
yourself is the key to unlock your change proficiency, to stay
@civilminds

focus
CRITICAL NOTES FOR THE PREPARATION OF GENERAL STUDIES PAPERS

Chairperson and Managing Director (CMD) : Dear Students,


Mr. V.P. Gupta
Chief Executive Officer : Abhishek Gupta
Executive Editor : Ritesh Singh
FOCUS is the Study Circle's monthly publication of contemporary issues and
Editorial Team : Ritesh Singh, Gajanan Dwivedi, current affairs analysis. It is in fact a collective effort, by experienced
Naweed Akhter, Sanjeev Kumar Pandey, Vikas educators in varied themes, of identifying current affairs in light of the UPSC
Chauhan, Hiralal Mandal, Ankur Sharma, Jasmine
Sokhi, M Burhanuddin, Vrinda Gupta, Maharshi syllabus, supplementing them with background information, explaining
Sharma, Mangal Singh, Nagendra Pratap Singh, their related dimensions, lending them a generalist viewpoint and thus
Pradip Singh
producing critical notes for the preparation of General Studies' syllabus.
Design and Production : Rohit Rajput, Deepti
Gupta
The publication, as the name suggests, focuses solely on issues which are
The purpose of Focus is to expound the applied
relevant to the factual as well as applied aspects of the General Studies'
nature of General Studies to Civil Services
aspirants thereby making them aware of the real syllabus. That is how Focus covers all anticipated issues and themes for the
demand of UPSC exam.
upcoming Civil Services Examination (CSE).
The sources relied for selection and analysis of
issues are: Features
1. The Hindu, The Hindu (Sunday)
2. The Indian Express Part One | Current Affairs Analysis
3. The Times of India
4. Asian Age n This month edition covers analysis of news from the
5. The Tribune
6. The Economic Times
immediately preceding month.
7. Hindu Business Line n All news/issues are categorized and clubbed syllabus-wise
8. Frontline
9. Economic and Political Weekly (International Relations, Polity & Governance, Science & Technology,
10. World Focus Ethics, Integrity & Aptitude and so on) for efficient study.
11. BBC
12. Yojana n Background information has been added to provide context.
13. Kurukshetra
n Related and Additional information-dimensions have been explained
to add depth to your understanding.
n Maps and figures have been provided for associative and retentive
learning.

HEAD OFFICE & NEW DELHI CAMPUS


309, Kanchanjunga Building, Barakhamba Road, Part Two | Contributors Zone
Connaught Place, New Delhi-110 001
Tel : 011-23317293, 23318135-36, 23738906-07 n Articles by Rau’s professors and Research team

BENGALURU CAMPUS n Essays by Rau’s students


2nd Floor, AKS Plaza, 10 Industrial Layout,
Jyoti Niwas College (JNC) Road, 5th Block,
Koramangala, Bengaluru – 560 095 (Karnataka) Part Three | Practice Zone
Tel : 080-25535536, 7, 8, 9, 09916035536
n MCQ’s from the current affairs analysis covered in this edition with
JAIPUR CAMPUS
emphasis on Preliminary General Studies - Paper I
701, Apex Mall, Lal Kothi,
Tonk Road, Jaipur-302 015 (Rajasthan) n Descriptive/Essay type questions from important editorials in this
Tel : 0141-6450676, 093515 28027, 07877036167
edition and Case Studies with emphasis on Main General Studies
email : contact@rauias.com
Papers.
website : www.rauias.com

connect : www.facebook.com/rausias
For further understanding or discussion in any topic, please consult your
NO PART OF THIS PUBLICATION MAY BE REPRODUCED respective professors.
OR TRANSMITTED, IN ANY FORM OR MANNER OR BY
A N Y M E A N S - E L E C T RO N I C , M E C H A N I C A L ,
PHOTOCOPY OR OTHERWISE, OR STORED IN ANY Good Luck!
RETRIEVAL SYSTEM OF ANY NATURE WITHOUT THE RAU'S IAS STUDY CIRCLE
WRITTEN CONSENT OF THE COPYRIGHT HOLDER,
RAU’S IAS STUDY CIRCLE
@civilminds

TABLE OF CONTENTS

Part ONE MEETING OF NITI AAYOG


#GOVERNANCE #INSTITUTION #COOPERATIVE

Current Affairs Analysis FEDERALISM 28


NEED TO INCLUDE CASTE IN UPCOMING 2021 CENSUS
#SOCIAL INDICATOR #GOVERNANCE 30

02 LATERAL ENTRY IN INDIAN BUREAUCRACY


#CIVIL SERVICE REFORMS #GOVERNANCE 31
INTERNATIONAL RELATIONS DELHI AGREEMENT OF 1952 AND ARTICLE 35A
# GS Paper (Prelims) & GS Paper II (Main) #CONSTITUTION #AGREEMENT 33

INDIA’S OUTREACH TO SOUTH EAST ASIA THE PROBLEMS WITH THE HECI DRAFT BILL
# FOREIGN POLICY # BILATERAL RELATIONS 02 # BILL # REFORMS (courtesy: The Hindu) 34
INDIA AND INDONESIA 02
INDIA AND SINGAPORE 08
INDIA AND SEYCHELLES
# BILATERAL RELATIONS 10
37
BRICS MINISTERIAL MEET ECONOMIC DEVELOPMENT
# INTERNATIONAL GROUPING 11 # GS Paper (Prelims) & GS Paper III (Main)
IBSA MINING SECTOR: PROSPECTS AND CHALLENGES
# INTERNATIONAL GROUPING 13 #SECTOR IN FOCUS (Lead coverage) 37
SHANGHAI COOPERATION ORGANISATION MARGINAL COST OF FUNDS BASED LENDING RATE
# INTERNATIONAL ORGANISATION 14 (MCLR) #BANKING AND FINANCE 40
ORGANISATION FOR THE PROHIBITION OF CHEMICAL HIGHER TARIFFS ON U.S. IMPORTS
WEAPONS (OPCW) # FOREIGN TRADE 41
# INTERNATIONAL ORGANISATION 16 INFRA FUNDS: AIIB TO INVEST
UN HUMAN RIGHTS COUNCIL # INVESTMENT 42
# INTERNATIONAL ORGANISATION 18 FINANCIAL STABILITY REPORT (FSR)
WORLD GEOGRAPHY IN CONTEXT # FINANCIAL SECTOR 42
#POLITICAL GEOGRAPHY 19 ASSET RECONSTRUCTION OR ASSET MANAGEMENT
COMPANY # FINANCE 43
CREDIT ENHANCEMENT FUND

20 # INVESTMENT 44
NATIONAL DATA CENTRE
CONSTITUTION, POLITY AND GOVERNANCE # SECURITY #DATA PRIVACY 45
# GS Paper (Prelims) & GS Paper II (Main) CHECK ON OUTWARD REMITTANCES

HIGHER EDUCATION COMMISSION OF INDIA # REMITTANCE 45


#BILL #REGULATORY BODY #GOVERNANCE 20 THE SLIDE OF INDIAN RUPEE
MEDICAL COMMISSION AMENDMENT BILL # FINANCIAL MANAGEMENT 46
#AMENDMENT #ACT #GOVERNANCE 22 ONE YEAR OF GST
DAM SAFETY BILL, 2018 # TAX REFORMS 48
#BILL #GOVERNANCE #INFRASTRUCTURE 24 DISCUSSION AROUND ECONOMIC RECOVERY
DNA PROFILE BILL, 2018 # ECONOMY 49
#BILL #GOVERNANCE 26 5G: PROMISES AND CHALLENGES IN INDIA
ASPIRATIONAL DISTRICT PROGRAMME – NITI AAYOG # TELECOM #TECHNOLOGY 51
#GOVERNANCE #COMPETITIVE FEDERALISM 27
@civilminds
OFF-SHORE WIND POWER WATER ICE UNDER THE SURFACE OF THE MOON?
# RENEWABLE ENERGY 52 #SPACE 71
ACTION PLAN OF AGRICULTURAL EDUCATION DIVISON MISSING DARK MATTER BAFFLES SCIENTISTS
# AGRICULTURE #SCHEME 53 #SPACE 72
OFF-GRID AND DECENTRALISED SOLAR PV APPLICATIONS DISCOVERY OF A NEW PLANET
PROGRAMME- PHASE III # ENERGY 54 #SPACE 72
COMMISSION FOR AGRICULTURAL COSTS AND PRICES FARMING IN SPACE
(CACP) 54 #SPACE 73
PRESUMPTIVE TAXATION SCHEME 55 WORLD’S MOST POWERFUL SUPERCOMPUTER
TERM MONEY MARKET 55 #COMPUTING 74
PANEL ON JOBS DATA 55 ‘FLYING BRAIN’ HEADS TO ISS
SHILLONG GETS SELECTED AS THE 100TH SMART CITY 56 #SPACE 74
ADOPT A HERITAGE SCHEME 56 INTEGRATED CHIPS FOR TRAIN ENGINES
SOLAR CHARKHA MISSION 56 #TECHNOLOGY 75
DOPPLER RADARS
#TECHNOLOGY 76
QUADRIVALENT INFLUENZA VACCINE
57 #HEALTH 77

SOCIETY AND SOCIAL JUSTICE NURTURING SCIENTIFIC TEMPER


(courtesy: The Hindu) 77
# GS Paper I & GS Paper II (Main)
TO COMBAT STUNTING
# HEALTH #EMPOWERMENT 57
PROMOTING WOMEN ENTREPRENEURS 79
# INCLUSIVE GROWTH # EMPOWERMENT 59
PRADHAN MANTRI MATRU VANDANA YOJANA (PMMVY) GEOGRAPHY, ENVIRONMENT, BIODIVERSITY &
#SCHEME #SOCIAL JUSTICE #WOMEN EMPOWERMENT 61 DISASTER MANAGEMENT
U.N. REPORT ON GLOBAL HUNGER # GS Paper (Prelims) and GS Paper I & III(Main)
# REPORT #JUSTICE #NUTRITIONAL SECURITY #INCLUSIVE
NITI AAYOG ON WATER CRISIS
DEVELOPMENT 62
#RESOURCE #WATER CRISIS 79
TOOTH PASTE AND ANTIBIOTIC RESISTANCE
ATAL BHUJAL YOJANA
#HEALTH 64 #CONSERVATION #SCHEME 80
SHARP DROP IN SMOKING TOBACCO IN INDIA URANIUM CONTAMINATION OF GROUND WATER
# HEALTH 65 #CONSERVATION #POLLUTION 81
GAMING: ANOTHER ADDICTION ON THE PROWL SWAJAL SCHEME
#HEALTH 65 #CONSERVATION #SCHEME 82
TOWARDS FOOD SECURITY ZERO BUDGET NATURAL FARMING (ZBNF)
# FOOD JUSTICE (Courtesy: The Hindu) 67 #BIODIVERSITY #CONSERVATION 82
WORLD ENVIRONMENT DAY
#ENVIRONMENT #CONSERVATION 84

69 PLASTIC BAN
#POLLUTION #CONSERVATION 85
SCIENCE & TECHNOLOGY CONVENTION ON BIODIVERSITY (CBD): BOON OR BANE
# GS Paper (Prelims) & GS Paper III (Main) #BIODIVERSITY #ENVIRONMENT 86

CATALOGUING MILKY WAY ‘BLUE FLAG’ TAG FOR ODISHA BEACH


#SPACE 69 #CONSERVATION #ENVIRONMENT 88
FARTHEST STAR SPOTTED DRAFTING OF ANTARCTIC POLICY
#SPACE 70 #CONSERVATION #ENVIRONMENT 88
@civilminds
MAITRI IRRIGATION PROJECT
107
#BILATERAL RELATIONS 89
WORLD DAY TO COMBAT DESERTIFICATION AND MISCELLANEOUS
DROUGHT #CONSERVATION 89
TOUR DE FRANCE 2018
STATE FRUIT OF TRIPURA # CYCLING 107
#CONSERVATION 90
WIMBLEDON 2018
DECLINE OF POLLINATORS THREATENS FOOD SUPPLY # TENNIS #GLOBAL TOURNAMENT 108
# CONSERVATION # SUSTAINABLE DEVELOPMENT
CHAMPIONS TROPHY 2018
(courtesy: The Hindu) 90
# HOCKEY #GLOBAL TOURNAMENT 108
FIFA WORLD CUP 2018
# FOOTBALL #GLOBAL TOURNAMENT 109

92 DAUGHTER’S OF THE SUN


# BOOKS AND AUTHORS 109
HISTORY, HERITAGE & CULTURE
# GS Paper (Prelims) & GS Paper I (Main)
CHALUKYAN SCULPTURE
# HERITAGE AND CULTURE 92
111
KAMAKHYA TEMPLE ETHICS, INTEGRITY & APTITUDE
# HERITAGE & CULTURE 93
# GS Paper IV (Main)
RAMANUJA
ARE WE GETTING THE CIVIL SERVICES THAT INDIA WANTS?
# PERSONALITY# CULTURE #MEDIEVAL INDIA 94
#ETHICS #INTEGRITY #CIVIL SERVICE REFORM 111
KABIR
THE PHILOSOPHY OF POWER AND PRESTIGE 113
# PERSONALITY # CULTURE # MEDIEVAL INDIA 95
IAS: THE BELIEF SYSTEM, AND SOME TRUTHS 115
BATTLE OF PLASSEY
# MODERN INDIA 96
RAJA RAMMOHAN ROY
# PERSONALITY # REFORMER #MODERN INDIA 98 117
JAMI MOSQUE, GULBURGA
GLOSSARY
# MEDIEVAL INDIA #CULTURE
# Terms
(Courtesy: The Hindu) 100
GUTENBERG-RICHTER LAW
(# GEOLOGY) 117
VOLITIONAL EVOLUTION

102 (#HUMAN BIOLOGY) 117


HIDING HAND PRINCIPLE
SECURITY (#ECONOMY) 117
# GS Paper III (Main) PROPINQUITY EFFECT
FINANCIAL ACTION TASK FORCE (FATF) (#PSYCHOLOGY) 117
#MONEY LAUNDERING #EXTERNAL SECURITY 102 ANTHROPOCENTRISM
FATF AND PAKISTAN 102 (# PHILOSOPHY) 118
FATF RECOMMENDATION FOR INDIA 102 GAIA HYPOTHESIS
ONLINE MONITORING OF FUNDS UNDER FCRA (# ECOLOGY) 118
#INTERNAL SECURITY 103 RESOURCESHIP
EXERCISE MALABAR – 2018 (# ECONOMICS) 118
# MILITARY EXERCISE 104 THRIFTY GENE
POKHRAN II: TWENTY YEARS LATER (# BIOLOGY) 118
# NUCLEAR TEST # ENERGY #SECURITY INVERSE PROBLEM
(Courtesy: The Hindu) 105 (# SCIENCE) 119
@civilminds
PATENT TROLL
(#ECONOMICS) 119
Part Three
Part TWO practiCe ZONE
CONTRIBUTORS ZONE
136
MULTIPLE CHOICE QUESTIONS (MCQS)
121 for Prelims GS Paper – I

LEAD ARTICLES
DOUBLING OF FARMERS INCOME BY 2022: IS IT A
REALISTIC TARGET? 144
#ECONOMY
ESSAY TYPE QUESTIONS
DR. J. C. SHARMA, FACULTY (GS: ECONOMY AND
OPTIONAL: ECONOMICS) 121 for Mains GS Papers – I, II & III

THE WATER USE EFFICIENCY IN AGRICULTURE


#ECONOMY #ENVIRONMENT
HIRALAL MANDAL, EDITORIAL TEAM 163
& EDUCATOR 125
CASE STUDIES
INDIA AND OBOR
for Mains GS Paper IV: Ethics, Integrity & Aptitude
#INDIA AND THE WORLD
RITESH KUMAR SINGH, FACULTY: G.S. CONTEMPORARY
ISSUES, ANSWER WRITING & ESSAY, AND EXECUTIVE
EDITOR 128

132
LEAD ESSAYS
MARITIME SECURITY
TUSHAR, GS-2A 132
@civilminds

Part ONE

Current
affairs
analysis
logical . simple . targeted
analysis & explanation
of all relevant news of the month
@civilminds

INTERNATIONAL RELATIONS
# GS Paper (Prelims) & GS Paper II (Main)

INDIA’S OUTREACH TO SOUTH EAST ASIA


# Foreign Policy # Bilateral relations

India follows ‘Act East Policy’(AEP) for its


relation with South East Asia.
This was upgraded from the ‘Look East Policy’ in
2014 at the India-ASEAN Summit held in Nay Pyi
Taw in Myanmar
It focusses on the extended neighbourhood in
the Asia-Pacific region.
The Objective of AEP is to promote economic
cooperation, cultural ties and develop strategic
relationship with countries in the Asia-Pacific
region through continuous engagement at
bilateral, regional and multilateral levels.
The North East of India has been a priority in
AEP. It provides an interface between North East
India including the state of Arunachal Pradesh
and the ASEAN region.

Recently, the Prime Minister of India visited South East Asian countries of Indonesia, Malaysia and Singapore.
The Prime Minister signed a Joint Vision of Indo-Pacific with Indonesia and outlined the contours of India’s
foreign policy and engagement within the larger Indo-Pacific region at the Shangri La Dialogue in Singapore.
Let us understand the various dimensions of the visit and India’s engagement with South East Asia.

INDIA AND INDONESIA

x The main theme of India’s visit to Indonesia was ‘Establishing


Comprehensive Strategic Partnership between Indonesia and
India’.
x The two countries are maritime neighbours wherein the distance
between India and Indonesia is just 90 nautical miles.
x Both countries affirmed that both countries share a deep
civilization links that extends millennia wherein both countries
have common cultural linkages on the basis of Hinduism,
Buddhism and Islam heritage.
x Both countries also shared common modern values of upholding
democracy, pluralism and diversity.
x Both countries are also rising economies and can have strong
complementarities to each other in economic cooperation wherein
Indonesia is India’s largest trading partner in ASEAN.

FOCUS | July 2018 | RAU’S IAS 2


@civilminds
International Relations

x Both countries intend to enhance their strategic partnership of forming continued bilateral
dialogue process, wherein both countries would conduct:
Dialogue
9 Annual Summit meetings, including on the margins of multilateral events;
Process
9 Regular bilateral consultations through Ministerial and Working Group Mechanisms in field of
trade, energy, security and defence

x India and Indonesia agreed to strengthen cooperation in all areas by establishing a New
Strategic Comprehensive Strategic Partnership.
Partnership x The basic contours of the framework of the New Comprehensive Strategic Partnership would be
evolved by their respective Foreign Ministers in given time.

Æ Defence and Security Cooperation

x Signed the Defence Cooperation Agreement that intends to further strengthen and renew the
existing cooperation in defence between both countries.

x Both countries would constitute regular bilateral dialogue and consultation on strategic
defence and military issues of common interest, exchange of strategic information, military
Defence
education, training and exercise, cooperation among the Armed Forces including Army, Navy, Air
Cooperation
Force and Aerospace, humanitarian aid, disaster relief, peace-keeping and medical services among
others.

x Both countries also endorsed upon the Finalization of Standard Operational Procedure by both
Navies and Air Forces for joint military exercises and training.

Both agreed to strengthen existing institutional dialogue mechanisms for bilateral defence
cooperation. The institutional dialogue mechanisms are:

9 Security Dialogue

9 Biennial Defence Ministerial Dialogue


Institutional
9 Joint Defence Cooperation Committee
Dialogue
9 Meetings & staff talks between respective armies, navies & air forces

The First Security Dialogue between both countries was held in January, 2018. Both countries
reaffirmed their intention to enhance bilateral cooperation in countering terrorism, terrorist financing,
money laundering, arms smuggling, trafficking in persons and cybercrime.

India and Indonesia would enhance liaison between the intelligence and law enforcement agencies to
Intelligence
collaborate and exchange information under the Mutual Legal Assistance in Criminal Matters
Sharing
agreement.

x Both condemned terrorism in all its forms and manifestations including cross-border terrorism
and terror-related incidents in Indonesia and India.

x Both countries also affirmed that terrorism should not be associated with any religion, creed,
Against
nationality and ethnicity.
Terrorism
x Both countries have asserted that all countries should work towards disrupting terrorist networks
and their financing channels, and halting cross-border movement of terrorists from terror
organizations.

FOCUS | July 2018 | RAU’S IAS 3


@civilminds
International Relations

x Both countries asserted for expeditious finalization and adoption of the Comprehensive
Convention on International Terrorism in the United Nations.

Æ Economic Cooperation and Connectivity

x Indonesia exports Palm oil to India. Both countries agreed to address all issues related to obstacles
Palm Oil to trade and Indian investment in palm oil products and industries in Indonesia.
Trade x Palm oil is used as a simple frying oil. Used by consumer retail food and snack manufacturers;
personal care and cosmetics; biofuel and energy.

x Both are under discussion for potential cooperation in the area of peaceful use of nuclear
Nuclear energy.
Energy x Are looking forward to the early renewal of an Agreement on Cooperation regarding the
Utilization of Nuclear Energy for Peaceful Purposes.

x Both agreed to explore further cooperation in refineries technology, gas infrastructure, sharing of
Energy experience in the use of LEDs and renewable energy in India, sharing the expertise of Indonesia in
Cooperation gasification of fuel oil.

x India may also undertake exploration of oil, gas and coal based on Indonesian existing regulations.

x Both signed an MoU on Health Cooperation that would pave the way for closer collaboration in
tackling common health challenges.
Health
x Both countries intend to develop a roadmap on health cooperation that would strengthen further
Cooperation
collaborations in the field of pharmaceuticals, biological products, cosmetics, research and
development, active pharmaceutical ingredients (API) and IT-based medical equipment.

x Both emphasized the need for establishing direct air connectivity between the major cities of both
Air countries.
Connectivity x The civil aviation authorities of both countries would discuss further for enhancing the traffic rights
through bilateral air services consultation.

Maritime Both plan to build naval connectivity such as for cruise tourism between Andaman & Nicobar to Aceh
Connectivity in Indonesia.

Æ Cultural, Technological and Educational Cooperation

x Both intend to enhance cooperation between their respective educational institutions and
therefore would negotiate the renewal of an earlier MoU on Cooperation in the Field of
Educational
Education and finalize the MoU in the Field of Higher Education.
Collaboration
x Both countries would also increase the number of scholarship offers provided to students from
the other country.

x Both signed an MoU in the Scientific and Technological Cooperation.

x Both countries concluded the Inter-Governmental Framework Agreement on Cooperation in


Technological
the Exploration and Uses of Outer Space for Peaceful Purposes.
Collaboration
x Both countries would augment cooperation in space sciences, exploration of outer space, use of
space technology, monitoring of Earth’s environment from outer space and remote sensing and

FOCUS | July 2018 | RAU’S IAS 4


@civilminds
International Relations

joint research and development activities in space technology

x Indonesia would host a new Indian Ground Station where research and development activities
would jointly be conducted by both sides.

x Both countries would use Integrated Biak Ground Stations in Indonesia for mutual benefit.

x India would support for launch services of National Institute of Aeronautics and Space of the
Republic of Indonesia (LAPAN) made satellites

x Both initiated the World Heritage Twinning Program between both countries with twinning of
Candi Prambanan and Taj Mahal.
World
x Candi Prambanan is a 9th-century Hindu temple compound in Central Java, Indonesia, dedicated
Heritage Sites
to the Trimurti i.e. the expression of God as the Creator (Brahma), the Preserver (Vishnu) and
the Transformer (Shiva).

x Both have agreed to organize an interfaith dialogue in Indonesia in 2018 and in India in 2019.

x Interfaith dialogue is a reflection of the united commitment of both countries to explore a new
Interfaith understanding on the rise of religious identities, the dynamic of pluralism and to eradicate
Dialogue radicalism, terrorism, and violent extremism.

x Both countries would strengthen the ideals of pluralism, tolerance, rule of law and values of
peaceful co-existence through this dialogue.

'Balijatra' festival is celebrated every year in the month of October-November in Odisha. Balijatra
Balijatra means ‘Journey to the Bali of Indonesia’, wherein hundreds of years ago sailors from Odisha
would sail from Cuttack to visit Java-Sumatra and Borneo in today’s Indonesia.

Æ Global Convergences

Both reaffirmed the unity of ASEAN was central to the evolving regional architecture and its
ASEAN
continued contribution to regional peace, security, and prosperity.

x India, Indonesia and Australia would have their first Trilateral Strategic Dialogue in
Trilateral November, 2018.
Dialogue x Both countries reaffirmed the importance of open and frank dialogue as well as exchange of best
practices among like-minded democracies.

x Both reaffirmed their commitment to achieve the Sustainable Development Goals (SDGs) and the
full and timely implementation of the Addis Ababa Action Agenda.
Sustainable
x The Addis Ababa Action Agenda adopted in 2015, provides a comprehensive set of policy actions
Development
by Member States, with a package of concrete measures to finance sustainable development,
transform the global economy and achieve the Sustainable Development Goals.

x India and Indonesia reaffirmed their commitment to work together for the effective
implementation of the Paris Agreement as a global effort to combat climate change.
Climate
x Both countries also emphasized the importance of adequate and predictable financial resources,
Change
capacity building and technology transfer to support mitigation and adaptation actions in
developing countries.

FOCUS | July 2018 | RAU’S IAS 5


@civilminds
International Relations

Æ Joint Vision of Indo-Pacific

x India occupies a central position in the Indo-Pacific with a coastline of ~ 7,500 kms, with more than
1,380 islands and more than two million sq. km of Exclusive Economic Zone.
x Indonesia as the largest archipelagic State in the world, with a coastline of 108,000 kms, 17K plus
Location
islands and features, and a total of maritime areas of 6,400,000 sq. km including Exclusive
Economic Zone.
x Indonesia is the fulcrum that connects the Indian Ocean and Pacific Ocean.

Both reaffirmed the rights and obligations under the international law under the:

Governing 9 Charter of the United Nations;


treaties 9 1982 UN Convention on the Law of Sea;
9 1976 Treaty of Amity and Cooperation in Southeast Asia.

Both affirmed a common vision for Indo-Pacific with:


9 Achieving a free, open, transparent, rules-based, peaceful, prosperous and inclusive Indo-Pacific
region;
9 Respect of sovereignty and territorial integrity, freedom of navigation and over flight, international
law, in particular United Nations Convention on the Law of the Sea (UNCLOS);
Common
9 Promoting sustainable development and an open, free, fair and mutually beneficial trade and
Vision
investment system;
9 Maintaining maritime safety and security for peace and stability;
9 Reaffirming the importance of the ASEAN centrality and unity in Indo-Pacific regional architecture;
9 Both countries would converge and cooperate based upon their respective polices with India’s Act
East Policy and Security and Growth for all in the Region (SAGAR), and Indonesian Ocean Policy

FOCUS | July 2018 | RAU’S IAS 6


@civilminds
International Relations

and Indonesia’s Global Maritime Fulcrum Vision.


Note: India had recently signed a similar with France – ‘Joint Strategic Vision of India-France
Cooperation in the Indian Ocean Region.’ Refer to the April edition of Focus to further understand
about the agreement.

x Both will expedite for mutually acceptable solutions on the delimitation of maritime boundaries
relating to EEZ in the area between the Great Nicobar, India and Sumatra, Indonesia in the
Maritime
Andaman Sea.
Delimitation
x The delimitation is based on the principles of international law including the 1982 United Nations
Convention on the Law of the Sea.

x Both signed a Statement on Maritime Cooperation in 2016 and would expedite the finalization
of the MoU on Maritime Security Cooperation between Badan Keamanan Laut (Indonesian
Coast Guard
Maritime Security Agency) and the Indian Coast Guard.
Cooperation
x The MoU will institutionalize training cooperation, capacity building, regular working group,
coordinated patrol and joint exercises.

x Both highlighted the importance of reaching the goals set in the Jakarta Concord and the IORA
Action Plan 2017 – 2021 endorsed during the IORA Leaders’ Summit 2017.
IORA is an inter-governmental organisation which was established in 1997 to enhance
IORA
cooperation among the member-states whose shores are on the rim of the Indian Ocean.
x Both countries would further intensify cooperation for a safe and secure Indian Ocean within the
framework of the Indian Rim Association Organisation.

x Both have asserted that the development of the Indo-Pacific concept must also be done in an
open, transparent, and inclusive manner, respect for the international law and uphold ASEAN
ASEAN
Centrality.
Centrality
x Both countries would strengthen the existing security architecture in the Indo-Pacific which is
anchored in ASEAN-led mechanisms.

Both would address the emerging maritime security issues facing the Indo-Pacific region, which are:

Maritime 9 Smuggling of people, arms, drugs and money;


Concerns 9 Illegal, unreported and unregulated fishing;
9 Movement of terrorists.

Both would enhance cooperation in:


9 Science-based management and conservation of marine living resources;
9 Combating climate change and ensuring protection of environment and natural resources;
9 Combating, preventing, deterring and eradicating illegal, unregulated, and unreported (IUU) fishing;
Marine
9 Recognizing crimes related to fisheries crime as one of the emerging crimes, which led marine
resources
environmental degradation;
9 Promoting Blue Economy as a key source of inclusive economic growth and job creation;
9 Combating marine plastic debris through bilateral and regional cooperation;
9 Pursuing bilateral cooperation in the areas of hydrography and marine cartography.

FOCUS | July 2018 | RAU’S IAS 7


@civilminds
International Relations

INDIA AND SINGAPORE

Æ Strategic Cooperation

x Both signed the Implementing Arrangement between the Navies of both countries for mutual
Defence coordination, logistics and services support.
Cooperation x Both countries would expedite a similar Army Bilateral Agreement, for maintaining a high level of
annual exercises, goodwill visits and professional exchanges.

x Singapore acknowledged India’s leading role in the Indian Ocean region.


Maritime x Singapore also agreed to India’s proposal for continuous and institutionalised naval engagements in
Cooperation their shared maritime space including establishing maritime exercises with like-minded regional /
ASEAN partners.

x Both emphasised the importance of international cooperation in tackling transnational security


threats and in particular measures to collectively tackle the threat of terrorism.
x Both countries reiterated their commitment to combat global terrorism, terror organisations and
networks and all those who encourage, support and finance terrorism.
Against x Both countries stressed that there can be no justification for acts of terror on any grounds
Terrorism whatsoever.
x Both countries intend for an early conclusion of the Comprehensive Convention on International
Terrorism in the United Nations.
CCIT was first proposed by India in 1996 to develop a universal definition of terrorism, to ban all
terrorist groups and to make cross-border terrorism an extraditable offence.

Æ Economic Cooperation

Basic Both concluded the Second Review of the Comprehensive Economic Cooperation Agreement,
Framework which provides the foundation for bilateral economic cooperation.

Both consider Financial Technology (Fintech) and innovation as the new driver of their bilateral
economic partnership.
Both countries established a Joint Working Group (JWG) on Fintech with relevant government
agencies on both sides. It would promote:

Fintech 9 Knowledge exchange in best practices;


Cooperation 9 Encourage cooperation between Fintech firms;
9 Develop Application Programming Interfaces to promote interoperability between both markets;
9 Increase cross-border linkages between payment systems in Singapore and India;
9 Internationalisation of the India Stack which an innovative infrastructure that enables digital
services delivery in India.

Both announced a partnership between Network for Electronic Transfers (NETS), Singapore and
National Payments Corporation of India. The partnership intends to:

International 9 Launch the RuPay card in Singapore;


Payment 9 Extend Bharat Interface for Money (BHIM) for payment in Singapore;
System 9 Future arrangements to support cross-border payments;
9 Both countries supported the international launch of India’s Unified Payments Interface (UPI)
based cross-border remittance app in Singapore, which will facilitate remittances by Indian

FOCUS | July 2018 | RAU’S IAS 8


@civilminds
International Relations

workers.

x Both reaffirmed their support for the development of ASEAN Smart Cities Network.
x Both countries intend for conclusion of the Urban Planning and Governance Programme
Urban
organised by the Centre for Liveable Cities to train 100 Indian officials in Singapore.
Infrastructure
x Singapore and Maharashtra would further collaborate opportunities in Maharashtra’s urban
infrastructure, port and logistics, and aviation sectors.

x Both signed agreements in areas such as investment promotion, start-ups, Fintech, innovation,
skills, artificial intelligence, data analytics and water.
x Both countries support an Indian initiative India-Singapore Entrepreneurship Bridge
Business (InSpreneur), to connect government agencies, start-ups, innovators, incubators, accelerators
Cooperation and investors in the two countries.
x Both countries formed the Singapore-India Incubation Programme. The programme, led by
Enterprise Singapore, aims to provide exposure and support for start-ups keen to explore both
markets.

x Both acknowledged the opportunities in Third Country Cooperation.


x Both countries would cooperate in third countries based upon their strategic partnership and
Third Country
economic complementarities.
Cooperation
x The possible areas of cooperation in third countries would include digital connectivity and
platforms, skills enhancement, human resource development and basic infrastructure.

Æ Global Convergence

x Both would work together to maintain peace, stability, growth and prosperity in the region and
beyond.
x Both sides also reiterated their support for security, safety and freedom of navigation and over
flight, and unimpeded commerce, in accordance with international law, including the 1982
Indo-Pacific UNCLOS.
Region x Both countries asserted that all parties to resolve disputes through peaceful means without
threat or use of force and exercise self-restraint in the conduct of actions that could escalate
tensions in the region.
x This is seen as an in-direct reference to China and its growing animosity with countries located in
South China Sea over boundary disputes.

Singapore’s reaffirmed its continuing support for India as a permanent member in a reformed
UN Reform
United Nations Security Council.

x India expressed its full support for ASEAN's unity, Centrality and leadership in the evolving
ASEAN
regional architecture.
Centrality
x Singapore reaffirmed that India has a vital role in the regional architecture.

SHANGRI LA DIALOGUE
The IISS Asia Security Summit was launched in 2002 by British think tank, the International Institute for Strategic
Studies and the Singaporean government. This annual dialogue brings together policy and opinion makers from
countries of Asia-Pacific region to talk about security in the region. It derives its name from the location of the
meeting, the Shangri-La hotel in Singapore.

FOCUS | July 2018 | RAU’S IAS 9


@civilminds
International Relations

MESSAGE FROM INDIAN PRIME MINISTER ON INDIAN FOREIGN POLICY

x India is evolving its regional role to encompass the Indo-Pacific and Eurasian region to ensure a balanced multipolar
world;
x India would continue to maintain its strategic autonomy irrespective of the prevailing geo-politics;
x India would continue to maintain interest based relationship with the great powers;
x India would ensure stability in power relations in Asian region among the great powers;
x India will not be a part of closed group of nations or aggregate Indian power in any particular bloc;
x India's friendship with any nation should not be mis-construed as an alliance;
x India's growing engagement in the Indo-Pacific region should not be considered anti-China;
x India would not base relations with other countries based on ideological differences;
x India would pro-actively focus on forming a 'common-rules based' international order;
x The security architecture for Indo-Pacific region should be based on consensus among all stakeholders to ensure a
'free, open and inclusive region;
x India continues to maintain multi-lateralism as form of foreign policy engagement with parallel interactions (Eg. SCO
& Quad) based on respective common mutual interests
x India's pursuit of a multi-polar world reflects such as by reaffirming the centrality of ASEAN in the Indo-Pacific.

INDIA AND SEYCHELLES


# Bilateral relations

Recently, the President of Seychelles visited India.


Seychelles is an archipelago of 115 islands in the Indian
Ocean, off East Africa. It has strategic location and is
important for maintaining maritime security.
India and Seychelles signed 7 MoUs during the recent
visit, mainly:
9 Indian Grant Assistance for Implementation of Small
Development Projects through Local Bodies,
Educational and Vocational Institutions
9 Cooperation in the area of Cyber Security
9 Cultural Exchange Programme
9 Sharing White Shipping Information

ISSUE OF ASSUMPTION ISLAND

x Assumption Island is a part of Seychelles and is located in the north of Madagascar, near the
Assumption Mozambique Channel.
Island x Seychelles and India signed a 20-year agreement in January, 2018 that will enable India to
construct military infrastructures on Assumption Island.

Issue with The Seychelles National Assembly, in-particular the opposition, has asserted that it will not ratify the
Agreement agreement.

FOCUS | July 2018 | RAU’S IAS 10


@civilminds
International Relations

The main reason for its non-ratification is that:


9 It would infringe on the sovereignty of Seychelles.
9 It would un-necessarily involve Seychelles in the competition between India and China in Indian
Ocean region.

x The agreement currently stands in suspended animation.


x India has to wait and see on how future government in Seychelles move forward with the
Future agreement.
Expectations x Seychelles has asserted that it will form the required military infrastructure on the island by itself.
x India has the potential to conduct soft diplomacy with providing monetary and material
requirement to Seychelles to form military infrastructure on the island.

Î RELATED INFORMATION: NEARBY MILITARY INFRASTRUCTURE

x Reunion islands is overseas territory of France in the Indian Ocean Region. It is situated at eastern
Reunion coast of Madagascar, and is near the Assumption Island of Seychelles.
islands x The recent logistics agreement with France will allow India access to the French base in the
Reunion Islands.

India had activated its first naval assets as a listening post in Madagascar in 2007. The naval station was
Madagascar
also India's first in southern Indian Ocean.

Agalega Agalega agreement was signed between India and Mauritius in 2015. Both countries are undertaking
agreement an infrastructure and logistics development of Agalega Island.

Chagos UK had split the Chagos archipelago from Mauritius before its independence in 1968. Thereafter, UK had
Archipelago leased Diego Garcia to the United States and the US had formed its Indian Ocean military base.

Recently, the respective ministers handling (Foreign Affairs/


BRICS MINISTERIAL
International Relations portfolios) in Brazil, Russia, India, China
MEET and South Africa met in June 2018. On this note, let us understand
about BRICS along-with stated global outlook at the Ministerial
# International Grouping th
meet and also about the upcoming 10 BRICS Summit.

THINGS TO UNDERSTAND

The acronym BRIC, (Brazil, Russia, India, China) was first used in 2001 by Goldman Sachs in a
paper that projected that the four economies would individually and collectively occupies far
Background
greater economic space and would be amongst the world’s largest economies in the coming
decades.

1. The grouping was formalized with the first meeting of BRIC Foreign Ministers in September,
2006.
2. The BRIC’s first summit was held in Russia in 2009.
Summit 3. A decision to expand BRIC into BRICS with the inclusion of South Africa was taken at BRIC
Millstones Foreign Ministers’ meeting in 2010. In the 3rd BRICS Summit at Sanya in 2011, South Africa
became member of expanded BRICS.
4. The last summit to be held in India was the 8th BRICS Summit in 2016 which culminated in the
‘Goa Declaration’. India also invited the member countries of BIMSTEC, as part of BRICS

FOCUS | July 2018 | RAU’S IAS 11


@civilminds
International Relations

outreach program.
5. The last 9th BRICS Summit was held in China in 2017 which culminated in the Xiamen
Declaration.

x 2018 marks a decade of BRICS with the upcoming 10th BRICS Summit in South Africa with
focussed theme of ‘BRICS in Africa’.

10th BRICS x The 10th Summit continues the BRICS tradition of outreach to extend its cooperation to
Summit fellow developing and emerging economies.
x In this respect, South Africa would have a two-pronged outreach approach through the BRICS-
Africa Dialogue and the BRICS Plus cooperation, to be held during the Summit.

Global Outlook of BRICS

x BRICS members have asserted for a comprehensive reform of UN Security Council to make it
more representative with an increase in the representation of the developing countries so that it
UNSC
can adequately respond to global challenges.
Reforms
x China and Russia reiterate the importance of increased role of Brazil, India and South Africa in
international affairs and support their aspiration to play a greater role in the UN.

x BRICS reaffirmed their support to the United Nations in coordinating and reviewing global
implementation of the 2030 Agenda for Sustainable Development.
x They also extended support for the need to reform the UN Development System with a view to
2030 Agenda enhancing its capability in supporting member States in implementing the 2030 Agenda.
x BRICS asserted for developed countries to honour their Official Development Assistance
commitments in time and in full and provide more development resources to developing
countries.

x BRICS members reaffirmed their commitment to multilateralism and a rules-based international


order with the centrality of UN, WTO and international law.
x They underlined their firm commitment to free trade, and the centrality of a rules-based,
WTO and transparent, non-discriminatory, multilateral trading system as embodied in the WTO.
Global Trade x They opposed the new wave of protectionism and the systematic impact of unilateral measures
that are incompatible with WTO rules, and undermine global trade and economic growth.
x They reiterated that the WTO Dispute Settlement System is a cornerstone of multilateral trading
system as it is designed to enhance security and predictability in international trade.

x BRICS members intend for the International Monetary Fund to complete the 15th General Review
of Quotas and agree upon a new quota formula as a basis for a realignment of quota shares.
They asserted for the IMF quota system to reflect an increased shares of emerging economies as
per their position in the world economy.
Note: IMF member countries are each assigned a quota — a value of its share in the IMF
financing system that is tied to its impact on the world economy. A country's quota at the IMF
IMF and NDB determines its voting power, the amount of financial resources it must provide to the IMF and its
access to IMF financing.
x They intend for increase institutional presence of the New Development Bank across all major
economic regions of the world. There is an upcoming establishment of the Americas Regional
Office in Brazil, with the earlier Africa Regional Centre (ARC), will help the NDB consolidate its
presence in respective continents.
New Development Bank (NDB) was formally referred as the BRICS Development Bank. It is a

FOCUS | July 2018 | RAU’S IAS 12


@civilminds
International Relations

multilateral development bank established by the BRICS states (Brazil, Russia, India, China and
South Africa).

x BRICS members reiterated the need for renewed diplomatic efforts to achieving a just, lasting and
comprehensive settlement of the Israeli-Palestinian conflict.
x They asserted for the basis for settlement to be focussed upon relevant United Nations
Israel and
resolutions, the Madrid Principles, the Arab Peace Initiative and previous agreements between
Palestine
the parties.
x They supported to create an independent, viable, territorially contiguous Palestinian State living
side by side in peace and security with Israel.

x BRICS members assert for cession of hostilities and to return to the UN brokered peace talks.
Yemen Crisis x They asserted for an inclusive Yemeni-led dialogue towards the achievement of a political
solution to the conflict.

x Asserted for a solution to the crisis in Syria through a "Syrian-led, Syrian-owned” inclusive
political process which safeguards the sovereignty, independence and territorial integrity of Syria.
Syrian Crisis x Moreover, they support the Geneva Peace Talks and the Astana process for end to Syrian
conflict.
x The Ministers reiterated their strong condemnation of the use of chemical weapons by any party.

BRICS members have asserted that all relevant parties of the Joint Comprehensive Plan of Action on
Iranian Deal
the Iranian nuclear issue fully comply with their obligations and ensure full and effective
Issue
implementation of the JCPOA.

Recently, IBSA – India, Brazil and South Africa signed a Declaration on


IBSA South-South Cooperation. On this note, let us understand about IBSA
# International Grouping and the difference highlighted by IBSA between South-South
Cooperation and Official Development Assistance.

THINGS TO UNDERSTAND

x The India-Brazil-South Africa forum was established in 2003 by the Brasilia Declaration and
intends for South-South cooperation.
x IBSA brings together India, Brazil and South Africa, three large democracies and major developing
economies from three continents.
Background
x It is a forum for consultation and coordination on global and regional political issues and
collaborates on international projects.
x IBSA countries would assist other developing countries by taking up projects in the latter through
IBSA Fund.

IBSA is bound together by a shared conviction in:


9 Universal values of democracy

Guiding 9 Plurality, diversity human rights


Principles 9 Rule of law
9 Commitment to sustainable development
9 Inclusivity of all communities and gender

FOCUS | July 2018 | RAU’S IAS 13


@civilminds
International Relations

9 Respect for international law

x The IBSA Facility Fund for Alleviation of Poverty and Hunger (IBSA Fund) was created in 2004 and
became operational in 2006.
x It aims at to support projects in developing countries in particular Least Developed Countries
IBSA Fund (LDCs) and Post Conflict Reconstruction and Development (PCRD) countries.
x The IBSA Fund is managed by the United Nations Office for South-South Cooperation (UNOSSC),
which lends its professional expertise to multiple stakeholders in promoting the development of
the Global South.

Æ IBSA on South-South Cooperation and Official Development Assistance

IBSA’s 2007 Tshwane Declaration, 2008 Delhi Declaration and 2010 Brasilia Declaration affirm that
Guiding
South-South Cooperation as a common endeavour of the Global South guided by equality, non-
Principles
conditionality, non-interference in domestic affairs, and mutual benefit.

x The Brasilia Declaration of 2010 states that SSC is not aid and developing countries engaged in SSC
are not donors and recipients but developing partners and have been incorporated in IBSA funded
SSC and projects.
ODA x Official development assistance (ODA) is a term coined by Organisation for Economic Co-operation
and Development (OECD) to measure aid such as in form of soft loan, etc. for economic
development for recipient country.

ODA SSC

ODA is obligatory in nature for developed SSC is voluntary in nature.


countries.

ODA is a supply driven process whereby it SSC is a demand driven process whereby it is the partner countries
is the donor country that determine the that determine the priorities in IBSA projects.
projects to fund.

ODA creates ‘new dependencies’ between SSC rests primarily in creating inter-dependence with recipient
donor and recipient country. country initiate, organise and manage activities on projects.

ODA mainly implements projects with SSC relies on economic and political non-conditionality.
economic and political conditionalities on
recipient countries.

SHANGHAI
COOPERATION India participated for the first time as a full member at the
recently held 18th Qingdao Summit of SCO in China. On this note,
ORGANISATION let us understand about SCO and the advantages of SCO for India.

# International Organisation
THINGS TO UNDERSTAND

Background x The Shanghai Cooperation Organisation is a permanent intergovernmental international

FOCUS | July 2018 | RAU’S IAS 14


@civilminds
International Relations

organisation that was formed in 2001 in Shanghai, China by Kazakhstan, China, Kyrgyzstan, Russia,
Tajikistan and Uzbekistan.
x Prior to SCO formation, all of the above countries except for Uzbekistan were members of the
Shanghai Five.

x Eight countries are currently full members: Kazakhstan, China, Kyrgyzstan, Russia, Tajikistan,
Uzbekistan with India and Pakistan becoming the latest full members in 2017.
Members x Four countries have an observer status: Afghanistan, Belarus, Iran and Mongolia
x Six countries have dialogue partner status: Azerbaijan, Armenia, Cambodia, Nepal, Turkey and
Sri Lanka.

x SCO decided to admit India and Pakistan in Ufa summit of 2015.


Full
x The formal process of India and Pakistan joining SCO began at the Tashkent summit in 2016.
Membership
of India x It was at the Astana summit in 2017 that both India and Pakistan officially became full members of
SCO.

The main goals of SCO are:


9 Strengthening mutual trust and neighbourliness among the member states;
9 Promoting their effective cooperation in politics, trade, the economy, research, technology and
Objectives
culture, education, energy, transport, tourism, environmental protection, and other areas;
of SCO
9 Making joint efforts to maintain and ensure peace, security and stability in the region;
9 Moving towards the establishment of a democratic, fair and rational new international political and
economic order.

x Regional Anti-Terrorist Structure (RATS) of SCO is a permanent body based in Tashkent, Uzbekistan.
x The objective of RATS is based upon the Shanghai Convention on Combating Terrorism,
SCO RATS
Separatism and Extremism.
x RATS possess information on terrorist organisations and terrorists.

Æ India and SCO Membership – Advantages

x SCO can provide India diplomatic support for Chabahar port. India is currently heavily invested in
Chabahar port in Iran which provides India access to Central Asia, but India’s engagement in
Chabahar
Chabahar port may face hurdles due to US sanctions after its withdrawal from Iran nuclear treaty.
Port
x Support of SCO members to continue conduct trade through the Iranian port despite US sanctions
provides India with extensive diplomatic guarantee.

x SCO is a major regional international platform that provides India diplomatic access to Central
Diplomatic Asian countries and to the countries of Commonwealth of Independent States (CIS).
Access x A gap had formed after the collapse of the Soviet Union in India’s economic and diplomatic
interactions with CIS member countries.

x SCO members, other than China, consider India’s membership as a means to balance China in SCO.
Balancing
x China is the largest economic power in SCO, and India’s membership provides Russia and other
China
members’ ability for balancing China, since India is now the 2nd largest economic power in SCO.

FOCUS | July 2018 | RAU’S IAS 15


@civilminds
International Relations

x SCO provides a rare platform for interaction with Pakistan. SCO as a platform is unlikely to be
Interaction affected by India-Pakistan rivalry unlike SAARC.
with Note: The upcoming military exercise – Peace Mission 2018 in Russia would make SCO the 1st
Pakistan platform (After UN Peace Mission Peace Keeping Force) for joint military engagement between India
and Pakistan.

x Membership to SCO provides India access to database of RATS, which contains information of
RATS multitude of terrorist organisations & terrorists.
Database x The Qingdao declaration has also asserted for creation of a unified global counter-terrorism front
under United Nations on the same framework as RATS, which India supports.

x Afghanistan currently has observer status and has been pushing for membership to SCO, which
Afghan India supports.
Membership x Afghanistan’s membership would provide India an ability to pressurise Pakistan through SCO to
open access for India to among the other members of SCO.

x Members of SCO which are mainly authoritarian and least democratic state provides an alternative
Alternative
view to current Western dominated models of global governance.
Governance
x SCO provides a democratic India a unique opportunity as a bridge to both models of global
Model
governance.

CONCERN

x Qingdao Declaration did not mention any terrorist groups wherein India was unable to include Pakistan based
terrorist groups in Qingdao Joint statement.
x This is mainly seen as a result of Pakistan also being a member of SCO and reluctance of their members to isolate
and target Pakistan, whereas China, India & Russia did mention and condemn terrorist groups in Pakistan in the
BRICS Xiamen Declaration in 2017.

The Organisation for the Prohibition of Chemical Weapons is the


ORGANISATION implementing body of the Chemical Weapons Convention (CWC),
FOR THE which entered into force in 1997. OPCW currently has 193 Member
States.
PROHIBITION OF Recently, India voted against a UK initiated and West-backed draft

CHEMICAL decision to expand powers of OPCW at the Fourth Special Session


of the Conference of Parties (CoP) of OPCW held in The Hague,
WEAPONS (OPCW) Netherlands.
On this note, let us understand about OPCW and the global
# International Organisation
context and reason behind India’s vote at OPCW.

THINGS TO UNDERSTAND

The mission of the OPCW is to implement the provisions of the Chemical Weapons Convention in
order to achieve the vision for a world that is free of chemical weapons and of the threat of their use.
The fundamental principles of OPCW are:
Mission
9 Centrality of the CWC’s multilateral character
9 Equal application of the provisions of the CWC to all members

FOCUS | July 2018 | RAU’S IAS 16


@civilminds
International Relations

The main broad aims of OPCW are:


9 To ensure a credible and transparent regime for verifying the destruction of chemical weapons;
9 To prevent the re-emergence of chemical weapons while protecting legitimate national security and
proprietary interests;
Objectives
9 To provide protection and assistance against chemical weapon;
9 To encourage international cooperation in peaceful uses of chemistry;
9 To bring about universal membership of the OPCW;
9 Preventing chemistry from ever again being used for warfare.

The CWC requires State Parties not to develop, produce, acquire, stockpile or retain, transfer, use, or
make military preparations to use chemical weapons. It entered into force in 1997.
The main objective of CWC are:
Chemical
9 Destroying all existing chemical weapons under international verification by the OPCW;
Weapons
9 Monitoring chemical industry to prevent new weapons from re-emerging;
Convention
9 Providing assistance and protection to States Parties against chemical threats;
9 Fostering international cooperation to strengthen implementation of the Convention and promote
the peaceful use of chemistry.

RECENT HAPPENINGS

x The Salisbury incident refers to the poisoning of Sergei Skripal and his daughter, wherein Sergei
Skripal is a former Russian military officer and who acted as a double agent for the UK's
Salisbury intelligence services.
Incident and x Both were poisoned using a chemical nerve agent in Sailsbury, UK, for which UK and other
Syria Western powers have blamed as an act of Russia.
x The Syrian conflict has seen regular usage of Chemical agents in warfare, for which there has
been counter-blame between Russia supported Assad Regime and the rebel Syrian groups.

x OPCW has earlier sent investigators to determine whether chemical attacks have taken place
such as recently in Sailsbury, but it does not apportion blame for the use of chemical agents.

Chemical x In 2015, the United Nations created a system for investigating chemical weapons attacks with
Attack the help of the OPCW and saying who was to blame for them.
Investigations x The Joint Investigative Mechanism drew criticism from Russia after blaming the Assad led
Syrian government for using chemical weapons in the Syrian conflict.
x The mandate for the mechanism expired in 2017 and Russia vetoed an attempt to renew it.

x The Western powers especially UK after the Sailsbury attack intend for ‘perpetrator identifying’
mechanism to be re-initiated.
x UK has asserted that the expansion of the powers of the OPCW technical secretariat was a
reaction to the Russian veto of a draft resolution in the UNSC to renew the mandate of a joint
Draft UN-OPCW team to determine responsibility for chemical weapons attacks in Syria.
Decision at
x UK led a draft decision to expand powers of OPCW at the Fourth Special Session of the CoP of
recent OPCW
OPCW. This would allow OPCW to apportion blame for illegal chemical attacks.
meet
x More than two-thirds of the member of OPCW voted to authorise OPCW to identify the
perpetrators and sponsors of a chemical weapons attack.
x OPCW’s technical secretariat would now begin work on identifying the perpetrators behind the
use of chemical weapons in Syria.

FOCUS | July 2018 | RAU’S IAS 17


@civilminds
International Relations

x India, along with Russia, China, Iran and others voted against the Draft Decision.
x India had asserted that it will grant the Director General as an individual, unprecedented and
unchecked powers in-particular ‘identification’ of perpetrators of an attack.
Indian x This would lead to the investigator also assuming the role of the judge. This may lead to partisan
Opposition use of the institution of the Director General for vested interests.
x India asserted that the creation of such a mechanism with OPCW technical secretariat is not in
keeping with the provisions of the Convention and would require amendment of CWC
convention.

Russian Russia has claimed that the mandate of the CWC had been changed with powers of the UNSC being
Opposition handed to the OPCW as a mean to bypass Russia’s veto in UNSC.

Note: The 2013 Nobel Peace Prize was awarded to the OPCW, for its extensive work to eliminate chemical weapons.

UN HUMAN RIGHTS
Recently, the United States of America (USA) withdrew from the
COUNCIL UNHRC. Let us understand about UNHRC and the reason for US
withdrawal.
# International Organisation
THINGS TO UNDERSTAND

The UN Human Rights Council is the principle intergovernmental body within the United Nations
responsible for:
Objective
9 Strengthening the promotion and protection of human rights around the globe;
9 Addressing and taking action on human rights violations around the globe.

x The Human Rights Council started working in 2006 wherein all countries at the UN take part in the
Working work of the Human Rights Council.
Committee x However, only some countries make decisions, with 47 countries elected by all UN members
making decisions in the Human Rights Council.

The functions of UNHRC includes:


9 Responding to human rights emergencies;
9 Making recommendations on how to better implement human rights;
9 Discuss all thematic human rights issues and country-specific situations that require its
Functions attention;
9 Establish international commissions of inquiry and fact-finding missions investigating and
responding to human rights violations;
9 Reviews the human rights records of all UN members under Universal Periodic Review, which
includes feedback from the respective country also;

The reasons cited by United States for withdrawing from UNHRC are:
9 UNHRC had admitted nations with questionable human rights records, however did not specifically
US
mention the countries;
Withdrawal
9 Specific targeting of Israel by UNHCR in its Universal Periodic Review;
9 Overlooking several countries such as Venezuela on their human rights violations.

FOCUS | July 2018 | RAU’S IAS 18


@civilminds
International Relations

x The UN set up the council in 2006 to replace the UN Commission on Human Rights which also
faced widespread criticism for letting countries with poor human rights records become members
such as China, Vietnam, Russia, Saudi Arabia, Venezuela, Philippines, among others.

Past x UNHRC is being alleged by US to be controlled by bloc voting by several Middle East and African
Criticism nations, supported by China, Russia and Cuba, which protect each other from criticism.
by US x The Bush Administration decided to boycott UNHRC when it was created in 2006 for many of the
same reasons as cited by the Trump administration.
x The Obama administration had joined UNHRC as a mean to reform the organisation but continued
to have and raise the same concerns.

WORLD GEOGRAPHY IN CONTEXT


#POLITICAL GEOGRAPHY
Æ Sentosa Island, Singapore
x Sentosa island served as the meeting place between US
President Donald Trump and North Korean Leader Kim
Jung Un in June, 2018.
x Both countries signed a joint statement which reaffirmed
the denuclearisation of Korean penisnsula as envisioned in
the Panmunjom Declaration, apart from other Confidence
Building Measures.
x However, the implications of the summit on India is yet to
be seen or ascertained.
(Refer to June edition of FOCUS to further understand about
the North Korean Crisis & the Panmunjom Declaration)

Æ Hodeidah, Yemen
x Saudi Arabia led Arab coalition forces seized the Hodeidah
airport from the Houthis in June, 2018.
x The Arab coalition have been engaged in a conflict for
political control of Yemen with the Iran allied Houthi’s that
has led to a humanitarian crisis in Yemen.
x Houthi’s are an ethnic group in Yemen that
predominately belong to the Shia sect of Islam.
x The Arab coalition intend to take control of the city as
Hodeidah port as it serves as a major route of Iranian
imports to Houthi fighters in Yemen conflict.
x UAE as part of the Arab coalition has extended for
diplomatic support from India to the Hodeidah operation
on international platforms.

FOCUS | July 2018 | RAU’S IAS 19


@civilminds

CONSTITUTION, POLITY AND GOVERNANCE


# GS Paper (Prelims) & GS Paper II (Main)

HIGHER EDUCATION COMMISSION OF INDIA


#Bill #Regulatory Body #Governance
The government of India with an intention to reform higher education has approved a Draft Bill to establish
a Higher Education Commission of India (HECI) by repealing University Grant Commission (UGC).
Higher Education Commission of India (Repeal of University Grants Commission Act) Bill 2018 has been
prepared by the Ministry of Human Resource & Development (MHRD) and it seeks to repeal the University
Grants Commission Act, 1956.
As of now, UGC has regulatory powers which include quality assessment and funding of higher education.
The new Bill aims to separate academics and funding of higher education. The proposed Bill has been placed
in the public domain for comments and suggestions. Let us understand the salient features of the proposed
Bill and also its impact on the quality of higher education.

PRINCIPLES UNDERLYING THE PROPOSED BILL

The proposed Higher Education Commission of India (Repeal of University Grants Commission Act) Bill 2018 is based
on the following principles:

Less Government
It aims to reduce interference in the management issue of educational institutions.
more Governance

Separation of The grant functions would be carried out by the Ministry of Human Resource & Development
Grant Functions and the Higher Education Commission will focus only on academic matters.

The Bill aims to encourage merit based decision making in a transparent manner on
End of Inspection
matters regarding standards and quality in higher education. This is proposed to be done
Raj
through transparent public disclosures.

HECI is tasked with the mandate to improve academic standards with specific focus on learning
Focus on
outcomes, evaluation of academic performance by institutions, mentoring of institutions, training
Academic Quality
of teachers, promoting use of educational technology etc.

Power of HECI shall have powers to enforce compliance of rules regarding academic quality and can
Enforcement sanction punishment of imprisonment for a term which may extend up to 3 years.

SALIENT FEATURES OF THE BILL

1. To promote the quality of academic instruction and maintenance of academic standards.


2. Take measures to promote the autonomy of higher educational institutions for the free
Functions of
pursuit of knowledge, innovation, incubation and entrepreneurship and to facilitate access,
the
inclusion and opportunities to all.
Commission
3. Provide comprehensive and holistic growth of higher education and research in a competitive
global environment.

Maintenance The Commission shall ensure maintenance of academic standards in the Higher Education system in
of Academic the following ways:

FOCUS | July 2018 | RAU’S IAS 20


@civilminds
Constitution, Polity and Governance

Standards x Specify learning outcomes for courses of study in higher education.


x Lay down standards of teaching / assessment / research or other aspect having bearing on
outcomes of learning in higher educational institutions including:
9 curriculum development;
9 training of teachers; and
9 skill development
x Evaluation of yearly academic performance of higher educational institutions by monitoring
performance criteria.
x Promote research in Higher Educational Institutions (HIEs) and coordinate with Government for
provision of adequate funding for research.
x By putting in place a robust accreditation system for evaluation of academic outcomes by
various HIEs.

Closure of HECI can order closure of such institutions which fail to adhere to minimum standards as
Institutions prescribed or such institutions which fail to get accreditation within the specified period.

Award of De- HECI may specify norms and standards for grant of authorization to a university or a higher
gree or educational institution empowered to award any degree or diploma to commence its first academic
Diploma operations.

x HECI may specify norms and standards for grant of authorization, to a university or a higher
Grant of educational institution empowered to award any degree or diploma, to commence its first
Authorization academic operations. No institution can award degree or diploma without authorization from
and HIEC.
Revocation x HECI can even revoke authorization granted to an institution if it is satisfied after conducting an
inquiry.

HIEC may lay down standards for grant of autonomy for institutions and provide flexibility and
Autonomy
freedom to institutions granted autonomy to develop their own curriculum.

x There shall be an Advisory Council chaired by the Union Minister for Human Resources
Development.
Advisory x The Council shall also have Chairperson/Vice-Chairperson, members of the Commission, and
Council Chairperson/Vice-Chairpersons of all State Councils for Higher Education as members.
x The Advisory Council shall identify issues of coordination in implementing the laid down
standards in higher education and find ways for their resolution.

x The Commission shall consist of a Chairperson, Vice Chairperson and twelve other Members to
be appointed by the Central Government. The Secretary of the Commission will act as the
Members of
Member-Secretary.
HECI
x A person appointed as Chairperson, Vice Chairperson and other Members shall hold office for a
term of five years from the date on which he enters upon his office.

CHALLENGES

Centre’s decision to shift grant-giving powers of higher education to the Ministry of Human
Resource Development (MHRD) or a separate body. This can hamper neutrality in grant allocation.
Grant
x So far UGC had been doing the granting allocation and ensured separation of funding decisions
allocation
from political consideration.
x Maintaining transparency in fund allocation to universities without any bias to any institution

FOCUS | July 2018 | RAU’S IAS 21


@civilminds
Constitution, Polity and Governance

despite political dispensation will now depend on the proposed Advisory Council to the HECI
chaired by Union Minister of MHRD.
x Advisory Council will also have members from State Councils for advice to ensure federal
representation but the centre will have the final say on any issue.
x Thus, the granting allocation of funds rests totally at the discretion of the Central
Government.

In India, different streams of higher education are regulated by an independent regulatory body of
their respective stream. For eg: All India Council for Technical Education (AICTE) for engineering; Bar
Council of India for Legal education and practice, National Medical Commission for Medical and
dental education and practice etc.
HECI in the x Yashpal Committee had earlier recommended that different streams of education should be
world of brought under a single commission.
multiple
x In this regard, Ministry of HRD had earlier proposed constituting Higher Education
educational
Empowerment Regulation Agency (HEERA) to replace UGC and AICTE, in an attempt at
regulators
eliminating overlaps in jurisdiction.
x However, the government does not seem much interested in the idea of an overarching
regulatory body for all streams of higher education presently.
Thus, the future role of HECI to regulate over these regulatory bodies is yet to be seen.

Quantification x Promoting the quality of academic standards is the most daunting challenge as it requires
of academic quantification of merits.
standards x Strict parameters need to set for measuring performance of higher education and followed
without any impartiality.
x However, such set benchmark incites manipulation from various quarters including the political
class along with bureaucracy for certain benefits to those in the higher education industry.

The quality of higher education in India requires major improvement to make it internationally competitive.
Do you think that the entry of foreign educational institutions would help improve the quality of technical and
higher education in the country. Discuss.
(UPSC GS PAPER II – 2015)

The National Medical Commission Bill, 2017 was introduced in the


Parliament to regulate the medical education system and its
practice in the country by replacing the Indian Medical Council
MEDICAL Act of 1956 which establishes the Medical Council of India.

COMMISSION However, the Union Cabinet has approved certain official


amendments to the National Medical Commission (NMC) Bill,
AMENDMENT BILL 2017. The Amendment to the NMC Bill comes in the backdrop of

#Amendment #Act its consideration in the Lok Sabha and report of the Department
Related Parliamentary Standing Committee (DRPSC).
#Governance The Government also took into considerations the general
feedback and views of medical students and practitioners
regarding certain provisions of the Bill.

FOCUS | July 2018 | RAU’S IAS 22


@civilminds
Constitution, Polity and Governance

KEY PROVISIONS OF THE NMC BILL

1. Provide for a medical education system that ensures availability of adequate and high quality
medical professionals;
2. Encourage medical professionals to adopt latest medical research in their work and to
Aims & contribute to research;
Objectives 3. Provide periodic assessment of medical institutions and facilitate maintenance of a medical
register for India;
4. Enforces high ethical standards in all aspects of medical services;
5. Provide effective grievance redressal mechanism.

The Bill establishes a National Medical Commission to lay down policies for
9 maintaining a high quality and high standards in medical education;
9 regulating medical institutions, medical researches and medical professionals;
National 9 assess the requirements in healthcare, including human resources for health and healthcare
Medical infrastructure.
Commission
NMC consists of: a Chairperson; Twelve ex officio Members; Eleven part-time Members, and an ex-
(NMC) officio Member Secretary
Among part-time members, three members to be appointed on rotational basis from amongst the
nominees of the States and Union territories in the Medical Advisory Council for a term of two years.
(Increased to 6 members)

National x The Council shall be the primary platform through which the States and Union territories may
Medical put forth their views and concerns before the Commission and help in shaping the overall
Advisory agenda, policy and action relating to medical education and training.
Council x The Council shall advise the Commission on measures to determine and maintain, and to
coordinate maintenance of, the minimum standards in all matters relating to medical education,
training and research.

x Admissions in all seats of MBBS/BDS courses will be done through National Eligibility Cum Entrance
Test (NEET) for undergraduates in Medical and Dental Colleges approved by Medical Council of
India/Dental Council of India under the Union Ministry of Health and Family Welfare.
x However, admission shall not be conducted through NEET for the institutions established through
NEET
an Act of Parliament i.e. AIIMS and JIPMER Puducherry.
x After the Supreme Court Order, it has become mandatory for all medical colleges to take
admission to MBBS and BDS courses through The National Eligibility cum Entrance Test (NEET).
x So effectively, NEET has become a pan India exam to get admission into medical colleges.

National x There shall be a National Licentiate Examination to obtain a licence to practice medical profession
Licentiate after graduation.
Examination x This examination will be the basis for admission to post-graduate medical courses.

Bridge x The Bill allowed for a Bridge Courses at undergraduate and post-graduate level to develop bridges
Courses across the various systems of medicine and promote medical pluralism.
x This effectively allowed AYUSH practitioners to prescribe modern medicine.
x However, there was a protest regarding “Bridge Course” because of which the 2017 Bill was
referred to a Parliamentary Standing Committee in January.

FOCUS | July 2018 | RAU’S IAS 23


@civilminds
Constitution, Polity and Governance

AMENDMENTS

x Final MBBS Examination to be held as a Common Exam across the country and would serve as
an exit test called the National Exit Test (NEXT).
x Thus, the students would not have to appear in a separate exam after MBBS to get license to
National Exit
practice.
Test
x NEXT would also serve as the screening test for doctors with foreign medical qualifications in
order to practice in India.

x The provision dealing with bridge courses for AYUSH practitioners to practice modern medicine
Removal of to a limited extent has also been removed.
Bridge Course x It has been left to the State Governments to take necessary measures for addressing and
promoting primary health care in rural areas.

Regulation of The maximum limit of 40% seats for which fee would be regulated in private medical institutions
fees and deemed universities has been increased to 50% seats.

Increased Number of nominees from States and UTs in National Medical Commission (NMC) increased from 3
Representation to 6.

x Another major concern which was the wide range of monetary penalty, ranging from one half to
ten times the annual fee recovered from a batch, to be imposed in a graded manner on a
Change in medical college non-compliant with the norms.
penalty norms x The clause has been replaced with a provision which provides different options for warning,
reasonable monetary penalty, reducing intake, stoppage of admission leading up to withdrawal
of recognition.

Stringent
The punishment for any unauthorized practice of medicine has been made more severe by
punishment for
including a provision for imprisonment of up to one year along with a fine extending up to Rs. 5
unqualified
lakhs.
practitioners

DAM SAFETY BILL, The Union Cabinet has approved the proposal for introduction of
Dam Safety Bill, 2018 in the Parliament. The objective of this Bill is
2018 to help develop uniform, countrywide procedures to ensure
safety of dams in India. It will help all the States and Union
#Bill #Governance
Territories to adopt uniform dam safety procedures which shall
#Infrastructure ensure safety of dams and safeguard benefits from such dams.

NEED FOR A DAM SAFETY BILL IN INDIA

x Safety of dams is important, not only to safeguard huge infrastructure investments but also to
safeguard human life and properties of people who live downstream of such dams.
x Most of the dams built in India is getting old and need proper repairs and maintenance due
Infrastructural
to structural deficiencies and shortcomings in operation and monitoring facilities.
Issues
x Most of the States are unable to provide sufficient budgets for maintenance and repair of the
dam. Many States also lack the institutional and technical capacities to address dam safety
issues.

x Dam Safety Institutional Framework in India are: The Central Water Commission, through the

FOCUS | July 2018 | RAU’S IAS 24


@civilminds
Constitution, Polity and Governance

Administrative National Committee on Dam Safety (NCDS), Central Dam Safety Organization (CDSO) and State
Issues Dam Safety Organizations (SDSO) advises on the maintenance of dams in India.
x However, such organisations do not have any statutory power to enforce regulation and their
role is advisory in nature.

Some of the dams in India share common territory between two states. Maintenance of dam and
flow of water becomes a controversial concern between the two states.
For eg: Mullaperiyar dam in Kerala, which has become a controversial issue between Kerala and
Federal Issue
Tamil Nadu due to height of the dam and sharing of water with Tamil Nadu. When Tamil Nadu
wanted to increase storage of the dam, Kerala opposed it citing safety threats. Eventually, a
Supreme Court team inspected the dam and confirmed in November 2014 that the dam was safe.

Thus, the central government has come up with a new legislation to regulate issues of dam safety and water
management. The new Bill addresses the following issues:
x Lack of uniform legal infrastructure for dam safety in India;
x Adoption of uniform safety standards for dam across the country;
x Security and safety concerns of citizens living around the vicinity of the dam in case of any emergency.

SALIENT FEATURES OF THE BILL

1. It provides for proper surveillance, inspection, operation and maintenance of all specified
dams in the country to ensure their safe functioning.
2. It provides for constitution of a National Committee on Dam Safety which shall evolve dam
Aims and
safety policies and recommend necessary regulations as may be required for the purpose.
Objectives of the
3. It provides for establishment of National Dam Safety Authority as a regulatory body which
Bill
shall discharge functions to implement the policy, guidelines and standards for dam safety in
the country.
4. It provides for constitution of a State Committee on Dam Safety by State Governments.

The Dam Safety Bill has bestowed NDSA the following functions:

9 It shall maintain liaison with the State Dam Safety Organisations and the owners of dams
for standardisation of dam safety related data and practices.
9 It shall provide the technical and managerial assistance to the States and State Dam Safety
Organisations.
9 It shall maintain a National level Data-Base of all dams in the country and the records of
major dam failures.
9 It shall examine the cause of any major dam failure and penalise states for not ensuring
National Dam adequate safety standards.
Safety Authority 9 It shall accord recognition or accreditations to the organisations that can be entrusted with
(NDSA) the works of investigation, design or construction of new dams.

9 It shall publish and update the standard guidelines and check-lists for the routine
inspection and detailed investigations of dams and appurtenances.
9 It will also look into unresolved issues between the State Dam Safety Organisation of two
states, or between the State Dam Safety Organisation of a State and the owner of a dam in
that State, for proper solution.
9 Further, in certain cases, such as dams of one State falling under the territory of another
State, the National Authority shall also perform the role of State Dam Safety Organization
and will help in eliminating potential causes for inter-state conflicts.

FOCUS | July 2018 | RAU’S IAS 25


@civilminds
Constitution, Polity and Governance

The Bill also provides for constitution of a State Committee on Dam Safety by the State
State Committees governments.
on Dam Safety 9 It will ensure proper surveillance, inspection, operation and maintenance of all specified
dams in that State and ensure their safe functioning.
9 It further provides that every State shall establish a "State Dam Safety Organisation", which
will be manned by officers from the field dam safety preferably from the areas of dam-
designs, hydro-mechanical engineering, hydrology, geo-technical investigation,
instrumentation and dam-rehabilitation.

DNA PROFILE BILL, The Union Cabinet has approved DNA Technology (Use and
Application) Regulation Bill, 2018 drafted by Department of
2018 #Bill #Governance Biotechnology.

ABOUT THE BILL

The primary intended purpose for enactment of the “DNA Based Technology (Use and Regulation)
Aims and
Bill 2018” is for expanding the application of DNA-based forensic technologies to support and
Objectives
strengthen the justice delivery system of the country.

x The Bill allows law enforcement agencies to collect DNA samples, create DNA profiles and create
special Databanks for forensic-criminal investigations.
Benefits of x The use of DNA profiling through databanks will help in enhancing and regulating the use of
DNA Sampling forensic science in criminal investigations.
x Use of forensic science will further enhance credibility of criminal investigations by providing
accurate evidences which will also help in increasing the conviction rate.

It will enable identification of persons by cross-matching between persons: -

9 who have been reported missing;


9 who are unidentified dead bodies found in various parts of the country;
Identification
9 victims of mass disasters or accident;
9 offenders, suspects or under-trials;
9 unknown deceased persons.

x The Bill provides for mandatory accreditation and regulation of DNA laboratories.
x Bill seeks to ensure that with the proposed expanded use of this technology in the country,
Mandatory
there is also the assurance that the DNA test results are reliable and the data remain protected
Accreditation
from misuse or abuse in terms of the privacy rights of our citizens.

x The Bill constitutes a DNA Profiling Board that would be the ultimate authority to regulate the
use of DNA based technologies.
DNA Profiling
x The Board will function to authorize the creation of State level DNA databanks, approve the
Board
methods of DNA collection and inquire about DNA- technologies.

FOCUS | July 2018 | RAU’S IAS 26


@civilminds
Constitution, Polity and Governance

ASPIRATIONAL DISTRICT The ‘Transformation of Aspirational Districts’


Programme (ADP) aims to expeditiously improve the
PROGRAMME – NITI socio-economic status of 117 districts from across 28
states as per report of NITI Aayog. Districts will be
AAYOG ranked on their "incremental progress" through Delta
#Governance #Competitive Federalism Ranking.

The Aspirational Districts programme, through real-time monitoring and proactive course corrections, reinforces the
mechanisms of cooperative & competitive federalism between the Centre and the States, down to the Districts.
For this, NITI Aayog has entered into partnership with Tata Trusts, and Bill & Melinda Gates Foundations to assist the
districts in enumerating improvement in key performance indicators a through household survey.

THINGS TO KNOW

The three core principles of the programme are –

1. Convergence of Central and State Schemes;


Core 2. Collaboration among citizens and functionaries of Central & State Governments including district
Principles teams); and
of the 3. Competition among districts.
programme
This initiative focuses on the strengths of each district and prioritizes the attainable outcomes for
immediate improvement. The programme is driven primarily by States.

The programme focuses on 5 main themes which have a direct impact on quality of life and economic
productivity of citizens –

Themes Weight Allotted

Health & Nutrition 30%

Focus Education 30%


Themes Agriculture & Water Resources 20%

Financial Inclusion & Skill 10%


Development

Basic Infrastructure 10%

KEY PARAMETERS ON EACH THEME

It focuses on antenatal care, postnatal care, gender parity, health of new-born, growth of children,
Health
contagious diseases, and health infrastructure.

It focuses on learning outcomes, infrastructure of schools and institutional indicators.

9 Learning Outcomes - transition rate from primary to upper primary, and subsequently to
Education secondary schooling, average scores in mathematics and languages etc.
9 Infrastructure - toilet access for girls, drinking water, electricity supply etc.
9 Institutional Indicators - RTE mandated pupil-teacher ratio, timely delivery of textbooks

Agriculture and Agriculture is the backbone of India, with more than 50% of our workforce engaged in cultivation
Water and allied activities and hence becomes one of the key factors of governance.
Resources The focus is on

FOCUS | July 2018 | RAU’S IAS 27


@civilminds
Constitution, Polity and Governance

9 Output - yield, price realisation etc.,


9 Inputs - quality seed distribution, soil health cards, and
9 Institutional Support - crop insurance, electronic markets, artificial insemination, animal
vaccination etc.

Progress in financial inclusion shall be measured through

9 Implementation of important government schemes – like Atal Pension Yojana, Pradhan Mantri
Financial
Jeevan Jyoti Bima Yojana etc.
Inclusion and
9 Reach of institutional banking - number of accounts opened under Jan Dhan Yojana
Skill
Development 9 Ease of institutional financing for small businesses - disbursement of Mudra loans
Skill Development shall be tracked through progress in skilling of youth, employment, and the
skilling of vulnerable/marginalized youth under Pradhan Mantri Kaushal Vikas Yojana (PMKVY).

x Housing for all with water, electricity, and road connectivity is the priority of the Government.
x Basic Infrastructure shall also be measured through availability of individual household
Basic
latrines, drinking water, electricity, and road connectivity.
Infrastructure
x Districts are also tracked for the number of internet connected Gram Panchayats, and
panchayats with Common Service Centres.

KEY BENEFITS

x Districts can utilise the data available on various indicators to improve their performance in each of the identified
core themes.
x The purpose of this ranking is to spur a sense of competition among dynamic teams in districts including
identification of challenging areas pertaining for each district.
x This will further help the Central Government in identifying key challenges for various districts in terms of
implementation of central government schemes. This will further help the government in identifying weak districts
for better targeting of government schemes for its citizens.
x This will not only help district authorities in improving quality of life of citizens but will help in promoting the
concept of competitive governance among districts on quantifiable parameters.

MEETING OF NITI The Prime Minister (PM) chaired the fourth meeting of the
Governing Council of NITI Aayog which was attended by Union
AAYOG Ministers, Chief Ministers of states, Lieutenant Governor of
Andaman & Nicobar Islands and other special invitees. The Prime
#Governance #Institution
Minister highlighted the importance of co-operative federalism in
#Cooperative Federalism order to move towards a path of development and growth.

THINGS TO KNOW

The Governing Council of NITI Aayog comprises of


About
9 Prime Minister of India;
Governing
9 Chief Ministers of all the States and Union Territories with Legislatures;
Council
9 Lt. Governor of Andaman and Nicobar Islands;

FOCUS | July 2018 | RAU’S IAS 28


@civilminds
Constitution, Polity and Governance

9 Four Union Ministers as ex-officio members and three Union Ministers as Special Invitees.

x The Council is the premier body tasked with evolving a shared vision of national development
priorities, sectors and strategies with the active involvement of States in shaping the development
narrative.

x It reviews the work done during the previous years and deliberates upon future developmental
priorities.

x The Governing Council, which embodies the objectives of cooperative federalism, presents a
platform to discuss inter-sectoral, inter-departmental and federal issues in order to accelerate the
implementation of the national development agenda in the spirit of Ek Bharat Shrestha Bharat.

x The NITI Aayog has been mandated to foster cooperative federalism through structured support

NITI Aayog initiatives and mechanisms with the States on a continuous basis.

and Co- x This help in understanding the fact that strong States make a strong nation thereby nurturing the
operative concept of federalism as envisaged by our Constitution.
Federalism x It seeks to help in implementing long-term policy frameworks and programme initiatives by
monitoring their progress and efficiency.

EMPHASIS ON CO-OPERATIVE FEDERALISM BY PM IN THE MEETING

x Stressed the need for effective center-state cooperation to advance development outcomes and achieve double-
digit growth for India.

x Advised the States to pay special attention to expanding their exports and attracting export oriented investment
while organizing their investments summits and events.

x The PM called upon the Chief Ministers of Madhya Pradesh, Bihar, Sikkim, Gujarat, Uttar Pradesh, West Bengal and
Andhra Pradesh to work together to make recommendations on a coordinated policy approach on the subject of
‘Agriculture and MNREGA’.

x Emphasized that States may set up their own Committees to look into ‘Ease of Living’ for the common citizen.

x Urged States to work towards the goal of a TB-Free India by 2025, five years prior to the global target of 2030.

x The Prime Minister also took into consideration State’s concerns over the ‘Terms of Reference’ of the Fifteenth
Finance Commission and encouraged States to come up with new ideas or a new formula, which incentivizes
states to improve their performance.

x The PM called for exploring the possibility of holding simultaneous election and the advantages of a common
voter list for each State.

The concept of cooperative federalism has been increasingly emphasised in recent years. Highlight the
drawbacks in the existing structure and extent to which cooperative federalism would answer the
shortcomings.
(UPSC MAINS QUESTION GS PAPER II – 2015)

FOCUS | July 2018 | RAU’S IAS 29


@civilminds
Constitution, Polity and Governance

There has been an increase in demand for reservation by different castes


NEED TO and sub-castes across the country. This sustained demand for reservation
is primarily to get government jobs. However, the government is not able
INCLUDE to accommodate all the demands regarding reservations made by different

CASTE IN communities.
It is important for the government to know about different castes, sub-
UPCOMING castes including their sizeable population spread across the country so as

2021 CENSUS to plan accordingly for their targeted welfare. Thus, the government must
plan towards including caste in the upcoming census of 2021.
#Social Indicator In this analysis, let us understand the historical background of caste
#Governance census in India, reasons to include caste in the upcoming census and the
possible benefits both for the government and the people.

BACKGROUND

x The process of enumerating the count of Indians by British started in the early 18th century of various cities.
x The British conducted the first census in 1871-72. The idea behind census of 1871 was accumulation of social
statistics for administrative purpose to efficiently match state resources with social needs.
x The 1871 census documented Nationality, Language, Religions, Caste divisions, female infanticide etc.
x The British conducted census periodically after every 10 years till 1931. Census of 1931 was the last census which
documented India’s caste as this practiced was stopped after independence.
x According to experts, the process of recording caste generated a conception of community as a homogeneous and
classifiable community. This influenced the processes of political representation along caste lines.
x Consequently, post-Independence Censuses have shied away from including questions about caste.

ABOUT SECC - 2011

The Ministry of Rural Development commenced the Socio Economic and Caste Census (SECC) 2011,
in June 2011 through a comprehensive door to door enumeration across the country.
The SECC, 2011 had the following objectives:

9 To enable households to be ranked based on their Socio- Economic status;


What is SECC –
2011? 9 Identify below poverty line persons for government benefit schemes in urban and rural areas;
9 To make available authentic information to enable caste-wise population enumeration of the
country;
9 To make available authentic information regarding the socio economic condition, and education
status of various castes and sections of the population.

In 2017, government constituted an Expert Group under the Chairmanship of former Finance
Secretary, Shri Sumit Bose for

9 Studying the objective criteria for allocation of resources to States; and


Expert Group
on SECC, 2011 9 Identification and prioritization of beneficiaries under various programme using Socio Economic
and Caste Census (SECC) data.
Expert Group has observed that regular updation and verification of SECC data is essential for
improved targeted delivery of essential services.

Recomme- x The SECC has the potential to move from being only a census-like socio-economic database to
ndations of becoming the core of a functioning Social Registry Information System (SRIS).

FOCUS | July 2018 | RAU’S IAS 30


@civilminds
Constitution, Polity and Governance

Expert Group x SRIS would result in several advantages in implementation of social sector schemes.
x It has the potential to streamline programme administration, reduce duplication of benefit and
fraud, saving on time and costs for both programme applicants and services providers,
monitoring the living standards of beneficiaries over time, better targeting of vulnerable and
marginalized sections of the society and enable expansion of the coverage of the programmes.
x Finally, the use of SECC data would lead to better budgetary planning and allocation of resources
for various programmes.

NEED TO INCLUDE CASTE IN 2021 CENSUS

x Land fragmentation and decades of agricultural stagnation have turned many upper caste landowners into
marginal farmers. Whereas rising rural wage has benefitted some of the backward classes including dalits.
x Caste based mobilization along political lines have increased over a period of years. This has also led to increase in
demand for reservation in jobs.
x The real challenge lies in recognising the need for a caste based census to find a possible solution for the growing
demands of reservation for different castes and communities across India.
x Thus, a caste census in 2021 will reveal the changed circumstances of rural and urban landscape of India including
their economic condition.
x Such a detailed data will further help in rationalizing reservation policy as per need of a specific caste or community.

The question that is raised is whether the senior positions in


government can continue to be the exclusive monopoly of the civil
LATERAL ENTRY services?

IN INDIAN At present all the civil services are cadre-based i.e. a person joins a
service and moves up the ladder. The natural corollary of this is that
BUREAUCRACY there are very few lateral entries and the civil services enjoy a virtual
#Civil Service Reforms monopoly over the all the positions in the government. With rapid
expansion of knowledge, increasing complexities in certain fields,
#Governance rapid expansion of private sector, a large amount of expertise has
developed outside government.

Taking this into consideration, the Department of Personnel and Training has published invitation of applications for
senior positions in Government of India at the level of Joint Secretary.
Even the NITI Aayog in its Three Year Action Agenda under civil services reform has suggested increasing specialisation
and inducting talent through lateral entry in bureaucracy. In this regard NITI Aayog had even suggested to amend rules
of recruitment to accommodate lateral entry.
Let us go through the decision of the government regarding lateral entry and its impact on governance.

x Government of India through a notification has invited talented and motivated Indian nationals
Decision of willing to contribute towards nation building to join the Government at the level of Joint
the Secretary.
Government x Persons from Private Sector Organisations and public institutions including state government,
Union Territory Administration, Public Sector Enterprises, can apply through lateral entry.

Purpose of x The purpose of lateral entry is aimed at bringing fresh ideas and new approaches to governance

FOCUS | July 2018 | RAU’S IAS 31


@civilminds
Constitution, Polity and Governance

the decision and also to augment manpower.


x Candidates who have expertise in specific areas of Revenue, Financial Services, Economic Affairs,
Agriculture, Road Transport & Highways, Shipping, Environment & Forests, New & Renewable
Energy, Civil Aviation and Commerce are invited to join at the level of Joint Secretary.

x A Joint Secretary is the head of the wing under the charge of Secretary of the Ministry/ Department
About the and they report to Additional Secretary/ Secretary.
post of Joint x Joint Secretaries are at a crucial level of senior management in the Government of India.
Secretary x They lead policy making as well as implementation of various programmes and schemes of the
Department concerned.

x Age: Minimum of 40 years as on July 1, 2018.


x Qualifications: Graduate from a recognized University/Institute. Higher qualifications would be an
added advantage.
x Officers of any State/UT Government who are already working at equivalent level or are eligible for
Criteria of appointment to equivalent level in their cadre, with relevant experience.
selection x Individuals working at comparable levels in Public Sector Undertakings (PSUs), Autonomous
Bodies, Statutory Organisations, Universities, Recognized Research Institutes with a minimum of 15
years’ experience.
x Individuals working at comparable levels in Private Sector Companies, Consultancy Organisations,
International/Multinational Organisations with a minimum of 15 years’ experience.

x Candidates from private sector organisation shall be appointed on contract basis whereas
members from public institutions shall be appointed on deputation including short term
Form of contract.
appointment
x The period of contract shall be for 3 years from the date of commencement of job. It can be
extended upto 5 years depending upon the performance.

IMPACT OF LATERAL ENTRY

x It will help in bringing expertise from the private sector in diverse fields who is not accustomed to the practices and
procedural labyrinth of bureaucracy.
x Such persons can help in infusing new thought process and help in imbibing best practices of both worlds. This will
help in providing new approaches to governance.
x This will encourage bureaucrats to gain expertise in specific fields in their early stage of careers. Specialisation in
diverse fields will help serving bureaucrats in having longer tenure as officer in their respective field of expertise.

1. Initially Civil Services in India were designed to achieve the goals of neutrality and effectiveness, which seems
to be lacking in the present context. Do you agree with the view that drastic reforms are required in Civil
Services. Comment
UPSC MAINS – GS PAPER – II (2017)
2. Has the Cadre based Civil Services Organization been the cause of slow in India? Critically examine.
UPSC MAINS – GS PAPER – II (2017)

FOCUS | July 2018 | RAU’S IAS 32


@civilminds
Constitution, Polity and Governance

DELHI AGREEMENT OF The Central Government has decided not to file any
“counter-affidavit” on Article 35A which has been
1952 AND ARTICLE 35A challenged in Supreme Court through public interest
#Constitution #Agreement litigation as the matter is still pending in the Court.

THINGS TO UNDERSTAND

x This was signed between Jawaharlal Nehru and the then Prime Minister of Jammu and
Kashmir, Sheikh Abdullah.
x As per the Delhi Agreement of 1952, it was agreed between state of Jammu & Kashmir and
Union of India that persons domiciled in Jammu and Kashmir shall be regarded as citizens of
Delhi Agreement India.
x But the State Legislature was empowered to make laws for conferring special rights and
privileges on the State's subjects.
x Thus, keeping the Delhi Agreement in mind, the 1954 Presidential Order provided for Article
35A to the Indian Constitution.

Article 35A grants the status of permanent residency to the residents of Jammu and Kashmir
(J&K). Article 35A has been added to the Indian Constitution through The Constitution (Application
to Jammu and Kashmir) Order, 1954 in exercise of the powers conferred by clause (1) of Article
370 of the Indian Constitution, and not through an amendment under Article 368 of the
Constitution.
The subjects on whom Article 35A applies are:

x A law defining the classes of persons who are, or shall be permanent residents of the State of
Article 35A Jammu and Kashmir
x A law conferring on such permanent residents any special rights and privileges or imposing
upon other persons any restrictions with respect to:
9 Employment under the State Government;
9 Acquisition of immovable property in the State;
9 Settlement in the State;
9 Right to scholarships and such other forms of aid as the State Government may provide.

The concept of Permanent Resident is implicit in Article 35A which is also prescribed in section 6
of Constitution of Jammu and Kashmir.
It says that every person who is, or is deemed to be, a citizen of India under the provisions of the
Constitution of India, shall be a permanent resident of the State -
Permanent 9 If on the fourteenth day of May, 1954 they were state subject,
Resident 9 He/she has acquired immovable property and has been resident of the state for not less than
10 years from May, 1954
9 Any person who, before the fourteenth day of May, 1954, was a State Subject – who migrated
to Pakistan after the first day of March, 1947 – has returned to the state under a permit for
resettlement – or has returned permanently to the state as acknowledged by state authorities.

x Article 370 of the Indian Constitution provides for temporary provisions with respect to State
Article 370 of Jammu and Kashmir.
x As per Article 370 (1)(b), Parliament can make laws only on those matters of Union List and

FOCUS | July 2018 | RAU’S IAS 33


@civilminds
Constitution, Polity and Governance

Concurrent List which corresponds to the subjects mentioned in the State’s Instrument of
Accession.
x Parliament can make law on such other subjects in the Union or Concurrent List as the
President may by order specify with the concurrence of State Government of J&K.
x Thus, Article 370 empowers the President to define the constitutional relationship of the
State in terms of the provisions of Indian Constitution, in concurrence with State of Jammu and
Kashmir and as per terms of “Instruments of Accession”.
x Maharaja Hari Singh, signed “The Instrument of Accession” which was accepted by the
Governor-General of India, Lord Mountbatten on 27th October 1947.

GROUNDS FOR CHALLENGE IN SUPREME COURT

1. President cannot amend the Constitution through a Presidential Order as amendment of the Constitution can be
carried only through Article 368 which confers the Power on the Parliament.
2. Position of Article 370 was always a temporary provision in our Constitution and must be removed so as to bring
State of Jammu and Kashmir at par with other states of the Indian Union.
3. Government of Jammu and Kashmir has discriminated against non-residents as they are not allowed to buy
property in the state, get government jobs or vote in local elections.
4. Permanent Resident Certificate as guaranteed by Constitution of Jammu and Kashmir violates Article 14 of Indian
Constitution.

View Point

THE PROBLEMS WITH THE HECI DRAFT BILL


# Bill # Reforms
(courtesy: The Hindu)

The draft Higher Education Commission of India (Repeal of University Grants Commission Act) Bill, 2018 (HECI),
aims to replace a historical statutory body, the UGC; push for more government control; and stifle critical thinking on
campuses. As the education system is the most potent instrument for shaping a country’s future, and given India’s
massive youth population, reframing the education system in a manner that will reflect the government’s agenda is
clearly imperative for it.

No time to discuss
That the government is in a hurry to pass this Bill in Parliament is reflected in the fact that Human Resource
Development Minister Prakash Javadekar gave merely 10 days to stakeholders to submit their feedback on the Bill. This
is worrying and definitely not the way a massive reform such as this should be deliberated.
Mr. Javadekar argues that the draft Bill is in accordance with the government’s commitment to reform the regulatory
mechanism to provide “more autonomy” to higher education institutes. He believes that the HECI will cater to the
changing priorities of higher education. The UGC, it is argued, is preoccupied with disbursing funds and is unable to
concentrate on mentoring higher education institutes, focus on research, and implement other quality measures
required in the education sector. So, the HECI will focus solely on academic matters while grants will be issued by the
Ministry.
The argument is perplexing as what is expected of the higher education system as envisaged by Mr. Javadekar can very
well be done by the UGC. To do so, the UGC needs to be restructured in a manner that will ensure that its autonomy is

FOCUS | July 2018 | RAU’S IAS 34


@civilminds
Constitution, Polity and Governance

strengthened without any scope for patronage politics and political interference. However, no such restructuring has
been attempted, taking into account the UGC’s founding goals, achievements, shortcomings and the reasons for such
shortcomings.
Instead, as the HECI draft Bill is already up on the Ministry’s website, it is important to dwell upon at least six of the
concerns that are being expressed by the media and by academics.

Six concerns
One, Mr. Javadekar tweeted that the transformation of the regulatory set-up is based on the principles of minimum
government and maximum governance, separation of grant functions, the end of inspection raj, powers to enforce
‘Saaf Niyat, Sahi Vikaas’, and focus on academic quality. This is clearly a case of doublespeak. The nature of the
structure of the commission and its advisory council shows that they are bound to have more “government” in
decision-making processes rather than academics.
Two, sweeping powers render the HECI more authoritative than the collective strength of campus authorities. The
powers and functions of the HECI trivialise the concept of autonomy, not the least because “non-compliance (of
directions of the HECI) could result in fines or jail sentence.” This means that the authority of the HRD Ministry will be
strengthened. Also, under the new terms of engagement, universities will have to take the concurrence of the HECI
before offering a course. This restricts the freedom of a university’s Board of Studies. The draft Bill states: If any
University grants affiliation in respect of any course of study to any institution in contravention of the provisions of the
regulation/rule/recommendation issued by the Commission...the Commission... may impose a penalty on such
University and/ or on such Institution which may include fine, or withdrawal of power to grant degrees/diplomas or
direction to cease operations.” If there is a threat of academic functions being usurped through this legislation, it calls
for reflection.
Three, with its mandate of improving academic standards with a specific focus on learning outcomes, evaluation of
academic performance by institutions, and training of teachers, the HECI is likely to overregulate and micromanage
universities.
Four, the proposal to empower the Centre to remove the HECI’s chairperson and vice-chairperson for reasons
including “moral turpitude” will again curtail the regulator’s autonomy, which in turn will impact the autonomy of
universities.
Five, instead of allowing institutions to evolve over time based on their specific needs, focussing on homogeneous,
one-size-fits-all administrative models will go against the ethos of academic freedom, diversity, and knowledge
production, and will help attempts to corporatise the education sector.
Six, the move to replace the UGC with the HECI points to the Centre’s aim to restrict the role of the States in matters
relating to education. Strangely, this legitimate apprehension has been articulated by only one politician, Kerala Chief
Minister Pinarayi Vijayan, even though others have been voicing concerns that the Terms of Reference for the 15th
Finance Commission will weaken federalism.

Access to education
Mr. Javadekar’s sudden decision to opt for the HECI is attributed to Prime Minister Narendra Modi’s lament in October
2017 that no Indian university figures among the world’s top 500. However, Mr. Modi’s worries are misplaced. This is
because one, the bigger concern for India is that despite being a country with a huge young population, higher
education remains a privilege; many do not yet have access to it, mainly because it is not affordable. Also, those who
do have access attend universities to further their life chances; aiming to get their university in the world’s top 500 list
is not their priority. Two, education is a continuum from lower to higher. The quality of higher education is determined
by the quality of lower education, which is extremely poor, and that should be our focus. Three, despite the Modi
government’s slogan, ‘Sabka Saath Sabka Vikas’, the fact is that the number of Scheduled Castes, Scheduled Tribes, and
Muslims who have access to even basic education, let alone higher education, remains abysmal. The Prime Minister
should concentrate his energies on improving this dismal scenario rather than lamenting about India not figuring in

FOCUS | July 2018 | RAU’S IAS 35


@civilminds
Constitution, Polity and Governance

the world’s top universities list. Even the poorest child in India should have access to the best education that will
benefit and improve his or her future. Education must serve as ladder for those in the lower rungs of society. In India
there is no such ladder, and many children continue to lead a poor quality life with no access to education. Seen from
this perspective, the fact that there is no reference to expand the higher education sector such that it will reach the
marginalised and the poor is what is actually a “blot”. Including the excluded should be India’s goal, and reservation
and affirmative action are the way forward.

FOCUS | July 2018 | RAU’S IAS 36


@civilminds

ECONOMIC DEVELOPMENT
# GS Paper (Prelims) & GS Paper III (Main)

MINING SECTOR: PROSPECTS AND CHALLENGES


#Sector in Focus
(Lead coverage)

RANK OF INDIA IN WORLD PRODUCTION OF PRINCIPAL MINERALS AND METALS, 2015


(factsheet)

Commodity Contribution (%age) India’s rank (Quantum of


production)

Mineral Fuels

Coal and lignite 8.13 3rd

Petroleum (crude) 0.88 23rd

Metallic Minerals

Bauxite 9.57 4th

Chromite 8.20 4th

Iron ore 4.69 4th

Manganese ore 4.04 6th

Industrial minerals

Kyanite, andalusite & sillimanite 18.72 3rd

Maganesite 0.59 11th

Apatite & rock phosphate 0.56 16th

Metals

Aluminium 4.08 5th

Copper (refined) 3.45 7th

Steel (Crude/liquid) 5.56 3rd

Lead (refined) 1.36 14th

Zinc (slab) 5.76 5th

Recently, the Union Government has allowed 100 per cent Foreign Direct Investment (FDI) in the mining
sector and exploration of metal and non-metal ores under the automatic route, which is expected to propel
growth of this sector.

FOCUS | July 2018 | RAU’S IAS 37


@civilminds
Economic Development

Opportunities Challenges

x The government’s ambition to ensure 24*7 x Presence of low grade minerals in the country
‘power to all’ has pushed demand for coal. x Land availability is a major hurdle in setting up almost
x Demand for Iron and Steel is set to grow, given any industry in India, it is specially challenging for mining
the strong growth expectations for the residential industry.
and commercial building industry. x Environmental challenges- as most of the minerals are lie
x India has world class reserves in Iron, in forest
Manganese, Bauxite, Chromite, Coal and few x Socio-economic challenges- most of the minerals bed
other minerals. have tribal population. Any effort of displacement causes
x India has world renowned mining engineering huge socio-economic risks
institutions in IIT Kharagpur, IIT (ISM) Dhanbad, x Mining industry has faced regulatory uncertainty in
IIT (BHU) etc. terms of statutory clearances stretching project
x India has the right demographics with 380 million construction timelines to the point of making these
persons between the ages of 18-35. unviable.
Demographics have a special significance in x Labour laws are drawn and enforced with the premise
mining industry as mining activities are labour that engagement of contract labour is only to cheat on
intensive and physically demanding. wages.
x Together with manufacturing, mining has the x Skill gap among workforce- this is extremely important
highest potential in the distribution of wealth by for mining industry as a mine requires different skills at
generating employment across geographies and different stages of operation.
education levels.
x Small size of mining properties discourages economy of
scale and investment in the sector.

Î ADDITIONAL INFORMATION: GOVERNMENT’S EFFORT IN MINERAL SECTOR

Mines & Minerals (Development The act is the law governing the mineral sector (other than Petroleum and
and Regulation) Act, (MMDR Act) Natural Gas) of the country. It recognises a substantial role of state
1957 governments.

Mines and Minerals (Development


The act seeks to incentivise private sector investment in exploration and
and Regulation) Amendment Act,
mines.
2015

x Provision of DMF was inserted by bringing an amendment in the MMDR


(Amendment) Act, 2015
District Mineral Foundation
x The objective of DMF is to work for the interest and benefit of persons
in and areas affected by mining related operations.

x The Pradhan Mantri Khanij Kshetra Kalyan Yojana (PMKKKY) will be


implemented by the DMFs of the respective districts using the funds
accruing to the DMF.
x At least 60% of PMKKKY funds will be utilized for: (i) drinking water
Pradhan Mantri Khanij Kshetra supply; (ii) environment preservation and pollution control measures; (iii)
Kalyan Yojana (PMKKKY) health care; (iv) education; (v) welfare of women and children; (vi) welfare
of aged and disabled people; (vii) skill development; and (viii) sanitation.
x Up to 40% of the PMKKKY funds will be utilised for: (i) physical
infrastructure; (ii) irrigation; (iii) energy and watershed development; and
(iv) any other measures for enhancing environmental quality in mining

FOCUS | July 2018 | RAU’S IAS 38


@civilminds
Economic Development

district.

x The Project was undertaken by the Indian Bureau of Mines, Ministry of


Mines and BISAG (Bhaskaracharya Institute for Space Applications and
Geo-informatics) of Ministry of Electronics and Information Technology
Mining Surveillance system (MEITY).
x It aims to develop a system for detection of incidence of illegal mining by
use of space technology and surveillance of area up to 500m beyond the
lease boundary to check incidences of illegal mining.

National Mineral Exploration NMEP aims towards accelerating mineral exploration and attracting
Policy, 2016 investment in the country

NATIONAL MINERAL POLICY, 2018 (DRAFT)

1. Role of states in mining will be facilitation and regulation of exploration activities by investors and
entrepreneurs.
2. There shall be transparency while reserving areas for state agencies unless security considerations or specific
public interests are involved.
3. Grant of clearances for commencement of mining operations shall be streamlined with simpler and time-bound
procedures.
4. To explore the country's entire geological potential, a detailed exploration should be carried out systematically
in a time-bound manner.
5. The role of the government agencies would be survey and exploration, while the private sector would be
encouraged to take up exploration activities.
6. Exploration would be incentivised to attract private investments through an adequate financial package or
through right of first refusal at the time of auction.
7. Clearances will be streamlined with simpler, accountable, and time-bound procedures.
8. The offshore areas, country's exclusive economic zone should be explored and exploited.

WAY FORWARD

1. There is need to establish right balance between interests of industry and land owners in transfer of land
ownership from non-industrial to industrial sectors.
2. Rationalization of environmental approval process, this is being seen as the right prescription for a major
challenge faced by the mining industry.
3. The current discourse of increase in salary levels, social security and quality of life of contract workers and
those in unorganised sector will surely facilitate development of pool of employees with specialised skills.
4. Exploration of the deep-seated minerals needs to be focused.
5. Private sector needs to play active role in exploration of mining, especially in deep-seated mining.
6. There is large number of small players in the mining business. Now the time has come for the consolidation,
either through consortium of miners.
7. The government needs to promote effective utilization of low grade ore.
8. The policy makers need to address challenges in land ownership, rationalisation of labour laws,
consolidation of mining properties
9. Training and skill development for the population.
10. Need to apply Information Technology and IT enabled services in mining

FOCUS | July 2018 | RAU’S IAS 39


@civilminds
Economic Development

MARGINAL COST The marginal cost of funds based lending rate (MCLR) refers to the
minimum interest rate of a bank below which it cannot lend,
OF FUNDS BASED except in some cases allowed by the RBI. It is an internal
LENDING RATE benchmark or reference rate for the bank.
The base rate is the erstwhile loan pricing mechanism which was
(MCLR) replaced by MCLR — the Marginal Cost of funding based Lending
#Banking and Finance Rate — from April 1, 2016.

x The RBI has not yet mandated banks to allow the customer to shift to the current marginal cost of
Recent
funds based lending rate (MCLR) structure from the erstwhile ‘Base Rate’ regime.
Controversy
x This is leading to customers paying higher interest rates on home and auto loans.

x RBI indicates that if banks offer their customers a lower interest rate, public sector banks will incur
huge loss of Rs. 40,000 crores.
x Since State-run banks are already reeling under huge losses due to a rise in non-performing loans
Why RBI has in the previous financial year, asking them to shift their customers to MCLR could impact their
not allowed? interest income further.
x The one-year MCLR of banks, to which a bulk of the loan rates are linked, is lower than the base
rate. For example, SBI’s base rate is 8.7% while its one-year MCLR is 8.25% — a difference of 45
basis points.

THINGS TO UNDERSTAND

x To improve the transmission of policy rates into the lending rates of banks.
x To bring transparency in the methodology followed by banks for determining interest rates on
Reasons for advances.
Introducing x To ensure availability of bank credit at interest rates which are fair to borrowers as well as
MCLR banks.
x To enable banks to become more competitive and enhance their long- run value and
contribution to economic growth.

Under the MCLR, the calculation of interest rates will be based upon the following:
9 Marginal cost of funds- Which in turn depends upon
1. Bank rate interest (charged by banks from consumers)
Calculation
2. Repo Rate (charged by RBI from banks)
of Interest
Rate under 9 Operating expense of bank- It is the daily cash requirement for running the bank

MCLR 9 Cost of carrying in the CRR- It means that banks have to take into account the amount of CRR
blocked with them which they can't use
9 Tenor premium- It means the higher interest charged on long-term loans which are equal for all
types of loans.

DIFFERENCE BETWEEN BASE RATE AND MCLR

BASE RATE MCLR

Based on marginal/incremental cost of funds.


Based on average cost of funds
Marginal cost of funds depends upon

FOCUS | July 2018 | RAU’S IAS 40


@civilminds
Economic Development

1. Bank rate interest (charged by banks from consumers)


2. Repo Rate (charged by RBI from banks)
3. Rate of Return on the capital

Calculated by considering tenor premium. It means the higher


Calculated by considering minimum rate of
interest charged on long term loans which are equal for all types of
return/profit margin
loans.

Base is also governed by operating The MCLR is also determined by considering deposit rates and repo
expenses, and expenses needed to maintain rates, along with operating cost and cost of maintaining cash reserve
cash reserve ratio ratio

India has notified higher tariffs on several items imported from the
HIGHER TARIFFS U.S., including agricultural products such as apples, almonds,
chickpeas, lentils, and walnuts, and industrial inputs such as some
ON U.S. IMPORTS grades of iron and steel products amounting to a $240 million.

# Foreign Trade The was done in retaliation to the U.S. action on steel and
aluminium. This is the result of increasing U.S. protectionism.

BACKGROUND

x The issue began in March, when U.S. President Donald Trump signed an order imposing a 25% tariff on steel
imports and a 10% tariff on aluminium imports, causing concern among its trading partners and members of the
WTO.
x Later, U.S. framed new tariffs on $200-billion worth of Chinese imports. The action saw retaliation from China
imposing a tariff on $50- billion worth in American goods.
x Now even, European Union joined the trade war, imposing tariffs on $3.3 billion of American goods.
With this India comes close on the heels of escalating trade tensions, with the U.S. Along with India, China and
European Union are also engulfed in tariff war with U.S.

IMPACT

x Imposing tariffs basically means either you look for other sources, or you import at higher
On trade and
prices.
growth
x This will have a detrimental impact on world trade and undermine growth prematurely.

The U.S is a large economy, of course, it has more sustaining power to bear losses but in the long-
On consumers
run, costs at both the ends would go up, for Indian consumers as well as for U.S. consumers.

Such actions will affect global financial market and investors will shy away from investing in the
On Investors market. Impact of such actions have started emerging in the global financial market and Indian
financial market will not remain immune from this.

x The only gainers in a trade war will be special interest groups, such as the U.S. steel industry.
x The fact is that all sides engaged in a trade war eventually lose. The longer it goes on, the greater
A brief
the cost as growth slows down under the increasing burden of taxes.
summation
x Hence, it is in the interest of all the countries that they endorse a rule-based trading and try their
best to bring an end to the ongoing trade war before it gets out of hand.

FOCUS | July 2018 | RAU’S IAS 41


@civilminds
Economic Development

National Investment and Infrastructure Fund (NIIF) is a fund


INFRA FUNDS: AIIB created in 2015 by the Government of India for enhancing
TO INVEST infrastructure financing in the country. It is envisaged as a fund
of funds with the ability to make direct investments as required.
# Investment As a fund of fund, it may invest in other SEBI registered funds.

BROAD OBJECTIVE

x The objective of NIIF would be to maximize economic impact mainly through infrastructure development in
commercially viable projects, both greenfield and, brownfield, including stalled projects.
x It could also consider other nationally important projects, for example, in manufacturing, if commercially viable.

In this, Asian Infrastructure Investment Bank (AIIB) has shown interest and has approved $100 million investment,
which could be followed by a similar amount later.

x It is a multilateral development bank, headquartered in Beijing, China.


x It commenced operation in January 2016.
x It aims to improve social and economic outcomes in Asia and beyond.
About AIIB
x Presently, it has 86 countries as its members.
x India is the second largest shareholder in AIIB after China and is also the largest recipient of
funds from the multilateral agency.

x The government of India has approached AIIB seeking $ 475 million for the Mumbai Urban
Transport Project-III which had already been approved in principle.
Other
x As per the Ministry of Finance, nearly 25% of funds committed by AIIB have been committed for
investments
projects in India.
in India
x AIIB has approved $4.4 billion investments so far, including $1.2 billion in India, making the
country the largest beneficiary so far.

FINANCIAL The FSR reflects the overall assessment of the stability of India’s
financial system and its resilience to risks emanating from global
STABILITY REPORT and domestic factors. The Report also discusses issues relating
(FSR) to developments and regulation of the financial sector. The RBI
has recently released June Financial Stability Report (FSR).
# Financial Sector
KEY OBSERVATIONS IN THE REPORT

x The report has observed that the 7.7% GDP growth in the last quarter indicates the economy is well on the
recovery track on the back of a sharp pick-up in gross fixed capital formation, which shows essential net
investment in the economy.
x Some of the sectors such as steel has recovered and capacity utilisation in the sector has increased.
x The report has also observed that overall aggregate demand composition indicates a broad-based growth with
the revival of investment.
x At the same time, the report warns that the emerging scenario including fiscal and current account challenges
posing risks to the economic momentum.

FOCUS | July 2018 | RAU’S IAS 42


@civilminds
Economic Development

RISKS IN THE ECONOMY

Global x Tightening of liquidity conditions in the developed markets alongside expansionary US fiscal
front policy and a strong US dollar have started to adversely impact emerging market currencies,
bonds, and capital flows.
x Firming commodity prices, evolving geopolitical developments and rising protectionist sentiments
pose added risks.

Domestic On the domestic front, economic growth is firming up. However, conditions that helped fiscal
front consolidation, moderation in inflation and a benign current account deficit over the last few years, are
changing, thereby warranting caution.

Domestic x The stress in the banking sector continues as gross non-performing advances (GNPA) ratio rises
financial further.
market x Profitability of scheduled commercial banks (SCBs) declined, partly reflecting increased provisioning.
x While this has added pressure on SCBs’ regulatory capital ratios, the provisioning coverage ratio has
increased.

ASSET RECONSTRUCTION The Central government has revived the idea of


setting up an asset reconstruction or asset
OR ASSET MANAGEMENT management company to deal with stressed assets.
This is a sort of ‘bad bank’ mooted for the first time by
COMPANY Chief Economic Adviser Arvind Subramanian in
# Finance January 2017.

THINGS TO UNDERSTAND

Stressed Assets- Stressed assets are loans on which the borrower has defaulted or it has been restructured.

Stressed assets = NPAs + Restructured loans + Written off assets.

An NPA is a loan or advance for which the borrower has failed to repay the principle or interest for a
NPA
period of 90 days.

x Restructured asset or loan are that assets which got an extended repayment period, reduced
Restructured interest rate, converting a part of the loan into equity, providing additional financing, or some
asset combination of these measures.

x It is part of bad loan or NPA which is modified as a new loan.

x A written off is the removal of an asset or liability from the accounting records and financial
Written off statements of a company.

x Companies tend to write off assets because the assets are no longer available or valid.

x An Asset Reconstruction Company is a specialized financial institution that buys the NPAs or bad
assets from banks and financial institutions so that the latter can clean up their balance sheets.
About ARC
Or in other words, ARCs are in the business of buying bad loans from banks.

x ARCs clean up the balance sheets of banks when the latter sells these to the ARCs. This helps

FOCUS | July 2018 | RAU’S IAS 43


@civilminds
Economic Development

banks to concentrate in normal banking activities.

x Banks rather than going after the defaulters by wasting their time and effort, can sell the bad
assets to the ARCs at a mutually agreed value.

x The Securitization and Reconstruction of Financial Assets and Enforcement of Security Interest
(SARFAESI) Act, 2002; enacted in December 2002 provides the legal basis for the setting up ARCs
SARFAESI Act, in India.
2002 x The SARFAESI Act helps reconstruction of bad assets without the intervention of courts. Since
then, large number of ARCs were formed and were registered with the RBI which has got the
power to regulate the ARCs.

Î RELATED INFORMATION: BAD BANK

x The Economic Survey 2016-17 had envisaged a Public-Sector Asset Rehabilitation Agency
What is ‘bad (PARA).
bank’? x The agency would take on public sector banks’ chronic bad loans and focus on their
resolution.

x The bad bank will require significant capital to purchase stressed loan accounts from public
sector banks.
How will it be x On account of the fact that the size of gross NPAs of public sector banks is currently over ₹10
capitalised? lakh crore and chances of private participation are low unless investors are allowed a major
say in the governance of the new entity, it is the government which will have to be the single
largest contributor.

x Shifting stressed loan accounts to a bad bank would free public-sector bank balance sheets

How would it from their deleterious impact and improve their financial position.

help in solving x This will improve the quality of bank’s assets and improve their capital position and reduces the
the NPA chances of insolvency.
problem? x It would also allow government-owned banks to focus on their core operations of providing
credit for fresh investments and economic activity.

It will not address the more serious corporate governance issues plaguing public sector banks
Drawback
that led to the NPA problem in the first place.

The government is set to unveil a Rs. 500-crore ‘credit

CREDIT ENHANCEMENT enhancement fund’ to facilitate infrastructure


investments by insurance and pension funds. The fund
FUND # Investment was first announced in the Budget for the fiscal year 2016-
17.

Objective x This will help in


9 upgrading credit ratings of bonds issued by infrastructure companies, and
9 facilitate investment from investors like pension and insurance funds.

FOCUS | July 2018 | RAU’S IAS 44


@civilminds
Economic Development

Operating The initial corpus of the fund will be sponsored by IIFCL (India Infrastructure Finance Company) and
Agency it will operate as a Non-Banking Finance Company - NBFC.

Î RELATED INFORMATION : NBFC

x Non-banking financial companies (NBFCs) are financial institutions that offer various banking services but do not
have a banking license.
x NBFCs can offer banking services such as loans and credit facilities, retirement planning, money
markets, underwriting and merger activities.
x Generally, these institutions are not allowed to take deposits from the public. This keeps them outside the scope of
traditional oversight required under banking regulations.

The centre has decided to set up the country’s biggest data centre in
NATIONAL DATA Bhopal. This will be set up by the National Informatics Centre (NIC),
CENTRE under the Ministry of Electronics and Information Technology
(MeitY). This will be the fifth National Data Centre after the ones at
# Security #Data Privacy Bhubaneswar, Delhi, Hyderabad and Pune.

ABOUT DATA CENTRE

x It is a facility that centralizes the IT operations and equipment of an organization.

What does x It stores, manages and disseminates the data.


the data x These National Data Centres host government websites, services and applications.
centre do? x Computing resources, vital telecommunications, including servers, storage systems, devices,
access networks, software, and applications are essential components of a data center services.

1. With the push of the digital ecosystem in the country, data centres are required to manage the
data.

Why the 2. To protect the data privacy in the country.


Data centre 3. E-commerce and online marketplaces are powered by data centers, and they are a vital part of
is required? today’s growing online economy.
4. Internet of Things (IoT) is a rapidly growing concept and data center is playing a significant role in
supporting it. It depends 100% on data centers for connectivity, availability and data storage.

Under the Liberalised Remittance Scheme (LRS), RBI is concerned over


funds sent abroad under the ‘maintenance of close relative’ category. As a
CHECK ON
result, RBI has narrowed the definition of relatives to check the flow of
OUTWARD funds.
This has brought about by defining ‘relatives’ under the Companies Act,
REMITTANCES 2013 instead of the Companies Act of 1956. Hence, funds under the
# Remittance ‘maintenance of close relative’ category can be sent only to immediate
relatives such as parents, spouses, children and their spouses.

ABOUT LIBERALISED REMITTANCE SCHEME (LRS)

x LRS is a facility provided by the RBI for all resident individuals including minors to freely remit up to a certain
amount in terms of US Dollar for current and capital account purposes or a combination of both.

FOCUS | July 2018 | RAU’S IAS 45


@civilminds
Economic Development

x Hence under the LRS, individuals are allowed to spend money in foreign countries for specific purposes like
education, tourism, asset purchase etc. The money can be remitted for the purchase of shares and property as
well.
x The remittance limit is set for a financial year. The Reserve Bank has raised the upper limit for foreign exchange
remittances under the LRS to $ 250,000 per person per year.
x The LRS was launched in 2004 and regulations for the scheme are provided under the Foreign Exchange
Management Act (FEMA), 1999.

THINGS TO UNDERSTAND

Why RBI x The central government doubt that this facility [maintenance of relatives] under the Liberalised
altered the Remittances Scheme is used for commercial purposes which are not its objective.
definition? x The move aims to prevent misuse of the facility.

x Outward remittances under ‘maintenance of close relatives’ shot up to almost $3 billion in 2017-
Explosive 18 from a mere $174 million in 2013-14. In fact, funds sent under this category have more than
growth in doubled since 2015-16.
Outward
x Overall outward remittances under LRS went up to $11 billion from $1 billion in the same period.
Remittances
x So, this required monitoring.

x RBI has introduced a system for daily reporting of individual transactions under the LRS by banks.
Benefit x This enables banks to view remittances already made by an individual during the fiscal, thus
improving monitoring and ensuring compliance.

THE SLIDE OF In 2018 itself, the Rupee has weakened by 6.2% which makes it among
INDIAN RUPEE the worst performing Asian Currencies. Resultantly, it can impact our
country’s macroeconomic stability.
# Financial management
REASONS FOR THE SLIDE OF RUPEE

High CAD occurred primary because of high merchandise trade gap which have occurred due
Current account to rising oil price leading to high import bill.
deficit (CAD) The current account deficit is a measurement of a country’s trade where the value of the goods
and services it imports exceeds the value of the goods and services it exports.

x It has been affected largely due to US sanctions against Iran and dwindling global supply
leading to increasing the price of oil in the international market.
Supply of oil x Increasing the price of oil means India will have to pay more in the international market.
x Since payment is done in the dollar in international market demand for dollar increases
and this leads to depreciation of rupee against dollar.

x Higher trade tariff against China is leading a rise in inflation in the U.S. due to rising cost
of Chinese product or higher prices of the local product. This happens especially in
Monetary
consumerist items.
tightening in U.S.
x However, increasing inflation forces fed to increase its interest rate. This leads investors
across the world to shift their investment from other developing countries to the US

FOCUS | July 2018 | RAU’S IAS 46


@civilminds
Economic Development

and India is not immune to this.

x Investors have shifted from high risk associated bond in developing countries to low-risk
associated bonds in the US market.

Bond market and x This has happened because investors are worried about inflation, fiscal balance and external
Exodus of Foreign funding.
Portfolio Investors Note: When inflation increases in the economy it erodes the benefit margin of investors.
(FPIs) Whereas increasing fiscal deficit and external funding increase the risk factor in the
economy due to the dependency of government on borrowing for funding their own project.
All these factors thwart investors investing in government bonds.

POSSIBLE WAYS OF DEALING WITH THE SLIDE

Raising of x This will help in preventing an exodus of Foreign Portfolio Investor (FPI) capital.
interest rate x However, this might squeeze credit for companies as well as lead to some cuts in capital spending
by RBI by the government — dragging down economic growth.

Using forex x Another option is that RBI intervening in the exchange market and selling the dollar in the
reserve market to stabilise rupee.
x RBI has taken this initiative recently which has caused a decline in forex reserve after reaching a
record high of $ 426 billion.

Bridging the x Large current account deficit is due to high import has caused more demand for dollar leading
export- to depreciation of rupee. This could be bridged by encouraging exports so that supply of dollar
import gap increases in the economy and hence rupee get strengthen.
x The Cabinet Committee on Economic Affairs has recently approved a capital infusion of Rs.
2,000 crores into the Export Credit Guarantee Corporation (ECGC) to be infused over the three
financial years 2017-20.
x The infusion would enhance insurance coverage to MSME exports and strengthen India’s exports
to emerging and challenging markets like Africa, CIS and Latin American countries.

FCNR If need occurs government can also raise funds through Foreign Currency Non-Repatriable (FCNR)
deposits deposits, sovereign bonds to increase reserves.
This is a Fixed Deposit Foreign Currency account and not a savings account. Non-Resident Indians
can invest in such accounts in any of the major currencies like US Dollar, UK Pound, Canadian Dollar,
Deutsche Mark, Japanese Yen and Euro.

Other routes Apart from that, it is also advised, in the long run, India should diversify its energy basket in order to
reduce dependency on oil.

A BRIEF SUMMATION

Dealing with rupee requires a response from both- monetary and fiscal policy. RBI’s response on three fronts is
watched in the market in such situation. These are- Inflation management, currency management in exchange
market and liquidity management in the bond market to keep government borrowing costs under control. Whereas
fiscal policy measures such as incentives to exporters and diversifying energy basket are key to control the
depreciation of Rupee.

FOCUS | July 2018 | RAU’S IAS 47


@civilminds
Economic Development

Since its midnight launch on July 1, 2017, India’s Goods and Services Tax regime
has evolved significantly. During its launch, the government had given
ONE YEAR arguments in favour of GST as it would lead to ease of doing business; make
markets efficient; yield higher tax collections; and lead to lower prices. With
OF GST higher tax collection, the government would be able to deliver better services.
Thus, the GST was presented as a win-win situation for everyone.
# Tax reforms
It has faced multiple challenges but the administrative will of both the centre
and the state helped the issue to get addressed in a real time.

POSITIVE OUTCOMES

x Despite initial challenges, the new tax has taken firm root and is altering the economic landscape positively.
x India’s tax net is widened due to new entry of over 4.5 million entities.
x Widening tax net will also help in increasing direct tax collections.
x GST has caused an increase in revenue. The government is eyeing a little over ₹90,000 crores a month to make up
for the revenues earned under the earlier regime and to compensate States for any losses due to the GST.
x The large part of the informal economy is becoming part of the formal economy.

CHALLENGES FACED IN IMPLEMENTATION

No ‘ease of Businesses have not yet experienced ‘ease of doing business’.


doing business’ GST rates were fixed rather late leading to difficulty for them to fix prices.

Unsatisfactory
The IT functioning of the Goods and Service Tax Network (GSTN) has been unsatisfactory due to
IT
problems or inordinate delays in access because of the volume of traffic.
Infrastructure

The complexity of the system still prevalent as businesses are supposed to file various return
Complexity
forms under GST.

Lack of Even though it was computerised, accounting was difficult. So, even though 17 taxes were
simplification replaced by one tax made up of many parts, simplification did not follow.

Problems faced
They encountered the problem of input tax credit. Further, they faced difficulty in inter-state sales
by small
in case they were at the border of the state.
businesses

The government made rapid changes which added confusion at the implementation level.
Ineffective
Such changes were made in the e-way bill, tax rates, inclusion and exclusion of various businesses
governance
under the composition scheme.

Instead, GST has contributed to inflation because services are now taxed higher, — the rate has
Price rise risen to 18% from 15%. Even though essential goods are exempt under the GST, as basic goods
and services prices rise, all prices increase.

Contrary to other countries, India does not have a full GST which is applicable from raw material to
Complex tax the final good/service.
structure The government has introduced multiple tax rates and exemptions and this only adds to the
complexity.

WAY FORWARD

x The government should further rationalise the GST tax structure.

FOCUS | July 2018 | RAU’S IAS 48


@civilminds
Economic Development

x Increase in revenue should be utilised to address the fiscal concerns of the centre and the states.
x The government should not tinker GST rates too often.
x In its second year, the GST Council must pursue a time-bound approach to execute plans already announced to
ease taxpayers’ woes, such as an e-wallet for exporters and a simpler return form.
x Besides, there must be a roadmap to bring excluded products — petroleum, real estate, electricity, alcohol — into
the GST net.
Finally, this reform still has miles to go, and the government must avoid the temptation to take populist steps ahead of
general elections.

The Indian economy has shown a strong V-shaped recovery


DISCUSSION AROUND driven largely by domestic growth. According to the Central
ECONOMIC RECOVERY Statistics Office (CSO), the Indian economy grew at 6.3%,
7.0% and 7.7% in the second, third and fourth quarters of
# Economy 2017-18, respectively.

MEANING OF V-SHAPED RECOVERY

x A V-Shaped economic recession describes the shape of the market's performance.


x It begins with a steep fall but then quickly find a bottom, turns back around and move immediately higher.
x It is used to measure employment, GDP and industrial output.
x Many economists use the V-Shape to forecast and analyze a country's health.
x A V-shaped recession is always mentioned as the best-case scenario.

FACTORS RESPONSIBLE FOR RECOVERY

Two segments which have supported growth, particularly in the fourth quarter of 2017-18, are
Demand government consumption and overall investment demand.
Side 9 Gross fixed capital formation- was as high as 14.4% in the fourth quarter of 2017-18
9 The real investment rate has also increased to 34.6%.

The government has focused upon productivity enhancing through various measures:
9 Demonetisation and the GST;
9 Inflation targeting-The New Monetary Policy Framework agreement has institutionalised a
consumer price index (CPI) inflation target of 4% on average.

Supply 9 Key policy initiatives (Make in India, Start-up India) also aim at improving productivity.
side 9 Market determination of mineral and spectrum prices.
9 Ujwal DISCOM Assurance Yojana scheme- This benefited power sector
9 The regulatory framework has been changed- in real estate and banking, the regulatory framework.
9 Additional fiscal space was created by better targeting of subsidies while the expansion for rail/road
projects was prioritised.

CHALLENGES IN THE ECONOMY

The Centre’s fiscal deficit-GDP ratio, after showing a steady improvement since 2014-15, slipped back
Prospects of
to a level of more than 3.5% of GDP in 2017-18, exceeding the fiscal responsibility and budget
fiscal
management (FRBM) target of 3% and the budgeted target of 3.2%. With the general election around
slippage
the corner, this situation may not improve.

FOCUS | July 2018 | RAU’S IAS 49


@civilminds
Economic Development

x The overall growth rate also appears to be better because of a better than expected performance
in the agriculture sector.
Agriculture x However, agricultural growth in real terms at 4.5% appears more of a statistical artefact than a
distress story of turnaround in agriculture.
x The statistics also confirm a collapse in agriculture prices which has affected the domestic demand
in rural areas.

x It may adversely affect most indicators of India’s macro balance including trade and current
Rising crude account deficits, inflation, exchange rate and fiscal deficit.
oil prices x Continued pressure on inflation may prompt the RBI to revise the repo rate upwards during the
current year. This, in turn, affects Investment, especially private investment.

x Despite recovery in the world economy, India’s export performance since 2014 is far less
Export impressive than it was in the five years following the crisis in 2007-08.
challenge x Further, the external environment is posing challenges due to
x increasing threats of China-US trade war affecting world trade including India’s trade.

Twin Balance Sheet Problem (TBS) deals with two balance sheet problems. One with Indian
companies and the other with Indian Banks. Thus, TBS is two two-fold problem for Indian economy
which deals with:
x Overleveraged companies – Debt accumulation on companies is very high and thus they are
Twin Balance unable to pay interest payments on loans.
Sheet Note: 40% of corporate debt is owed by companies who are not earning enough to pay back their
Problem interest payments. In technical terms, this means that they have an interest coverage ratio of less
than 1.
x Bad-loan-encumbered-banks – Non-Performing Assets (NPA) of the banks is 9% for the total
banking system of India. It is as high as 12.1% for Public Sector Banks. As companies fail to pay
back principal or interest, banks are also in trouble.

Î RELATED INFORMATION: GROSS FIXED CAPITAL FORMATION

x Gross fixed capital formation is essentially net investment. It refers to the net increase in
physical assets (investment minus disposals) within the financial year.
Definition x It does not account for the consumption (depreciation) of fixed capital, and also does not
include land purchases.
x It is a component of the Expenditure method of calculating GDP.

x It includes spending on land improvements (fences, ditches, drains, and so on); plant,
What it machinery, and equipment purchases; the construction of roads, railways, private residential
includes? dwellings, and commercial and industrial buildings.
x Disposal of fixed assets is taken away from the total.

What it x Land Purchases


excludes? x Effects of depreciation (referred to as consumption of capital)

FOCUS | July 2018 | RAU’S IAS 50


@civilminds
Economic Development

x In macro theory, a rise in investment should contribute towards higher aggregate demand
(AD) and also increase productive capacity. Increasing investment should lead to higher
Macroeconomic economic growth in the long-term (though it depends on how effective the investment is).
Impact x Opportunity cost of investment- The opportunity cost of gross fixed capital investment is
lower consumption – at least in the short-term. If more resources are spent on capital goods, it
leads to decline in consumption of consumer goods.

5G: PROMISES AND


A government panel has suggested that the telecom department
CHALLENGES IN should quickly release spectrum for R & D of 5G networks so that
INDIA India does not miss the next generation of mobile connectivity.

# Telecom #Technology
ABOUT 5G

x 5G networks would be the next generation of mobile connectivity, promising faster data speeds and
more reliable wireless connections, not just on smartphones but also other devices.
x 5G will not just offer average download speeds around 1GB per second but will also have the ability to
propel the growth of Internet of Things, since it can provide the necessary infrastructure to carry
What is
large amount of data.
5G
x 5G has low Network latency, which will help run autonomous cars, remote surgery and other
network?
functions.
Note: Network latency is the term used to indicate any kind of delay that happens in data communication
over a network. Network connections in which small delays occur are called low-latency networks
whereas network connections which suffers from long delays are called high-latency networks.

x In September 2017, the government formed a high-level forum to strategize on early deployment of
5G.
x The panel aims to develop a competitive product portfolio for 5G to target 50% of the Indian market
India’s and 10% of the global market over the next seven years.
Progress x The DoT has approved a 5G test-bed at IIT Madras.
x TRAI in August 2017, floated a consultation paper inviting views on the next auction of various
spectrum bands, ones suitable for 5G.
x Most telcos are doing 5G trials.

x A steering committee under the high-level forum has already submitted some recommendations
related to spectrum availability to the government and will submit its final report in August, 2018.
What x The panel has said 6,000 MHz of spectrum can be made available without delay for the next-
else? generation mobile service.
x The DoT plans to set up labs to showcase potential uses to industry across verticals such as banking,
railway and agriculture.

Leading 5G networks are seen to be launching across the world by 2020. China, South Korea and the US are
countries leading the pack. China may be the biggest 5G market by 2022.

FOCUS | July 2018 | RAU’S IAS 51


@civilminds
Economic Development

CHALLENGES

x Indian companies need to invest in R & D and focus on building design and manufacturing capacity for chips and
devices.
x The government also needs to allocate airwaves. With eroding revenue and little hope for a recovery in the near
term, telecom operators, who are in the midst of a tariff war, also face an unfavourable funding climate as they
expand 4G network rollouts and gear up for 5G.

Recently, the Ministry of New & Renewable Energy invited


Expressions of Interest (EoI) for the first 1 GW offshore wind
OFF-SHORE WIND project in India. In addition to this, the Ministry has also declared a
POWER medium and long-term target for off-shore wind power capacity
additions, which are 5 GW by 2022 and 30 GW by 2030. With this,
# Renewable energy the Offshore wind power would add a new element to the already
existing basket of renewable energy for the country.

ABOUT OFF-SHORE WIND POWER

Offshore wind power or offshore wind energy is the use of wind farms constructed in bodies
Offshore wind
of water, usually in the ocean on the continental shelf, to harvest wind energy to generate
power
electricity.

x Offshore wind speeds tend to be faster than on land. Small increases in wind speed yield
large increases in energy production.
x Offshore wind speeds tend to be steadier than on land. A steadier supply of wind means a
Advantages more reliable source of energy.
x Offshore wind farms have many of the same advantages as land-based wind farms – they
provide renewable energy; they do not consume water; they provide a domestic energy
source; they create jobs; and they do not emit environmental pollutants or greenhouse gases.

x Offshore wind farms can be expensive and difficult to build and maintain.
Disadvantages x Effects of offshore wind farms on marine animals and birds are not fully understood.
x It may affect tourism and property values.

Potential in India has an estimated 127 gigawatts (GW, or 1,000 MW) of offshore wind power potential,
India mostly off the coasts of Tamil Nadu, Gujarat, and Maharashtra.

x The Ministry of New & Renewable Energy had notified National Off-Shore Wind Policy in
October 2015 to realize the offshore wind power potential in the country.
Efforts made by
x The government has conducted two preliminary studies on off coast of Gujarat and Tamil Nadu
the government
x For precise wind quality measurements, one LiDAR has been installed near Gujarat coast
which is generating data about quality of the off-shore wind since November 2017.

Globally there has been the installation of about 17 to 18 GW of the off-shore wind power led by
Global level
countries such as the UK, Germany, Denmark, Netherlands & China.

Î RELATED INFORMATION: LIDAR

x LiDAR, which stands for Light Detection and Ranging, is a remote sensing method.
x It uses light in the form of a pulsed laser to measure ranges (variable distances) to the Earth. These light pulses
generate precise, three-dimensional information about the shape of the Earth and its surface characteristics.

FOCUS | July 2018 | RAU’S IAS 52


@civilminds
Economic Development

x A LiDAR instrument principally consists of a laser, a scanner, and a specialized GPS receiver. Airplanes and
helicopters are the most commonly used platforms for acquiring LiDAR data over broad areas.
x Two types of LiDAR are topographic and bathymetric. Topographic LiDAR typically uses a near-infrared laser to
map the land, while bathymetric LiDAR uses water-penetrating green light to also measure seafloor and riverbed
elevations.
x LiDAR systems allow scientists and mapping professionals to examine both natural and manmade
environments with accuracy, precision, and flexibility.

ACTION PLAN OF The Union Cabinet has approved the continuation


of the Three-Year Action Plan (2017-2020) of the
AGRICULTURAL EDUCATION scheme for Agricultural Education Division and

DIVISON # Agriculture #Scheme ICAR Institutes.

ABOUT THE SCHEME

x The scheme is aimed to generate quality human resources from the institutions of higher agricultural education.
x It encompasses several new initiatives including, steps to attract talented students, reducing academic
inbreeding and addressing faculty shortage.
x It will also take care of green initiatives, mitigating faculty shortage, promoting innovations, inspired teacher
network, reducing inbreeding, academia interface, technology-enabled learning, Post-doctoral Fellowships,
Agriculture Education Portal, Scientific Social responsibility etc.
x Support for strengthening and modernization of infrastructure related to student and faculty amenities and
capacity building of both faculty and students in the cutting-edge areas through Niche Area of Excellence
programme will improve teaching and encouraging the holistic development of the students.

Î RELATED INFORMATION

x It is an autonomous organisation under the Department of Agricultural Research and


Education (DARE), Ministry of Agriculture and Farmers Welfare, Government of India,
headquartered in New Delhi.
x Formerly known as Imperial Council of Agricultural Research, it was established in 1929 as a
Indian Council
registered society under the Societies Registration Act, 1860 in pursuance of the report of the
for Agriculture
Royal Commission on Agriculture.
Research
x It is the apex body for co-ordinating, guiding and managing research and education in
agriculture including horticulture, fisheries and animal sciences in the entire country.
x It has played a pioneering role in ushering Green Revolution and subsequent developments in
agriculture in India through its research and technology development.

National x It was established by the Indian Council of Agricultural Research in 1976 at Hyderabad.
Academy of x The mandate of the institution is to
Agricultural
9 Enhance individual and institutional competencies in managing innovation through capacity
Research
development, research and policy advocacy.
Management
9 Serve as a think tank for National Agricultural Research and Education System and facilitate
(NAARM)
strategic management of human capital.

FOCUS | July 2018 | RAU’S IAS 53


@civilminds
Economic Development

OFF-GRID AND The Cabinet Committee on Economic Affairs (CCEA)


DECENTRALISED SOLAR PV has given its approval for implementation of Phase-
Ill of Off-grid and Decentralised Solar PV (Photo
APPLICATIONS Voltaic) Application Programme to achieve
additional 118 MWp (Mega Watt peak) off-grid solar
PROGRAMME- PHASE III PV capacity by 2020.
# Energy
COMPONENT OF THE PROGRAM

Solar Street 3,00,000 numbers of solar street lights will be installed throughout the country with special emphasis
Lights on areas where there is no facility for street lighting systems through grid power, North Eastern
States and Left-Wing Extremism (LWE) affected districts.

Stand-alone x Solar power plants of individual size up to 25 kWp (kilo Watt peak) will be promoted in areas
Solar Power where grid power has not reached or is not reliable.
Plants x This component is mainly aimed at providing electricity to schools, hostels, panchayats, police
stations and other public service institutions.
x The aggregated capacity of solar power plants would be 100 MWp

Solar Study 25,00,000 numbers of solar study lamps will be provided in the North Eastern States and LWE
Lamps affected districts.

BENEFITS

x The off-grid solar systems will also open better livelihood opportunities for beneficiaries in rural and remote
areas thereby increasing self-employment in such areas.
x Besides increasing self-employment, the implementation of Phase-Ill is likely to generate employment opportunity
equivalent to 8.67 lakh man-days for skilled and unskilled workers.
x It will have an impact on the rural and remote areas of the country where grid power has either not reached or
is not reliable. During the Phase-Ill, the programme is likely to benefit 40 lakh rural households.
x The mission will help in increasing participation of women in education, social and livelihood activities.

In-Short
x The Commission for Agricultural Costs & Prices (CACP) is an attached office of the
Ministry of Agriculture and Farmers Welfare, Government of India.
COMMISSION x It came into existence in January 1965.

FOR x It is mandated to recommend minimum support prices (MSPs) to incentivize the


cultivators to adopt modern technology, and raise productivity and overall grain
AGRICULTURAL production in line with the emerging demand patterns in the country.
COSTS AND x MSPs for major agricultural products are fixed by the government, each year, after
taking into account the recommendations of the Commission.
PRICES (CACP)
x As of now, CACP recommends MSPs of 23 commodities, which comprise
9 7 cereals (paddy, wheat, maize, sorghum, pearl millet, barley and ragi),

FOCUS | July 2018 | RAU’S IAS 54


@civilminds
Economic Development

9 5 pulses (gram, tur, moong, urad, lentil),


9 7 oilseeds (groundnut, rapeseed-mustard, soyabean, seasmum, sunflower, safflower,
nigerseed), and
9 4 commercial crops (copra, sugarcane, cotton and raw jute)

x Presumptive taxation scheme (PTS) allows you to calculate your tax on estimated
income or profit.
x The scheme can be used by businesses having a total turnover of less than Rs
2crore and eligible professionals with gross receipts of less than Rs 50 lakh in the
financial year. Those who adopt PTS to file their returns are not required to maintain
books of accounts.

PRESUMPTIVE x Under PTS, the eligible businesses can estimate their net income at the rate of 6% of
the total turnover, if gross receipts are received through the digital mode of payments
TAXATION or at the rate of 8% in case of cash receipts. However, the assesse is allowed to
SCHEME willingly declare income at a higher rate than minimum of 6-8% of the total turnover.
x Professionals who are governed or regulated by an institute or body such as doctors,
lawyers, architects, interior designers and others can adopt PTS.
x For them, 50% of the total receipts during the fiscal will be considered as profit and get
taxed accordingly.
x A professional can voluntarily declare the income at a higher rate than mandatory 50%
of the total receipts.

x The money market primarily facilitates the lending and borrowing of funds
between banks and entities like Primary Dealers (PDs). Banks and PDs borrow and lend
overnight or for the short period to meet their short-term mismatches in fund
positions.
x Call Money is the borrowing or lending of funds for 1day.
x If money is borrowed or lend for the period between 2 days and 14 days, it is known as
TERM MONEY
Notice Money.
MARKET x Term Money refers to borrowing/lending of funds for the period between 15 days
and one year.
x Scheduled commercial banks (excluding RRBs), co-operative banks (other than Land
Development Banks) and Primary Dealers (PDs), are permitted to participate in
call/notice money market both as borrowers and lenders.
x Eligible participants are free to decide on interest rates in call/notice money market.

x The Union government has set up a Technical Committee to bring transparency in


the jobs calculation and presentation.
x The committee is headed by former chief statistician of India T.C.A. Anant.
x Labour bureau conducts two major surveys on employment scenario—Annual
PANEL ON Employment-Unemployment Survey and Quarterly Employment Survey.
JOBS DATA x The Annual Employment Unemployment Survey was later replaced by Periodic
Labour Force Survey (PLFS) being conducted by Ministry of Statistics and Programme
Implementation (MoSPI).
x The PLFS would not only have annual data on employment-unemployment on the
rural sector but will also have quarterly data on employment-unemployment on urban

FOCUS | July 2018 | RAU’S IAS 55


@civilminds
Economic Development

sector based on larger sample size. Therefore, the corresponding AEUS data set would
be comparable.
x Besides, the Employees Provident Fund Organization (EPFO) has started releasing
monthly payroll data.

x Shillong, the capital city of Meghalaya has been selected as 100th Smart City after
SHILLONG evaluating the proposal submitted by it.
GETS SELECTED x Previously, 20 cities were selected in January 2016, 13 cities in May 2016, 27 cities in

AS THE 100th September 2016, 30 cities in June 2017 and 9 cities in January 2018.
x With the selection of Shillong, the total proposed investment in the finally selected 100
SMART CITY cities under the Smart Cities Mission would be Rs.2,05,018 crores.

x The scheme is being implemented by the Ministry of Tourism, Ministry of Culture and
Archaeological Survey of India.
x The scheme aims to involve public sector companies, private sector companies and
corporate citizens/individuals to take up the responsibility for making our heritage and
tourism more sustainable through development, operation and maintenance of world-
class tourist infrastructure and amenities at ASI/ State heritage sites and other
ADOPT A important tourist sites in India.
HERITAGE x The Project aims to develop synergy among all partners to effectively promote
“responsible tourism”
SCHEME
x The project primarily focuses on providing basic amenities that includes
cleanliness, public conveniences, drinking water, ease of access for differently abled
and senior citizens, standardized signage, illumination and advanced amenities such as
surveillance system, night viewing facilities, tourism facilitation centre and an
enhanced tourism experience that will result in more tourist footfalls, both domestic
and foreign.

x The Ministry of Micro, Small and Medium Enterprises (MSMEs) has launched Solar
Charkha Mission.
x Mission has been launched in the 50 clusters of the country and each cluster will
SOLAR employ 400 to 2000 artisans.
CHARKHA x The mission will disburse subsidy of Rs 550 crore to the artisans.

MISSION The 10 centers which will become operational soon are located in Durg (Chhattisgarh),
Bhiwadi (Rajasthan), Rohtak (Haryana), Visakhapatnam (Andhra Pradesh), Bengaluru
(Karnataka), Sitarganj (Uttarakhand), Baddi (Himachal Pradesh), Bhopal (M.P.), Kanpur
(U.P.) and Puducherry.

FOCUS | July 2018 | RAU’S IAS 56


@civilminds

SOCIETY AND SOCIAL JUSTICE


# GS Paper I & GS Paper III (Main)

Stunting is the impaired growth and development that children


TO COMBAT experience from poor nutrition, repeated infection, and inadequate
STUNTING psychosocial stimulation. It is defined as the percentage of children,
aged 0 to 59 months, whose height for age is below the WHO Child
# Health #Empowerment Growth Standards.

THINGS TO UNDERSTAND

x Stunting reflects chronic undernutrition during the most critical periods of growth and development
in early life.
x Stunting starts from pre-conception when an adolescent girl and who later becomes mother is
More to
undernourished and anaemic; it worsens when infants’ diets are poor, and when sanitation and
know
hygiene are inadequate. It is irreversible by the age of two.
x Overall impact: Stunting is associated with an underdeveloped brain, with long-lasting harmful
consequences, including diminished mental ability and learning capacity, poor school performance in

FOCUS | July 2018 | RAU’S IAS 57


@civilminds
Society and Social Justice

childhood, reduced earnings and increased risks of nutrition-related chronic diseases, such as
diabetes, hypertension, and obesity in future.

x In India, 38 per cent of children younger than five years of age are stunted, a manifestation of chronic
undernutrition.
x While India’s economy has been growing at impressive rate, the country still has the highest number
of stunted children in the world, (46.8 million children) representing one-third of the global total of
Indian
stunted children under the age of five.
scenario
x As high as 70 per cent of adolescent girls in India are anaemic and half of adolescents are below the
normal body mass index, which has an impact on the health of their future pregnancies and children.
Thus, preventing stunting is critical to survival in the immediate term, and in the longer-term, to
ensure healthy, well-educated and productive adults.

RECENT FINDINGS

According to the International Food Policy Research Institute (IFPRI) study which analysed data
from the National Family Health Survey (NHFS)- IV, stated that parameters related to women,
New study
including education and age at marriage, account for 50% of the difference between districts with high
and low levels of stunting among children below the age of five.

FOCUS | July 2018 | RAU’S IAS 58


@civilminds
Society and Social Justice

x Northern States account for more than 80% of stunted children.


Worst
x Southern states fare better, accounting for around 13% cases.
affected
x North-Eastern states and islands, accounting for around 4% cases.

The research highlights the need for targeted policy intervention to combat stunting. For this four
crucial parameters concerning women become important which can result in 44% reduction in
stunting among children. These are:
Targeted o Higher body mass index of women;
intervention
o Education;
o Age at marriage; and
o Ante-natal care

Î RELATED INFORMATION

x IFPRI provides research-based policy solutions to sustainably reduce poverty and end hunger and
malnutrition in developing countries.
IFPRI x It was established in 1975 and is headquartered in Washington D.C.
x It is a research centre of Consultative Group on International Agricultural Research (CGIAR), a
worldwide partnership engaged in agricultural research for development.

x NFHS is a large-scale, multi-round survey conducted in a representative sample of households


throughout India.
x The survey provides state and national information for India on fertility, infant and child
mortality, the practice of family planning, maternal and child health, reproductive health, nutrition,
anaemia, utilization and quality of health and family planning services.
x NFHS has two specific goals: a) to provide essential data on health and family welfare needed by
NFHS
the Ministry of Health and Family Welfare and other agencies for policy and programme purposes,
and b) to provide information on important emerging health and family welfare issues.
x The Ministry of Health and Family Welfare (MOHFW) has designated the International Institute for
Population Sciences (IIPS) Mumbai, as the nodal agency, responsible for providing coordination
and technical guidance for the survey.
x The first survey was done in 1992-93.

Key focus points (Prelims): Stunting; Data; IFPRI; NFHS

Women still lag behind on social and economic mobility whether by

PROMOTING facing constraints of glass-ceiling or limited opportunities in terms


of being promoted as entrepreneurs.
WOMEN On the other side, parity in opportunity also helps in catapulting

ENTREPRENEURS the pace of economic growth and development. In full potential


scenario when women participate in the economy, equally to men,
# Inclusive growth # it could add $2.9 trillion to India’s GDP by 2025.

Empowerment However, Indian women entrepreneurs continue to face challenges


in accessing investors and raising capital.

FOCUS | July 2018 | RAU’S IAS 59


@civilminds
Society and Social Justice

RECENT ACTION TO PROMOTE WOMEN ENTREPRENEURS

The UN India Business Forum and the Women Entrepreneurial Platform of NITI Aayog formed
a consortium to reduce gender disparities in start-up investments by providing mentorship and
New initiative
networking opportunities and accelerating financial and market linkages for women
entrepreneurs.

About UN India It is an alliance of India’s businesses, financial institutions, the government and the UN, aims to
Business Forum accelerate India’s rapid growth and achieve the SDGs.

This consortium will bring together key stakeholders, including venture capitalists and impact
Benefits investors, international donor and funding agencies, private sector partners and state
governments.

x Women entrepreneurs will be identified through key partners, including Women


Identification Entrepreneurial Platform (WEP), UN Women, and UNDP.
x The consortium secretariat will then connect entrepreneurs with relevant members.

Î RELATED INFORMATION

x WEP was launched by NITI Aayog on the occasion of International Women’s Day on 8th March, 2018.
x It is aimed at building an ecosystem for women across India to realize their entrepreneurial
WEP
aspirations, scale-up innovative initiatives and chalk-out sustainable, long-term strategies for their
businesses.

x It is an UN organization dedicated to gender equality and the empowerment of women.


x It works globally to make the vision of the Sustainable Development Goals a reality for women
and girls and stands behind women’s equal participation in all aspects of life, focusing on four
strategic priorities:
9 Women lead, participate in and benefit equally from governance systems;
9 Women have income security, decent work and economic autonomy;
UN
9 All women and girls live a life free from all forms of violence;
Women
9 Women and girls contribute to and have greater influence in building sustainable peace and
resilience, and benefit equally from the prevention of natural disasters and conflicts and
humanitarian action.
x The entity fully works to position gender equality as fundamental to the Sustainable Development
Goals, and a more inclusive world.
x Established in 2010 and is headquartered in New York.

x It is the United Nations' global development network.


x It advocates for change and connects countries to knowledge, experience and resources to help
people build a better life.
x It provides expert advice, training and grants support to developing countries, with increasing
emphasis on assistance to the least developed countries.
UNDP
x It promotes technical and investment cooperation among nations.
x To accomplish the SDGs and encourage global development, UNDP focuses on poverty reduction,
HIV/AIDS, democratic governance, energy and environment, social development, and crisis
prevention and recovery.
x Established in 1965 and is headquartered in New York.

FOCUS | July 2018 | RAU’S IAS 60


@civilminds
Society and Social Justice

Key focus points (Prelims): UN India Business Forum; WEF; UN Women; UNDP

PRADHAN MANTRI MATRU VANDANA YOJANA (PMMVY)


#Scheme #Social Justice #Women Empowerment

ABOUT THE SCHEME

Implemented by The Ministry of Women and Child development.

x It was previously known as Indira Gandhi Matritva Sahyog Yojana (IGMSY).


x It is a maternity benefit program. In its new form, it was rolled out in 2017.
x It is a conditional cash transfer scheme for pregnant and lactating women of 19 years of
age or above for first two live births.
What? x It provides a partial wage compensation to women for wage-loss during childbirth and
childcare and to provide conditions for safe delivery and good nutrition and feeding
practices.
x It was brought under the National Food Security Act, 2013 to implement the provision of
cash maternity benefit of Rs. 6,000 stated in the Act.

According to a recent report, the government is able to provide cash incentives to nearly 23.6 lakh beneficiaries.

NFSA, 2013

The National Food Security Act, 2013 (NFSA 2013) converts into legal entitlements for existing food security
programmes of the Government of India. It includes the Midday Meal Scheme, Integrated Child Development Services
scheme and the Public Distribution System.

x To provide for food and nutritional security in human life cycle approach, by ensuring access
to adequate quantity of quality food at affordable prices to people to live a life with dignity.
x It provides for coverage of upto 75% of the rural population and upto 50% of the urban
population for receiving subsidized foodgrains under Targeted Public Distribution System
Objectives (TPDS), and thus covering about two-thirds of the population.
x The eligible persons are entitled to receive 5 Kgs of foodgrains per person per month at
subsidised prices of Rs. 3/2/1 per Kg for rice/wheat/coarse grains.
x The existing Antyodaya Anna Yojana (AAY) households, which constitute the poorest of the
poor, will continue to receive 35 Kgs of foodgrains per household per month.

x Pregnant women and lactating mothers and children in the age group of 6 months to 14 years
will be entitled to meals as per prescribed nutritional norms under Integrated Child
Development Services (ICDS) and Mid-Day Meal (MDM) schemes.

Focus on Women x Such women are also be entitled to receive maternity benefit of not less than Rs. 6,000.
and Children x Eldest woman of the household of age 18 years or above to be the head of the household
for the purpose of issuing of ration cards.
x In case of non-supply of entitled foodgrains or meals, the beneficiaries will receive food
security allowance.

Implementation x It provides that every State Government shall, by notification, constitute a State Food

FOCUS | July 2018 | RAU’S IAS 61


@civilminds
Society and Social Justice

Commission for the purpose of monitoring and review of implementation of the Act.
x It contains provisions for setting up of grievance redressal mechanism at the District and State
levels.

Key focus points (Prelims): PMMVY; NFSA 2013; ICDS; MDM

With reference to the provisions made under the National Food Security Act, 2013, consider the following
statements : (UPSC 2018)
1. The families coming under the category of 'below poverty line (BPL)' only are eligible to receive subsidised food
grains.
2. The eldest woman in a household, of age 18 years or above, shall be the head of the household for the purpose
of issuance of a ration card.
3. Pregnant women and lactating mothers are entitled to a 'take-home ration' of 1600 calories per day during
pregnancy and for six months thereafter.
Which of the statements given above is/are correct ?
(a) 1 and 2
(b) 2 only
(c) 1 and 3
(d) 3 only

U.N. REPORT ON According to the United Nations Sustainable Development Goals


(SDG) 2018 report, the number of hungry people in the world has
GLOBAL HUNGER risen for the first time in more than a decade. The major
contributors to this increase are conflicts (such as Yemen, Syria,
# Report #Justice
Somalia etc.), and climate change which are creating hurdles in
#Nutritional Security meeting development goals. The report covers the data till the year

#Inclusive Development 2016.

FOCUS | July 2018 | RAU’S IAS 62


@civilminds
Society and Social Justice

FACTS ONLY FOR REFERENCE

x In Asia: 520 million


Break-up of hungry
x In Africa: 243 million
population
x In Latin America and the Caribbean: 42 million

x 11% in the entire world


Share of the global
x Asia: 11.7%
population who are
x Africa: 20% (in eastern Africa, 33.9%)
hungry
x Latin America and the Caribbean: 6.6%

CLIMATE CHANGE EXACERBATES THE RISKS OF HUNGER AND UNDERNUTRITION THROUGH:

x Climate change increases the frequency and intensity of some disasters such as droughts, floods
Extreme
and storms. This has an adverse impact on livelihoods and food security.
weather
x Climate-related disasters have the potential to destroy crops, critical infrastructure, and key
events
community assets, therefore deteriorating livelihoods and exacerbating poverty.

Long-term x Sea-level will rise as a result of climate change, affecting livelihoods in coastal areas and river
and gradual deltas.
climate x Accelerated glacial melt will also affect the quantity and reliability of water available and change
risks patterns of flooding and drought.

CLIMATE CHANGE AFFECTS ALL DIMENSIONS OF FOOD SECURITY AND NUTRITION:

x Changes in climatic conditions have already affected the production of some staple crops, and
Food future climate change threatens to exacerbate this.
availability x Higher temperatures will have an impact on yields while changes in rainfall could affect both crop
quality and quantity.

x Climate change could increase the prices of major crops in some regions.
x For the most vulnerable people, lower agricultural output means lower incomes. Under these
Food
conditions, the poorest people who already use most of their income on food will have to sacrifice
access
additional income and other assets to meet their nutritional requirements, or resort to poor coping
strategies.

x Climate-related risks affect calorie intake, particularly in areas where chronic food insecurity is
already a significant problem.
Food
x Changing climatic conditions could also create a vicious cycle of disease and hunger.
utilization
x Nutrition is likely to be affected by climate change through related impacts on food security, dietary
diversity, care practices and health.

The climatic variability produced by more frequent and intense weather events can upset the stability of
Food
individuals’ and government food security strategies, creating fluctuations in food availability, access and
stability
utilization.

CONFLICT AND IMPACT ON FOOD SECURITY

Conflict can reduce food availability and access and can also disrupt health services and basic sanitation.
Impact
The impacts can be direct and indirect and they are:

FOCUS | July 2018 | RAU’S IAS 63


@civilminds
Society and Social Justice

Direct Impact: Destruction of crops and food stocks, forced migration, etc.
Indirect Impact: Disruption of markets, leading to increased food prices or decreased household
purchasing power, or reduced access to water and fuel for cooking, impacting feeding practices.

Î RELATED INFORMATION: SDG

These are collection of 17 global goals set by the United Nations in 2015. "Global Goals for Sustainable Development".
The formal name for the SDGs is: "Transforming our World: the 2030 Agenda for Sustainable Development." That has
been shortened to "2030 Agenda."
The goals were developed to succeed the Millennium Development Goals (MDGs) which ended in 2015. Unlike the
MDGs, the SDG framework does not distinguish between "developed" and "developing" nations. Instead, the goals
apply to all countries.

Key focus points (Prelims): Hunger factsheet; SDG

TOOTH PASTE AND Triclosan, a common ingredient found in toothpastes


ANTIBIOTIC RESISTANCE and hand washes could be contributing to the rise of
antibiotic resistant bacteria.
#Health
THINGS TO UNDERSTAND

x Triclosan (abbreviated as TCS) is an antibacterial and antifungal agent found in some consumer
products, including toothpaste, soaps, detergents, toys, and surgical cleaning treatments..
Triclosan
x New emerging studies have raised the possibility that exposure to triclosan contributes to making
bacteria resistant to antibiotics. Might be harmful to the immune system.

x Antibiotic resistance occurs when bacteria change in response to the use of antibiotic medicines.
Antibiotic
x These bacteria may infect humans and animals, and the infections they cause are harder to treat
resistance
than those caused by non-resistant bacteria.

FOCUS | July 2018 | RAU’S IAS 64


@civilminds
Society and Social Justice

x It leads to higher medical costs, prolonged hospital stays, and increased mortality.

Key focus points (Prelims): Triclosan; Antibiotic resistance

SHARP DROP IN According to a World Health Organisation report, the prevalence


of smoking tobacco in India dropped down to 11.5% in 2005 from
SMOKING 19.4% in 2000. It projected the prevalence to drop down further
TOBACCO IN INDIA to 9.8% by 2020 and 8.5% by 2025. Overall tobacco users among
adults in India are: Smokers 7%, Smokeless 18% and both 4%.
# Health

x It has covered smokers not chewers


What is of tobacco.
not x India has a large population of
covered chewing tobacco users, thus posing
in additional burden.
survey? x Overall, 3/4th tobacco users in India
have it in the chewing form.

x Most people know that using


tobacco causes cancer and lung
disease, but many people are not
aware that it also causes heart
Concern disease and stroke.
x Tobacco use and second-hand
smoke exposure are major causes of
cardiovascular diseases, including
heart attacks and stroke.

Key focus points (Prelims): Data on tobacco users; Harmful effects of Tobacco

Gaming attracts children, adolescents and adults. But, it comes with


GAMING: an induced force of more and more indulgence. It starts as a simple
time pass but gradually becomes a daily habit and sometimes it even
ANOTHER creates negative consequences. There was a suspicion that ‘Blue
Whale’ game prompted suicides.
ADDICTION ON
This addiction to gaming is now taken seriously and in the regard, the
THE PROWL World Health Organisation (WHO) has classified gaming addiction as
mental health disorder in its International Classification of Diseases
#Health
(ICD 11). Let us understand the significance of this move.

FOCUS | July 2018 | RAU’S IAS 65


@civilminds
Society and Social Justice

ABOUT GAMING ADDICTION

x According to WHO, gaming disorder can be understood as a pattern of behaviour characterized by


Pattern of impaired control over gaming, prioritizing gaming over other activities to such extent that gaming takes
behaviour precedence over interests and daily activities.
x Victims generally continue gaming despite occurrence of negative consequences.

For gaming disorder to be diagnosed, the behaviour pattern must be of sufficient severity to result in
Diagnosis significant impairment in personal, family, social, educational, occupational or other important areas of
functioning and would normally have been evident for at least 12 months.

Î RELATED INFORMATION: INTERNATIONAL CLASSIFICATION OF DISEASES (ICD)

x The ICD is the basis for identification of health trends and statistics globally; and the
international standard for reporting diseases and health conditions.
x It is used by medical practitioners around the world to diagnose conditions and by
Purpose
researchers to categorize conditions.
x The inclusion of a disorder in ICD is a consideration which countries take into account
when planning public health strategies and monitoring trends of disorders.

Gaming x A decision on inclusion of gaming disorder in ICD-11 (11th revision of the disease list) is based on

FOCUS | July 2018 | RAU’S IAS 66


@civilminds
Society and Social Justice

Disorder in ICD reviews of available evidence and reflects a consensus of experts from different disciplines and
geographical regions.
x The inclusion of gaming disorder in ICD-11 follows the development of treatment
programmes for people with health conditions identical to those characteristic of gaming
disorder, and will result in the increased attention of health professionals, and relevant
prevention and treatment measures.

ICD 11 will be presented at World Health Assembly in May 2019 for adoption by WHO member
Implementation
states, and it will come into effect on January 1, 2022.
Key focus points (Prelims): ICD

Viewpoint

TOWARDS FOOD SECURITY


# Food Justice
(Courtesy: The Hindu)

Read and ponder!

The right to food is a well-established principle of international human rights law. It has evolved to include an
obligation for state parties to respect, protect, and fulfil their citizens’ right to food security. Our current
understanding of food security includes the four dimensions of access, availability, utilisation and stability. As a
state party to the Universal Declaration of Human Rights and the International Covenant on Economic, Social and
Cultural Rights, India has the obligation to ensure the right to be free from hunger and the right to adequate food.

FROM AVAILABILITY TO ACCESS

Broadly speaking, attitudes towards food security in India can be divided into two generations subsequent to
Independence. While the demarcation is far from exact, it indicates how the importance given to different elements of
food security altered over time.

The years post-Independence were turbulent for India. Memories of the Bengal famine remained fresh and fears of a
food shortage were rampant. Hunger was thought to be a function of inadequate food production. In 1974, the World
Food Conference defined food security primarily in terms of production — as the “availability at all times of adequate
world food supplies.”

It is arguable that the framing of food security in quantitative terms sparked India’s determination to initiate the Green
Revolution to boost food production. While the programme achieved dramatic increases in rice and wheat production
in some parts of the country, it’s devastating environmental impact has also rightly been critiqued.

Two occurrences over the 1980s and 1990s set the stage for what we understand as food security in India today. The
first was when the Supreme Court dramatically expanded the ambit of rights that citizens could claim against the
state. While no explicit ‘right to food’ could be made out, there was an increased mention of food as being among a
cluster of basic rights integral to human dignity. The second was a shift of the frame from the problem of availability
to the problem of access.

The 1980s and 1990s saw an increasing acknowledgement that India’s focus on increasing food supplies was falling
short of actually ameliorating hunger. Even as the data showed that India had transformed from a food deficit nation
to a food surplus one, seminal research by Amartya Sen and others revealed that hunger and food security were tied
to the issue of access — that is, in spite of ample quantities of grain, and a variety of government efforts such as the

FOCUS | July 2018 | RAU’S IAS 67


@civilminds
Society and Social Justice

Public Distribution System, people were dying of starvation because they were unable to physically or financially (or
both) reach this food. This view of food security was mirrored at an international level too. In 1996, the World Food
Summit stated that food security was achieved “when all people, at all times, have physical and economic access to
sufficient, safe and nutritious food.”

This focus on access culminated in India in a 2001 case brought by the People’s Union for Civil Liberties, in which
the Supreme Court evolved a right to food and read it into the right to life provisions of the Constitution. Following
that, a host of court orders and directions ultimately resulted in the 2013 National Food Security Act (NFSA), which
has been lauded for guaranteeing a quantitative “right to food” to all Indians. However, the NFSA suffers from serious
lacunae in its drafting, which severely undermine its stated objective of giving legal form to the right to food in India.

ASSESSING THE FOOD SECURITY ACT

The NFSA surprisingly does not guarantee a universal right to food. Instead, it limits the right to food to those identified
on the basis of certain criteria. It then goes on to further restrict the right to 75% of the Indian population. It also
specifies that a claim under the Act would not be available in times of “war, flood, drought, fire, cyclone or earthquake”
(notably, it is within the Central government’s remit to declare whether such an occasion has arisen). Given that a right
to food becomes most valuable in exactly these circumstances, it is questionable whether the Act is effective in
guaranteeing the right that it is meant to.

Another problematic aspect of the NFSA is its embrace of certain objectives that are to be “progressively realised”.
These provisions include agrarian reforms, public health and sanitation, and decentralised procurement, but they
make no mention of the need to reconsider fundamental assumptions about our agricultural systems and look at food
security in a more comprehensive manner. It is arguable that the rubric of “progressive realisation” actually retards
food security reform in the country. This is because some of the elements mentioned under this head are already
incorporated in laws and policies at the State and national levels. Demarcating them as obligations to be realised
“progressively” will lead to counter-intuitive results where the States will simply refrain from doing any more than what
the NFSA explicitly requires them to do.

Worryingly, the framing of the NFSA as being the final word on government commitments to provide food security to
citizens might instead have the result of limiting the courts with respect to how far citizen entitlements can be
extended. This fear was borne out in the recent Swaraj Abhiyan cases that address the impact of government failures
in tackling consecutive drought years in India. While the court took a strong stance in ordering the executive to
implement the provisions of the NFSA, it was reluctant to go beyond the provisions of the NFSA in terms of what it
could order the government to give citizens. Given that the NFSA predominantly mentions just rice and wheat, and that
too for only some citizens, this has worrying implications.

Finally, while the NFSA addresses issues of access, availability and, even tangentially, utilisation, it is largely silent on
the issue of stability of food supplies — a startling omission given India’s vulnerability to climate change impacts, to
name one impending threat to food security.

Thus there is a need to frame a “third generation” food security law and recognise and mainstream issues
including increasing natural disasters and climate adaptation. Such a framework would robustly address the challenges
facing the country’s food security across all four dimensions and make a coordinated effort to resolve them instead of
the piecemeal efforts that have characterised such attempts so far.

Food security brings together diverse issues such as inequality, food diversity, indigenous rights and environmental
justice. Given the current crises in India, it is time we prepare a third generation right to food legislation that recognises
that a climate-as-usual scenario no longer exists. Such a legislation would ideally be rooted in the principle of a right to
food security in its true spirit and not merely as a sound bite.

FOCUS | July 2018 | RAU’S IAS 68


@civilminds

SCIENCE & TECHNOLOGY


# GS Paper (Prelims) & GS Paper III (Main)

Our Sun (a star) and all the planets around it are part of a galaxy known
CATALOGUING as the Milky Way Galaxy. A galaxy is a large group of stars, gas, and dust
bound together by gravity. They come in a variety of shapes and sizes.
MILKY WAY The Milky Way is a large spiral galaxy.
#Space Our galaxy is called the Milky Way because it appears as a milky band of
light in the sky when you see it in a really dark area.

THINGS TO UNDERSTAND

What’s A 3-D map of more than a billion stars in the Milky Way galaxy, complete with their distance from Earth,
new? colour, and motion through space has been produced from data gathered by Europe’s GAIA satellite.

x It is a space observatory of the European Space Agency.


About
x It is designed for Astrometry: Measuring the positions and distances of stars.
GAIA
x It was launched in 2013. It aims to construct the largest and most precise 3D space catalogue.

x From the brightest of stars, the


level of precision equates to Earth-
based observers being able to
spot a coin lying on the surface of
the moon.
x It has also produced the most
Precision
accurate survey of asteroids in the
solar system.
x It also revealed the orbits of some
14,000 “solar system objects” i.e.
an intricate spider-web of space
rocks orbiting the Sun.

These results allow enhanced study of almost all branches of astronomy from:
9 Traces of the formation of the Solar System;
9 How stars evolve;
9 The current structure of the Solar System;
Benefits
9 The assembly and evolutionary history of the Milky Way;
9 Mapping the distribution of Dark Matter in the Galaxy;
9 Establishing the distance scale in the Universe;
9 Discovery of rare objects.

Further The full data will be published in a series of scientific papers in a special issue of the journal Astronomy &
actions Astrophysics, and thus laying the foundation for further study.

Key focus points (Prelims): Galaxy; Milky Way; Astrometry; GAIA

FOCUS | July 2018 | RAU’S IAS 69


@civilminds
Science & Technology

FARTHEST STAR Twinkle Twinkle little star…..and here comes the farthest individual
SPOTTED star, nicknamed "Icarus". It has been discovered by astronomers by
using NASA’s Hubble Space Telescope.
#Space

THINGS TO UNDERSTAND

Distance The star is located at a distance of 9 billion light years from Earth.

x The phenomenon that made star visible is called “gravitational lensing.”


x It is one of the predictions of Einstein’s theory of General Relativity.
x The presence of matter (energy density) can curve spacetime, and the path of a light ray will be
deflected as a result. This process is called gravitational lensing and in many cases can be
Method of described in analogy to the deflection of light by (e.g. glass) lenses in optics.
discovery x In the case of Icarus, a natural “magnifying glass” is created by a galaxy cluster called MACS,
located about 5 billion light-years from Earth.
x This massive cluster of galaxies sits between the Earth and the galaxy that contains the distant star.
x By combining the strength of this gravitational lens with Hubble’s exquisite resolution and
sensitivity, astronomers can see and study Icarus.

Study of distant stars


x The discovery of Icarus through gravitational lensing has initiated a new way for astronomers to
Benefits study individual stars in distant galaxies.
x These observations provide detailed look at how stars evolve, especially the most luminous stars.

FOCUS | July 2018 | RAU’S IAS 70


@civilminds
Science & Technology

Study of Dark Matter


x The amplification of a single, pinpoint background star provided a unique opportunity to test the
nature of dark matter in the cluster.
x Dark matter is an invisible material that makes up most of the universe’s mass.

Key focus points (Prelims): Icarus; Gravitational Lensing; MACS

WATER ICE UNDER In the desert of northwest Africa, a mineral that requires the
THE SURFACE OF THE presence of water to form is discovered. This mineral is
Moganite.
MOON? #Space

THINGS TO UNDERSTAND

x It is a crystal of silicon dioxide similar to quartz.


About x It is known to form on Earth in specific circumstances in sedimentary settings from alkaline
Moganite fluids.
x It has never before been detected in samples of lunar rock.

x According to the scientists, the mineral is formed on the surface of the moon in the area called
Procellarum Terrane.
x Difficult to detect as water originally present in lunar dirt evaporated due to exposure to strong
About new
sunlight.
discovery
x Moganite is an apt evidence, because moganite forms from the evaporation of water.
x Now there is a good reason to believe that, deeper under the lunar surface, crystals of water ice
could be abundant which is protected from the harsh solar rays.

FOCUS | July 2018 | RAU’S IAS 71


@civilminds
Science & Technology

x It is estimated that the water content in the lunar soil under the surface could be up to 0.6 percent.
x Future moon pioneers could theoretically extract about 1.6 gallons of water per 36 cubic feet (6
Possibilities liters per cubic meter) of lunar rock.
x It would be enough for future astronauts and people that could perhaps live on the moon in the
future to extract enough water to cover their needs

Key focus points (Prelims): Moganite; Silicon Dioxide; Procellarum Terrane

The visible universe including Earth, the sun, other stars, and galaxies
MISSING DARK is made of protons, neutrons, and electrons bundled together into
atoms. But this visible baryonic matter makes up less than 5 percent of
MATTER the mass of the universe. The rest of the universe appears to be made
BAFFLES of a mysterious, invisible substance called dark matter (25 percent) and
a force that repels gravity known as dark energy (70 percent).
SCIENTISTS Unlike normal matter, dark matter does not interact with the

#Space electromagnetic force. This means it does not absorb, reflect or emit
light, making it extremely hard to spot.

THINGS TO UNDERSTAND

x Dark matter’s existence is inferred from its gravitational influence on visible objects, which suggests it
dominates over ordinary matter by a ratio of 5:1.
x Clearest evidence of dark matter comes from tracking stars in the outer regions of galaxies,
Inference which consistently appear to be orbiting faster than their escape velocity, the threshold speed at
of Dark which they ought to break free of the gravitational binds holding them in place and slingshot into
Matter space. This suggests there is unseen, but substantial, mass holding stars in orbit.
x The galaxies in the universe rotate with such speed that the gravity generated by their observable
matter could not possibly hold them together, so here comes the role of Dark Matter which provide
these galaxies extra mass, generating the extra gravity they need to stay intact.

x Several scientific groups, including one at CERN's Large Hadron Collider, are currently working to
Research
generate dark matter particles for study in the lab.

PUZZLE TO SOLVE

x Recently, astronomers have found a distant galaxy where there is no dark matter.
x It is a type of diffuse galaxy. A diffuse galaxy is one that covers enormous areas but have relatively
Baffling
few stars.
discovery
x Even though the galaxy is mostly empty, but there are clusters of densely grouped stars.
x There’s no good explanation why and how this galaxy has no dark matter.

Key focus points (Prelims): Baryonic matter; Dark Matter; Dark Energy; Diffuse Galaxy

DISCOVERY OF A A team of scientists and engineers of Physical Research Laboratory


(PRL), Ahmedabad have discovered a planet around a Sun-like star.
NEW PLANET The size of this planet can be termed either as sub-Saturn or super-
#Space Neptune size.

FOCUS | July 2018 | RAU’S IAS 72


@civilminds
Science & Technology

THINGS TO UNDERSTAND

Name It will be known as EPIC 211945201b or K2-236b.

Mass About the mass of 27 Earth.

Size Six times the radius of the Earth.

Technology Indigenously designed “PRL Advance Radial-velocity Abu-sky Search” (PARAS) spectrograph integrated
issued with 1.2m Telescope at PRL’s Gurushikhar Observatory at Mount Abu.

x It is a ground-based extrasolar planet search device.


x It is the first of its kind spectrograph in Asia, which can measure the mass of a planet going around a
star.
PARAS
x With the help of simultaneous calibration technique, PARAS has achieved a Radial Velocity (RV)
accuracy of 1.3 m/s.
x Thorium-Argon lamp is used for calibration.

x Only 23 such planetary systems (including this discovery) are known to this date.
Importance x With this discovery, India has joined a handful of countries, which have discovered planets around
stars beyond our solar system.

ABOUT

x It is known as the cradle of Space Sciences in India, founded in 1947 by Dr. Vikram Sarabhai.
x It is a unit of Department of Space, Government of India.
PRL,
x It carries out fundamental research in selected areas of Physics, Space & Atmospheric Sciences,
Ahmedabad
Astronomy, Astrophysics & Solar Physics, and Planetary & Geo-Sciences.
x It also manages the Udaipur Solar Observatory.

Key focus points (Prelims): PRL; EPIC; PARAS; RV; Thorium-Argon LAMP

In order to find a viable way for the astronauts at the International Space Station
(ISS) to grow their own vegetables, the scientists have sent broccoli seeds coated
FARMING with a healthy dose of good bacteria to space. These seeds were aboard the
Orbital ATK Cygnus spacecraft.
IN SPACE Three of the seeds are travelling to space as it is, while the other three were
#Space coated with two different species of beneficial bacteria, developed at the
University of Washington, that can live inside crop plants and improve their
growth.

THINGS TO UNDERSTAND

Role of x These are also called as endophytes.


Beneficial x Thy help plants grow better in extreme low-gravity environments, and where nutrients or water
microbes could be lacking.

Why x It is an anti-carcinogenic food source that is a good dietary candidate for deep-space explorers.
Broccoli? x Further specific endophytes and broccoli plants performed well together in greenhouse tests under

FOCUS | July 2018 | RAU’S IAS 73


@civilminds
Science & Technology

growing conditions similar to Mars, where nitrogen and phosphorus are limited.

x The microbes are first encapsulated inside a coating that covers the broccoli seeds, which
protects the seeds from dehydration and allows for safe dry storage before the seeds are hydrated
Process and grown in orbit.
x When the endophyte-coated broccoli seeds reach the space station, they will be hydrated in a small
plant-growth chamber that provides constant light to promote photosynthesis.

x Is to learn how to grow vegetables in the challenging, microgravity conditions of the space station,
Goal of the
and eventually on the Moon and Mars.
experiment
x This will be beneficial for a long haul in future space explorations.

ABOUT ISS

x It is a multi-nation habitable space project and is the largest single structure humans ever put into space. The ISS
includes contributions from 15 nations.
x NASA (United States), Roscosmos (Russia) and the European Space Agency are the major partners of the space
station who contribute most of the funding; the other partners are the Japanese Aerospace Exploration Agency
and the Canadian Space Agency.
x The space station flies at an average altitude of 248 miles (400 kilometers) above Earth and circles the globe every
90 minute.
x It is a platform for long-term research for human health, as a key stepping stone to letting humans explore other
solar system destinations.

Key focus points (Prelims): Endophytes; ISS

WORLD’S MOST POWERFUL With the launch of ‘Summit’, the U.S. has overtaken
Chinese supercomputer Sunway TaihuLight. It is
SUPERCOMPUTER developed by the US Department of Energy's Oak
#Computing Ridge National Laboratory.

THINGS TO UNDERSTAND

x It can complete over 200,000 trillion calculations per second.


Speed x It will also be capable of more than three billion billion mixed precision calculations per second, or
3.3 exaops.

x For research in energy, advanced materials and artificial intelligence (AI).


x It offers opportunities for the integration of AI and scientific discovery, enabling researchers to apply
Purpose
techniques like machine learning and deep learning to problems in human health, high-energy
physics, materials discovery and other areas.

Key focus points (Prelims): Summit; exaops

‘FLYING BRAIN’ As if picked out of Sci-fi movies, ‘CIMON’ a robotic astronaut, which
looks like a flying brain is on its way to the ISS to assist the European
HEADS TO ISS Space Agency's cosmonauts. It is launched onboard SpaceX's Dragon
#Space cargo ship, powered by Falcon 9 rocket.

FOCUS | July 2018 | RAU’S IAS 74


@civilminds
Science & Technology

THINGS TO UNDERSTAND

x It is an artificial intelligence assistant robot made to help the astronauts on ISS.


About
x Crew members will be able to correspond with the flying brain via voice commands. The AI can also
CIMON
detect the mood of the crew members to react accordingly.

x Will help in the study of crystals on the space station, solve a Rubik's cube using video data and play the
What role of an "intelligent camera" to document a medical experiment onboard.
will it
x It can also detect dangerous situations or technical problems to warn the astronauts.
do?
x The goal for this flight is mainly to demonstrate the technology works.

Key focus points (Prelims): CIMON

INTEGRATED Unmanned railway crossings are the most accident prone locations
in India. To deal with these by the use of technology, ISRO has
CHIPS FOR TRAIN developed satellite-based early warning system, to alert road users
against approaching trains at unmanned level crossings of the
ENGINES Indian Railways network.
#Technology The Phase-I trial has been successfully completed.

THINGS TO UNDERSTAND

x Under the project, integrated circuit (IC) chips will be installed on train engines.
x These chips will send a signal to hooters installed at the level crossings with the engine at a distance
Process of four km from the crossing.
x These hooters, linked to the Indian Regional Navigation Satellite System (IRNSS) or NaVIC will
become louder as trains approach the crossings and will fall silent once the trains have passed.

Î RELATED INFORMATION

Integrated x An integrated circuit (IC), sometimes called a chip or microchip, is a semiconductor wafer on which
circuit (IC) thousands or millions of tiny resistors, capacitors, and transistors are fabricated.

FOCUS | July 2018 | RAU’S IAS 75


@civilminds
Science & Technology

chips x An IC can function as an amplifier, oscillator, timer, counter, computer memory, or microprocessor.
x A particular IC is categorized as either linear (analog) or digital, depending on its intended application.

x The Indian Regional Navigation Satellite System (IRNSS) operational name is NAVIC, which also stands
for NAVigation with Indian Constellation.
x It is an autonomous regional satellite navigation system, that provides accurate real-time positioning
NAVIC and timing services.
x It covers India and a region extending 1,500 km around it, with plans for further extension.
x The system at present consists of a constellation of seven satellites.
x 3 satellites are in geostationary orbit (GEO) and 4 geostationary satellites (GSO).

Key focus points (Prelims): IC; NAVIC; GEO; GSO

DOPPLER India usually witness severe weather conditions like violent storms, cloud burst,
heavy rains which cause extreme damage to life and property. To deal with this,
RADARS early warning systems need to be improved. For this, India Meteorological
#Technology Department (IMD) will add 30 Doppler radars in couple of years.

THINGS TO UNDERSTAND

x It is the change in frequency or wavelength of a wave in relation to observer who is moving


relative to the wave source.
x It is named after the Austrian physicist Christian Doppler, who described the phenomenon in
Doppler effect
1842.
x A common example of Doppler shift is the change of pitch heard when a vehicle sounding a
horn approaches and recedes from an observer.

x A Doppler radar is a specialised radar that uses the Doppler effect to produce velocity data
about objects at a distance.
x It does this by bouncing a microwave signal off a desired target and analysing how the
Doppler radar
object's motion has altered the frequency of the returned signal.
x This variation gives direct and highly accurate measurements of the radial component of a
target's velocity relative to the radar.

x It provides precise information about thunderstorms, dust storms, hailstorms, rainfall and
Benefit of wind patterns.
Doppler radar x With a radius of 250 km, it helps in issuing now-casts two-three hours prior to severe weather
conditions

Areas identified The hilly states of Uttarakhand, Himachal Pradesh, Jammu and Kashmir, and states of the North-
for installation East.

x The first Doppler radar was installed in Chennai in 2002.


Current status x Its need became more compelling after the 2005 Mumbai floods.
x There are currently 27 Doppler radars in the country.

Key focus points (Prelims): Doppler Effect; Doppler Radar

FOCUS | July 2018 | RAU’S IAS 76


@civilminds
Science & Technology

QUADRIVALENT In a first, the WHO has issued an official recommendation for a


quadrivalent vaccine. The quadrivalent vaccine will contain four
INFLUENZA influenza virus strains (two A subtypes and two B subtypes — H1N1
and H3N2, and Victoria and Yamagata respectively).
VACCINE Till now, trivalent vaccine is used comprising of two A subtypes and
#Health one B subtype.

THINGS TO UNDERSTAND

x Sanofi Pasteur
Developed by x The viruses used in the vaccine are killed and this eliminates the possibility of the virus in the
vaccine itself causing infection.

x Drug Controller General of India (DCGI) has approved active immunisation of adults of age 18
to 64 years. In India, the vaccine will be available as single dose pre-filled syringe.
x For children, paediatric indication is under review by the DCGI.
Status in India x In the case of H1N1, there are two strains — California and Michigan — that cause influenza.
x In India, the Michigan strain was earlier circulating and has been replaced by the California
strain. For 2018, the WHO has recommended the Michigan strain for the southern hemisphere,
including India.

Vulnerable Pregnant mothers, children aged below five and young people with asthma, cardiovascular disease,
groups in India diabetes and high blood pressure are at a greater risk of infection and death.

Key focus points (Prelims): Subtypes of Influenza; Trivalent and Quadrivalent; strains of H1N1

Viewpoint

NURTURING SCIENTIFIC TEMPER


(courtesy: The Hindu)

On April 22, 2017, “Earth Day,” almost a million people across 600 cities in the US, UK, Europe, Australia, New Zealand
and even the Antarctic, marched for science. It was a non-partisan celebration of science that called for the use of
evidence-based policies by the governments and in the public’s best interest. Scientists and “commoners” asked for
increased governmental belief in, and funding for, science, and for transparency in action.

When American scientists first held a “Rally to Stand up for Science” in February, it was criticising the Trump
government’s denial of climate change and policies that go straight against scientific evidence. But it soon took on a
non-partisan, beyond- the-US rally that highlighted our belief in science and the value of science in human
development. When it was announced that this March for Science will be held on Earth Day, our colleague Subbaiah
Arunachalam asked several of us in the Indian science academies whether we too would March for Science. I believe
we should.

FOCUS | July 2018 | RAU’S IAS 77


@civilminds
Science & Technology

Friends with science

We in independent India have used science and technology for our national development. Former Prime Minister
Nehru gave the clarion call: “let us make friends with science.” And we did so in several forms, in the form of
technology and the application of science for human development. A country devastated by the wounds of partition,
poverty and pestilence, rallied together through the application of science and technology. We rid ourselves of
smallpox and polio, jumped our food grain output from 50 million tons to 270 million in a matter of 60 years. Today,
India contributes almost a third of the world’s supply of childhood vaccines. We supply affordable drugs to the rest of
the world. We provide midday meals to schoolchildren. We undertake to launch satellites for several “developed”
nations. We have gone beyond aiming for the Moon and Mars.

The Aadhar card movement for personal identification and supply of benefits is one of a kind in the world. Recall that
soon after the WWII, over 80 countries became free of colonial bondage and began to rule themselves. Among these,
India alone declared that it will make friends with science and built “modern temples” for human development, and
this policy has borne fruits. It has been policy and not prayers. Prayers are for personal fulfilment; policies are for the
population.

And yet, over the years we have fallen below what we could achieve. Cholera is not gone. Malaria, which we fought 60
years ago, has come back with vengeance and we need new and more science to fight it. Swachh Bharat, a noble
initiative, is not getting anywhere because the people do not abide by its suggestions. Open defecation leads not just to
illness, but stunted development of children, our future generations. Ethiopia, one of the poorest countries in the
world, has shown how to solve the problem, and we need to learn from them. Our school and college systems of
education are a scandal. We know that the best way to control population is through education and awareness, yet are
not correcting the errors. And here we need to learn from Finland, which has the best schooling system in the world.

Stop being passive

We are right now witnessing the effect of the global warming across the country this summer. We in India do not deny
climate change, yet we do not adopt methods that help reverse it. Four thousand trees are planned to be cut in West
Bengal to make room for a highway, and 180 trees are to be cut in the KBR Park in Hyderabad, again for an express
highway, also in Delhi for residences. Logic tells us that newly planted trees takes decades, even centuries, to grow to
their full size, yet we do this. We know that coal-based power houses contribute to global warming, yet permit more
coal mining. These cannot be! Our March for Science will thus be a march for rational thinking and action, by the
people and the politicians. Science is common sense.

The litany can go on. But we can attempt to find solutions in a rational, evidence-based way. It is here that a March for
Science in India would be of value. But it cannot be a one-day affair. We have our annual Indian Science Congresses,
India Science Day (February 28), Technology Day (May 11), Education Day (Nov 11), Children’s Day (Nov 14) and several
more. Let us March for Science on each of these days, year after year, suggest and demand that appropriate policies
are put in place through the use of science and technology for the common good. These marches cannot just be
tokenisms with people wearing T-Shirts and golf caps. They need to be completely non-partisan, non-political, and led
by scientists, engineers, doctors, teachers and common citizens. Let these demand far higher budgets for science,
health, education, and conservation. Let us request funding from private, non-government sources (companies,
individuals, foundations; see how they do so in the US); the government cannot do everything.

Such marches should show examples of success obtained through proper planning (such as the success of IITs, IISERs,
IIMs, vaccines, ISRO, Amul, Delhi Metro….), point out pitfalls and ways to correct them, persuade the public, politicians
and policy makers. They have to be non-partisan and rational to the core, not confrontational but persuasive. It cannot
be a one-shot affair. We need to do this over and over again. It will be a long march, let us tie up our shoelaces.

FOCUS | July 2018 | RAU’S IAS 78


@civilminds

GEOGRAPHY, ENVIRONMENT, BIODIVERSITY &


DISASTER MANAGEMENT
# GS Paper (Prelims) and GS Paper I & III (Main)

India is standing at the cross-roads as it is facing its “worst”


NITI AAYOG ON WATER water crisis in history as the demand for potable water will
outstrip supply by 2030. This was stated by NITI Aayog that
CRISIS ranked 24 States on how well they managed their water;
#Resource #Water crisis based on the parameters of Composite Water
Management index developed by it.

The Composite Water Management Index (CWMI) is


developed to evaluate States for Water management. The
indicators include restoration of water bodies, irrigation,
farm practices, drinking water, policy and governance. As
per it, the current scenario is
x Nearly 600 million Indians faced high to extreme water
stress and about 2,00,000 people died every year due to
inadequate access to safe water
x Twenty-one cities, including Delhi, Bengaluru, Chennai
and Hyderabad will run out of groundwater by 2020,
affecting 100 million people
x There will be a 6% loss in the country’s Gross
Domestic Product (GDP) by 2050 if water crisis
continues
x Water stress could be a threat to the Food security as
well

x The groundwater resources, which account for 40% of India’s water supply, are being depleted
at unsustainable rates.
Critical facts
x Up to 70% of India’s water supply is contaminated.
x About 60% of the States were marked as “low performers”.

x Among them are the states of Uttar Pradesh, Odisha, Chhattisgarh, Jharkhand, Bihar and
Worst
Haryana.
performers
x Himachal Pradesh is the worst performer among a separate 8-member list of States clubbed
together as ‘North-Eastern and Himalayan.’

Better
Is shown by Gujarat, Madhya Pradesh, Andhra Pradesh, Karnataka, Maharashtra and Telangana
management

FOCUS | July 2018 | RAU’S IAS 79


@civilminds
Geography, Environment, Biodiversity & Disaster
Management

ATAL BHUJAL For sustainable ground water management with the community
YOJANA participation in select over-exploited and ground water stressed areas,
the government has proposed Atal Bhujal Yojana (ABHY).
#Conservation #Scheme

ABOUT THE SCHEME

It is designed as a Central Sector Scheme under Ministry of Water Resources, River Development and
Ministry
Ganga Rejuvenation and will be implemented with the assistance of World Bank.

1. To address the criticality of ground water resources in major part of the country.
2. The scheme aims to improve ground water management in priority areas in the country
Aim through community participation.
3. Ensuring active community participation in groundwater management is among the major
objectives of the scheme.

x It envisages active participation of the communities in various activities.


x These activities include:
9 Formation of Water User Associations;
9 Monitoring and disseminating ground water data;
9 Water budgeting;
Community
Participation 9 Preparation and implementation of Gram-Panchayat wise water security plans; and
9 Information, Education and Communication (IEC activities) related to sustainable ground water
management.
x Community participation is also expected to facilitate bottom-up groundwater planning process
to improve the effectiveness of public financing and align implementation of various government
programs on groundwater in the participating states.

x The priority areas identified under the scheme fall in the states of Gujarat, Haryana, Karnataka,
Madhya Pradesh, Maharashtra, Rajasthan and Uttar Pradesh.
x These States represent about 25% of the total number of over-exploited, critical and semi-
Priority
critical blocks in terms of ground water in India.
Areas
x They also cover two major types of groundwater systems found in India - alluvial and hard
rock aquifers- and have varying degrees of institutional readiness and experience in
groundwater management.

x The scheme is to be implemented over a period of five years from 2018-19 to 2022-23 with
World Bank assistance and Rs 6000 crores have been sanctioned for this scheme.
x Funds under the scheme will be provided to the states for strengthening the institutions
responsible for ground water governance, as well as for encouraging community involvement
Implemen-
for improving ground water management to foster behavioral changes that promote
tation
conservation and efficient use of water.
x The scheme will also facilitate convergence of ongoing Government schemes in the states by
incentivizing their focused implementation in identified priority areas.
x Funds under the scheme will be made available to the participating states as Grants.

Expected 1. Better understanding of the ground water regime;


Benefits 2. Focused and integrated community based approach for addressing issues related to ground

FOCUS | July 2018 | RAU’S IAS 80


@civilminds
Geography, Environment, Biodiversity & Disaster
Management

water depletion;
3. Sustainable ground water management through convergence of on-going and new schemes;
4. Adoption of efficient water use practices to reduce ground water use for irrigation; and
5. Augmentation of ground water resources in targeted areas.

URANIUM India’s groundwater aquifers are facing twin crises: Depletion;


and Contamination from Uranium. Over 30 micrograms per
CONTAMINATION OF litre (mcg/l) of Uranium is found in parts of north-western,
southern and south-eastern India. Many residents of these
GROUND WATER regions are drinking drinking contaminated water from the
#Conservation #Pollution wells. Uranium contamination is linked to damage of kidneys.

Terrains with heavy contamination are: Rajasthan


and other north-western regions where uranium
occurs mostly in alluvial aquifers. In southern
regions such as Telangana, crystalline rocks such as
granite seem to be the source.
Reasons for uranium Contamination:
Natural Reasons: Water-rock interactions cause the
uranium to be extracted from those rocks. It creates
a conditions that enhance the extracted uranium's
solubility in water.
Interaction of extracted uranium with other
chemicals in the groundwater, such as bicarbonate
can further enhance Uranium’s solubility in water.
Human Activity : Over exploitation of ground
water for agriculture and irrigation. Over pumping
of groundwater leads to decline in water level which
creates such a condition that enhance uranium
enrichment in the remaining shallow groundwater.

SAFETY STANDARDS

x The WHO has set a provisional safe drinking water standard of 30 micrograms of uranium per
WHO litre.
standards x The U.S. Environmental Protection Agency's (EPA) standards has also set the same limits for safe
drinking water.

x Bureau of Indian Standards (BIS) does not specify a norm for uranium level contamination in
water. Thus, water is not tested regularly for such contamination.
Standards in
x Several studies have revealed water contamination of uranium in Bangalore, Telangana and parts
India
of Andhra to be around 200 to 500 micrograms per litre (mcg/l). This is way above WHO
standards.

x BIS should specify reasonable limits of uranium contamination for drinking purpose.
Way forward
x Steps must be taken in collaboration with BIS and state governments to test ground waters

FOCUS | July 2018 | RAU’S IAS 81


@civilminds
Geography, Environment, Biodiversity & Disaster
Management

contamination for quality monitoring.


x Citizens must be made aware of the health hazards of drinking such contaminated water.

SWAJAL SCHEME The government has launched Swajal scheme in 115 aspirational
districts of the country. It is a community owned drinking water
#Conservation #Scheme programme for sustained drinking water supply.

ABOUT THE SCHEME

Ministry Implemented by the Ministry of Drinking Water and Sanitation.

x It will be implemented under the existing National Rural Drinking Water Programme
(NRDWP) budget.
Implementation x Under the scheme, 90% of the project cost will be taken care by the Government and the
remaining 10% of the project cost will be contributed by the community.
x The Operations and management of the project will be taken care by the local villagers.

Î RELATED INFORMATION: NRDWP

x The National Rural Drinking Water Programme (NRDWP) is a Centrally Sponsored Scheme aimed at providing every
person in rural India with adequate safe water for drinking, cooking and other domestic basic needs on a
sustainable basis.
x In 2009, it was modified from the Accelerated Rural Water Supply Programme.
x It focuses on the creation of the infrastructure for safe water to be readily and conveniently accessible at all times
and in all situations.

ZERO BUDGET NATURAL FARMING (ZBNF)


#Biodiversity #Conservation
Farmers in India increasingly find themselves in a vicious cycle of debt, because of the high production costs,
high interest rates for credit, the volatile market prices of crops, the rising costs of fossil fuel based inputs,
and private seeds. This has led to a deep agrarian crisis that is making small scale farming an unviable
vocation. In this regard ‘Zero Budget Natural Farming’ is a viable solution.
Andhra Pradesh has announced that the State would fully embrace Zero Budget Natural Farming(ZBNF), that
would cover all farmers by 2024.
NITI Aayog has called for the scientific validation of zero budget natural farming (ZBNF) through multi-
location trials. If found to be successful, an enabling institutional mechanism could be set up to promote this
technology so that Indian farmers would benefit.

THINGS TO UNDERSTAND

x It a farming practice that believes in natural growth of crops without adding any fertilizers and
pesticides or any other foreign elements.
ZBNF
x The word ‘Zero Budget’ refers to the zero net cost of production of all crops. The inputs used
for seed treatments and other inoculations are locally available in the form of cow dung and cow

FOCUS | July 2018 | RAU’S IAS 82


@civilminds
Geography, Environment, Biodiversity & Disaster
Management

urine.
x A ZBNF practicing farmer has lower cost of inputs and thus has better capacity to increase the
incomes. At the same time, ZBNF crops helps in retaining soil fertility and is climate change
resilient.

9 Seeds treated with cow dung and urine;


9 Soil rejuvenated with cow dung, cow urine and other local materials to increase microbes;
Process 9 Cover crops, straw and other organic matter to retain soil moisture and build humus;
9 Soil aeration for favorable soil conditions;
9 Natural Insect management methods.

1. In ZBNF, yields of various cash and food crops have been found to be significantly higher when
compared with chemical farming.
2. Input costs are near zero as no fertilizers and pesticides are used
3. Profits in most areas under ZBNF were from higher yield and lower inputs
4. Model ZBNF farms were able to withstand drought and flooding, which are big concerns with
Benefits regard to climate change.
5. The planting of multiple crops and border crops on the same field has provided varied income
and nutrient sources.
6. As a result of these changes, there is reduced use of water and electricity, improved health of
farmers, flourishing of local ecosystems and biodiversity and no toxic chemical residues in the
environment.

Î RELATED INFORMATION: ZERO TILLAGE FARMING

x Zero tillage farming is a way of growing crops or pasture from year to year without disturbing
the soil through tillage.
x No-till is an agricultural technique which increases the amount of water that infiltrates into
the soil, the soil's retention of organic matter and its cycling of nutrients.
What is
x In many agricultural regions, it can reduce or eliminate soil erosion.
meant by
x It increases the amount and variety of life in and on the soil, including disease-causing
this?
organisms and disease organisms.
x The most powerful benefit of no-tillage is improvement in soil biological fertility, making soils
more resilient.
x It forms the part of Conservation Agriculture.

1. Crop residues are distributed evenly and left on the soil surface.
2. No implements are used to turn the soil over, cultivate it, or incorporate crop residues
3. Weeds and/or purpose-planted cover crops are controlled by a pre-planting application of a
non-pollutant desiccant herbicide.
4. A specialized planter or drill cuts through the desiccated cover and residues accumulated on
Process the soil surface, slotting seed (and fertilizer) into the soil with minimal disturbance.
5. Subsequent weed control is carried out with some pre- but mostly post-emergent herbicides,
which also used in conventional tillage.
6. Crop rotation is fundamental to Zero Tillage, since this promotes adequate biomass levels for
permanent mulch cover; it also assists in the control of weeds, pests and diseases, as well as in
improving the physical condition of the soil.

FOCUS | July 2018 | RAU’S IAS 83


@civilminds
Geography, Environment, Biodiversity & Disaster
Management

7. Soil erosion is reduced by about 90% and soil biological activity and bio-diversity are maximized.

Î RELATED INFORMATION: CONSERVATION AGRICULTURE

Conservation Agriculture is a farming system that


promotes:
maintenance of a permanent soil cover, Three principles of Conservation Agriculture:

minimum soil disturbance (i.e. no tillage),


and diversification of plant species.
The first, key step in CA is to minimize the disturbance
Minimum mechanical soil disturbance
of the soil—no plowing or harrowing is necessary. Using
(i.e. no tillage) through direct seed and/or fertilizer
specialized zero-tillage seeders, seed is planted directly
placement.
into undisturbed soil, along with the fertilizer it needs.
This improves the soil fertility and eliminates the costs
associated with pillowing, mainly fuel and labour. Less
fuel also means reduced emissions of greenhouse
gases.
Leaving stubble and other crop residues from the
past harvest on the soil surface and leaving the soil Permanent soil organic cover
undisturbed protects farmland from wind and water (at least 30 percent) with crop residues and/or cover crops.
erosion and from extremes of heat. The occurrence of
dust storms is reduced significantly. It also increases
infiltration of rainfall, reduces runoff, and reduces
evaporation of moisture from the soil surface.
The stubble and crop residues accumulate in the soil, Species diversification
increasing soil organic matter, improving the soil’s through varied crop sequences and associations involving
structure and potentially sequestering carbon. This at least three different crops.
boosts the soil’s ability to hold water and make it
available to plants.

WORLD June 5 is marked by the United Nations as World Environment Day. It was
ENVIRONMENT established by the UN General Assembly in 1972 on the first day of the
Stockholm Conference on the Human Environment. Since it began in
DAY 1974, it has grown to become a global platform for public outreach that is
widely celebrated in over 100 countries.
#Environment
The first WED in 1974 was held with the theme "Only One Earth".
#Conservation
THINGS TO UNDERSTAND

The things associated with it are:

x Focus is on people behaviour and initiative.


People's x The World Environment Day is the "people's day" for doing something to take care of the Earth. That
Day "something" can be focused locally, nationally or globally; it can be a solo action or involve a crowd.
Everyone is free to choose.

FOCUS | July 2018 | RAU’S IAS 84


@civilminds
Geography, Environment, Biodiversity & Disaster
Management

x Each World Environment Day is organized around a theme that focuses attention on a particularly
Theme pressing environmental concern.
x The theme for 2018 is ‘Beat Plastic Pollution’.

x Every World Environment Day has a different global host country, where the official celebrations take
place. The focus on the host country helps highlight the environmental challenges it faces, and
Host
supports the effort to address them.
x This year's host was India.

Being the host nation for the World Environment Day 2018 with the theme
PLASTIC BAN ‘Beat plastic pollution’, India has shown proactive zeal in dealing with the
#Pollution menace. Before this, the GoI notified the Plastic Waste Management Rules,
2016. Recently to take this forward, the government of Maharashtra has
#Conservation started enforcing ban on plastic usage.

PLASTIC WASTE MANAGEMENT RULES, 2016 AIMS TO

x Increase minimum thickness of plastic carry bags from 40 to 50 microns and stipulate minimum thickness of
50 micron for plastic sheets also to facilitate collection and recycle of plastic waste;
x Expand the jurisdiction of applicability from the municipal area to rural areas, because plastic has reached
rural areas also;
x To bring in the responsibilities of producers and generators, both in plastic waste management system and
to introduce collect back system of plastic waste by the producers/brand owners, as per extended producers
responsibility;
x To introduce collection of plastic waste management fee through pre-registration of the producers,
importers of plastic carry bags/multi-layered packaging and vendors selling the same for establishing the
waste management system;
x To promote use of plastic waste for road construction as per Indian Road Congress guidelines or energy
recovery, or waste to oil etc. for gainful utilization of waste and also address the waste disposal issue; to
entrust more responsibility on waste generators, namely payment of user charge as prescribed by local
authority, collection and handing over of waste by the institutional generator, event organizers.

RECENT ACTION BY MAHARASHTRA

x The ban covers the manufacture, use, transport, distribution, wholesale and retail sale, storage
and import of plastic bags with and without handle.
x It also covers disposable products, made from plastic and thermocol (polystyrene), such as single-
use disposable dishes, cups, plates, glasses, fork, bowl, container, disposable dish/bowl used for
packaging food in hotels, spoon, straw, non-woven polypropylene bags, cups/pouches to store
liquid, packaging with plastic to wrap or store the products and packaging of food items and grain
Coverage
material.
x Not covered: Plastic used for packaging of medicines, compostable plastic bags or material used
for plant nurseries, handling of solid waste, plastic bags not less than 50 micron thickness used
for packaging of milk (with the specific purpose printed on it), plastic manufactured for export in
SEZs and plastic to wrap the material at the manufacturing stage are excluded from the ban.
x The ban is applicable to manufacturers and consumers as well as the chain in between, which

FOCUS | July 2018 | RAU’S IAS 85


@civilminds
Geography, Environment, Biodiversity & Disaster
Management

includes shops, hawkers, vendors and offices.

Disaster
Will lead to unclogging of drainage network and can prevent urban floods in city like Mumbai.
mitigation?

FOR IMPLEMENTATION…

x Lack of effective Recycling due to ineffective segregation of plastic waste at source. This
Segregation at source seems difficult due lack of awareness, official support or infrastructure to
Challenges handle waste.
x Companies which generate plastic waste do not follow the principle of Extended Producer
Responsibility (EPR).

Potential of recycling needs to be realised.


x In order to recycle the plastic, we need to segregate the waste at the source and segregated
waste should be transported and treated separately.
x Primary responsibility for collection of used plastic and multi-layered plastic sachets (Branded
Chips, Biscuits and Snack Packets) lies with their producers, importers and brand owners.
x These companies can collectively implement Extended Producer Responsibility (EPR) by
geographically dividing a region into zones and handle the waste generated in their designated
Way forward
zones.
x This strategy was used in Switzerland to recycle Thermocol.
x Companies, large corporates and governments can co-operate to implement innovative
means to realise the monetary value of plastic disposed through recycling while
simultaneously investing in ways and means to phase out the use of plastics.
x However, the most important aspect for managing plastic waste is to generate awareness
amongst plastic users.

The CBD is an international legally-binding treaty signed by 168


CONVENTION ON countries, came into force on 29 December 1993. Its major goals
BIODIVERSITY include biodiversity conservation; the sustainable use of
biodiversity; and the fair and equitable sharing of benefits arising
(CBD): Boon or from genetic resources originating from plants, animals, or
microbes. These aims are further formalised in the Nagoya
bane Protocol.
#Biodiversity #Environment India ratified CBD in 1994

CONCERNS RAISED BY RESEARCHERS

x Researchers have questioned stringent regulations in CBD, as they limit biodiversity research, curtails
international collaboration and restricts access to genetic resources.
x With the enforcement of CBD, the signatory countries now own the biodiversity that lies within their
boundaries and can make use of the genetic resources appropriately. This provision has led to many
countries stepping up legislative processes to tighten their control over genetic and biodiversity
resources in their territory.

FOCUS | July 2018 | RAU’S IAS 86


@civilminds
Geography, Environment, Biodiversity & Disaster
Management

x As a consequence, obtaining permits for entry into protected areas and access to collect biological
samples or specimens for non-commercial research has become increasingly difficult in many countries
across South Asia, East Africa, and South America.
x This also includes countries like Brazil and India that have global biodiversity hotspots like the Amazon
rainforests, the Western Ghats and the Eastern Himalayas.

CONVENTIONS RELATED TO BIODIVERSITY

x The Cartagena Protocol on Bio-safety to the Convention on Biological Diversity is an


international treaty governing the movements of living modified organisms (LMOs) resulting
Cartagena
from modern biotechnology from one country to another.
Protocol
x It was adopted in 2000 as a supplementary agreement to the Convention on Biological Diversity
and entered into force in 2003.

x The Nagoya Protocol on Access to Genetic Resources and the Fair and Equitable Sharing of
Benefits Arising from their Utilization to the Convention on Biological Diversity is a supplementary
agreement to the Convention on Biological Diversity.
x It provides a transparent legal framework for the effective implementation of one of the three
objectives of the CBD: the fair and equitable sharing of benefits arising out of the utilization of
Nagoya genetic resources.
protocol x The Protocol was adopted in 2010 in Nagoya, Aichi Province, Japan, and entered into force in
2014.
x Its objective is the fair and equitable sharing of benefits arising from the utilization of genetic
resources, thereby contributing to the conservation and sustainable use of biodiversity.
x The Strategic Plan consists of 20 new biodiversity targets for 2020, termed the 'Aichi Biodiversity
Targets'.

The 20 Aichi Targets are divided into 5 sections:


1. Strategic Goal A: Address the underlying causes of biodiversity loss by mainstreaming
biodiversity across government and society
'Aichi
2. Strategic Goal B: Reduce the direct pressures on biodiversity and promote sustainable use.
Biodiversity
Targets' 3. Strategic Goal C: To improve the status of biodiversity by safeguarding ecosystems, species and
genetic diversity.
4. Strategic Goal D: Enhance the benefits to all from biodiversity and ecosystem services.
5. Strategic Goal E: Enhance implementation through participatory planning, knowledge
management and capacity building.

x Popularly known as the International Seed Treaty.


x International agreement in harmony with the Convention on Biological Diversity.
International
Treaty on Aims at:
Plant 9 Recognizing the enormous contribution of farmers to the diversity of crops that feed the
Genetic world
Resources
9 Establishing a global system to provide farmers, plant breeders and scientists with access to
for Food and
plant genetic materials
Agriculture
9 Ensuring that recipients share benefits they derive from the use of these genetic materials
with the countries where they have been originated.

FOCUS | July 2018 | RAU’S IAS 87


@civilminds
Geography, Environment, Biodiversity & Disaster
Management

SOLUTIONS SUGGESTED BY RESEARCHERS

x They suggest that the “Seed Treaty”, which ensures worldwide public accessibility of genetic resources of
essential food and fodder, could be used as a model for exchange of biological materials for non-commercial
research.
x Another solution may be to add an explicit treaty or annex in the CBD to promote and facilitate biodiversity
research, conservation, and international collaboration

The Chandrabhaga beach on the Konark coast of Odisha is the


‘BLUE FLAG’ TAG FOR first in Asia to get the Blue Flag certification. This tag is given to
environment-friendly and clean beaches, equipped with
ODISHA BEACH amenities of international standards for tourists. It was awarded
th
#Conservation #Environment the honour on the World Environment Day celebrated on 5
June.

THINGS TO UNDERSTAND

x These standards were established by the Copenhagen-based Foundation for Environmental


Education (FEE) in 1985.
x The Blue Flag is a trademark owned by FEE which is a not-for-profit, non-governmental organisation.
Blue Flag x Blue Flag criteria includes standards for water quality, safety, environmental education and
Standards information, the provision of services and general environmental management criteria.
x To achieve the Blue Flag standards, a beach must be plastic-free and equipped with a waste
management system. Clean water should be available for tourists, apart from international
amenities. The beach should have facilities for studying the environmental impact around the area.

x The Environment Ministry started the Blue Flag project in December 2017.
Blue Flag
x Twelve more beaches in the country are being developed by the Society for Integrated Coastal
project in
Management (SICOM), an Environment Ministry’s body working for the management of coastal
India
areas, in accordance with the Blue Flag standards.

x Society for Integrated Coastal Management (SICOM) has been established under the aegis of Ministry
of Environment, Forests and Climate change.
SICOM x It is implementing the World Bank assisted Integrated Coastal Zone Management (ICZM) project.
x The project has four components viz., National ICZM Capacity Building, and Piloting ICZM approaches
in Gujarat, Orissa and West Bengal.

India is among the countries that have acceded to the


DRAFTING OF ANTARCTIC Antarctica Treaty. So it is expected to have a clear policy
POLICY on the consequences of its activities in the region. In
this regard, the Ministry of Earth Sciences is tasked to
#Conservation #Environment draft a dedicated Antarctic policy.

THINGS TO UNDERSTAND

Antarctica Treaty x The Antarctic Treaty was signed in Washington on 1st December 1959 by the twelve

FOCUS | July 2018 | RAU’S IAS 88


@civilminds
Geography, Environment, Biodiversity & Disaster
Management

countries. It entered into force in 1961 and has since been acceded to by many other
nations.
x The total number of Parties to the Treaty is now 53.
x The treaty is framed to ensure ‘in the interests of all mankind that Antarctica shall
continue forever to be used exclusively for peaceful purposes and shall not become the
scene or object of international discord.’
x It prohibits military activity, except in support of science; prohibits nuclear explosions
and the disposal of nuclear waste; promotes scientific research and the exchange of data;
and holds all territorial claims in abeyance.

Related Conventions
x The Conservation of Antarctic Seals (1972)
to the Antarctica
x The Conservation of Antarctic Marine Living Resources (1980)
treaty

x Dakshin Gangotri, the first Indian base was established in 1984. It is currently being used
as a supply base and transit camp
Indian presence in x The government is rebuilding its station, Maitri which is the second permanent station in
Antarctica Antarctica.
x Bharati is India's third Antarctic research facility and one of two active Indian research
stations, alongside Maitri.

MAITRI IRRIGATION PROJECT


#Bilateral Relations

x Recently, India has extended a financial aid of about Rs 10 crores to Nepal as the final payment for the Nepal-
Bharat Maitri Irrigation Project.
x This project was launched in January 2017 to boost growth of Nepal’s agricultural sector through enhanced
facilities. It is aimed at installing 2,700 shallow tube wells in 12 districts of Nepal.
x The project would ensure all-season irrigation facility to about 8,000 hectares of farm land to augment
productivity of wheat, rice and seasonal fruits, vegetables and other crops.
x It would also uplift the socio-economic status of farming families in the 12 districts covered under the project.

Desertification is the degradation of land in arid, semi-


arid and dry sub-humid areas. It is caused primarily by
WORLD DAY TO COMBAT human activities and climatic variations. Desertification
does not refer to the expansion of existing deserts.
DESERTIFICATION AND It occurs because dryland ecosystems, which cover over

DROUGHT one third of the world‘s land area, are extremely


vulnerable to overexploitation and inappropriate land
#Conservation use. Poverty, political instability, deforestation,
overgrazing and bad irrigation practices can all
undermine the productivity of the land.

FOCUS | July 2018 | RAU’S IAS 89


@civilminds
Geography, Environment, Biodiversity & Disaster
Management

x 17th June is observed every year to promote public awareness of international efforts to
World Day to
combat desertification.
Combat
x The day is a unique moment to remind everyone that land degradation neutrality is
Desertification
achievable through problem-solving, strong community involvement and co-operation at all
and Drought
levels.

Theme for 2018 The theme for 2018 is “Land has true value – invest in it.”

STATE FRUIT OF TRIPURA


#Conservation

Tripura's Queen variety Pineapple is declared as the ‘State Fruit ‘by the President of India. Export of the pineapple
to foreign countries is a major step for connecting the state with the global trade. Tripura holds a very important
position to develop trade with ASEAN countries including neighbouring Bangladesh.

Viewpoint

DECLINE OF POLLINATORS THREATENS FOOD SUPPLY


# Conservation # Sustainable development
(courtesy: The Hindu)

Most of our staple food crops such as wheat, rice, sorghum, barley and maize do not require animals for their
pollination. However, wild pollinators play a very important role in the production of other crops such as some pulses,
sunflower seeds, cardamom, coffee, cashew nuts, oranges, mangoes and apples. An army of more than 20,000 species
of pollinators including birds, bats and insects service these crops. For most of our food crops, though, the most
important pollinators are the thousands of species of bees. The annual economic value of the crops pollinated by
animals worldwide is estimated to be between $235 billion and $577 billion (in 2015).
The wild pollinators are declining, and their loss will imperil our food supply, warns a recent United Nations report,
based on the global assessment of pollinators by an international team of more than 75 scientists from different parts
of the world, including India. The large scientific panel was brought together by the Intergovernmental Platform on
Biodiversity and Ecosystems Services (IPBES). Endorsed by the governments of 124 countries, the report was released
in Kuala Lumpur.
Created in 2012 by more than 100 governments, the IPBES seeks to provide scientific information about biodiversity
and ecosystem services to policymakers of the member countries. The IPBES, with its secretariat in Germany, is
administered by the UN, including the United Nations Environmental Programme (UNEP) and the United Nations
Development Programme (UNDP).
How does the IPBES define ‘ecosystem services’? They are the many benefits which society derives from nature.
They include fresh water; fertile soil; wild plant resources such as foods, fibres, medicinal plants and the wild relatives
of crops; wild pollinators and the natural enemies of crop pests; carbon sequestration from the atmosphere; and the
important spiritual, aesthetic and recreational values of nature.

FOCUS | July 2018 | RAU’S IAS 90


@civilminds
Geography, Environment, Biodiversity & Disaster
Management

The IPBES was established as an intergovernmental body akin to the Intergovernmental Panel on Climate Change
(IPCC). Reports of the IPCC have become the basis of policies on climate change. It is assumed that the IPBES will have
the same impact on policymaking as the IPCC.

Indian context
According to the IPBES report, the pollinator declines are well-documented in North America and Europe but have not
yet been well-researched in other parts of the world.
In India, the important pollinators of food crops are various species of honeybee, Apis, such as A. Dorsata, A.
Cerana, A. Florae, A. Andreniformes and A. Laboriosa. The European honeybee, A. Mellifera, also pollinates many
crops and fruits such as apples.
Many of these pollinators are declining. For almost 20 years, my colleagues at Ashoka Trust for Research in Ecology and
the Environment (ATREE) have been monitoring the abundance of colonies of the giant Asian bee, Apis Dorsata, in
Biligiri Rangana Hills near Mysore. The number of bee colonies has shrunk significantly over the last decade. Many
other researchers have also reported a decline in the number of honeybee colonies in India.
In the Himalayas, apple yields in recent years have decreased. The decreases have been attributed to reduction in the
number of bees, but the exact causes of low yields are not known. In general, for the country as a whole, we have a
very poor knowledge of the pollination systems of our animal pollinated crops, and how best we can manage the
pollinators for optimal yields.
How are our wild and managed pollinators responding to ongoing loss and fragmentation of natural habitats? What
are the effects of widespread pesticide use? Is climate change implicated in the spread of new diseases among
honeybee colonies? These questions cannot yet be answered, but they may all be factors in the loss of pollinators
documented in the IPBES report.
And this is a potential crisis not only for biodiversity but also for our agricultural economy. The economic stakes are
huge. The value of animal-pollinated crops in India is in the tens of billions of dollars. Poor management of our
pollinator species may be leading to lower crop yields and to losses of hundreds or thousands of crores annually.
Compared with this, our level of investment in research on pollinators has been negligible.

What should be done?


The IPBES report makes a number of recommendations to restore the integrity of pollinators: improvements in the
science of pollination, better land management, strong regulations underlying pesticide use, and restoration and
protection of habitats for wild pollinators. Above all, there is an urgent need for monitoring wild pollinators, and for
strengthening the governance of natural assets.
The Ministry of Environment, Forests and Climate Change has recently launched a programme to establish a network
of Indian Long Term Ecological Observatories (I-LTEO) to monitor the country’s ecosystems. The I-LTEO network offers
tremendous opportunities to monitor wild pollinators.
But pollinators span wild and managed habitats, agricultural and urban landscapes. Pollinators in urban areas can
service and enhance food production in peri-urban areas. Wild biodiversity, including pollinators, must become a
significant component of future ‘smart cities’.
This brings us to a key point regarding pollinators: it is not only the science that requires attention. Policies and
governance for managing landscapes — natural, agricultural, urban — are equally important. The IPBES assessment
serves notice to government agencies that they must rethink conventional sectoral approaches and narrow disciplinary
perspectives. There are many factors involved in the complex environmental challenges threatening human security
today. Only well-integrated approaches can successfully address them..

FOCUS | July 2018 | RAU’S IAS 91


@civilminds

History, HERITAGE & Culture


# GS Paper (Prelims) & GS Paper I (Main)

CHALUKYAN In one of the rarest discoveries, a rare red sandstone sculpture slab of
Lord Siva and Goddess Parvati was found at a Chalukyan temple in
SCULPTURE Satyavolu village of Prakasam district (Karnataka). This sculpture belongs
th
# HERITAGE AND CULTURE to early 7 century A.D. Chalukyan School of Art.

DESCRIPTION OF THE SCULPTURE

x Portrays Lord Siva as the therapeutic physician (Rudra


Bhaishajana) as described in Rigveda. In this posture, he holds
a bowl in his left hand. The bowl contains medicine from herbs
to revive the ailing horse lying at his feet.
x It is a highly exceptional iconographic form of Lord Siva had
not been discovered so far.
x The Lord is gracefully seated on a pedestal with the left leg on
the seat and the other with knee bent and resting on the
ground.
x With neatly entangled hair and knotted in a mountain dweller
fashion.
x Goddess Parvati is in a gracious posture standing to Siva’s left,
carrying a vessel in her right hand, while the left hand rests on
her waist.
x Beneath the panel is a horse standing with a lowered head.

Î THINGS TO KNOW: CHALUKYAS AND THEIR ARCHITECTURE

x THE Chalukya rulers of Vatapi (or Badami) ruled the


central Deccan from A.D. 540 to A.D. 757.
x Pulakesin I was the real founder of the Chalukya
dynasty of Badami.
x He adopted the title Vallabheshvara and performed
the ashvamedha.
Great patrons
x Pulakesin II checked the forces of Harshavardhana
of art and
on the banks of the Narmada.
architecture
x Chalukyas have left behind a wealth of temple
architecture in Aihole, Badami and Pattadakal.
x The architectural style that developed in this part is
known as the Chalukyan style.
x Chalukyan artists blended the Indo-Aryan Nagara
and Dravidian styles and evolved their own distinctive style.

FOCUS | July 2018 | RAU’S IAS 92


@civilminds
History, Heritage & Culture

Reasons x They wanted to:


behind 9 Overawe their enemies and subjects with an enormous show of wealth and power, represented by
building these temples.
huge
9 Impress their subjects with a show of love for the religion.
temples
9 Atone for the sins, they had committed by killing innumerable innocent people in the bloody wars they
fought.

Temples in Aihole Temples in Vatapi (Badami) Temples in Pattadakal

x Aihole, a small village on the banks of the x Vatapi is a small town x Pattadakal is another small
Malaprabha river, was the cradle of located on the bank of village on the banks of the
ancient Hindu temple architecture. Malabrabha river. Malaprabha. This place was
x It was here that the artisans worked on x It was the original capital of considered to be very auspicious
the rocks to create the earliest rock-cut the Chalukya empire and holy. The Chalukya kings
shrines based on Buddhist monuments. founded by Pulakesin I in were crowned here.

x Ravana Phadi is one of the oldest rock- the 6th century A.D. x During the middle of the 7th
cut temples in Aihole. There is a Sivalinga x Here are four beautiful century, temple building activity
in the inner room. The walls and sides of rock-cut temples carved shifted from Badami to
the temple are covered with large figures. out of sandstone hills. Each Pattadakal.
The sculptures are superb - especially that has a square sanctum, a x The Mallikarjuna and the larger
of dancing Siva. hall with pillars, and a Virupaksha temples were built
x Durga temple dates back to the 7th pillared verandah. The around A.D. 740 by
century A.D. This is a Hindu adaptation halls have exquisite carvings Trailokyamahadevi and
of the Buddhist Chaitya Hall with a and sculptures. Lokamahadevi. They were the
rounded end. x Of the four temples, the queens of Vikramaditya II.

x The temple has perforated windows and a fourth is actually a Jain x They build these temples to
statue of Mahishasuravardhini - Durga temple. It has a statue of commemorate their husband's
destroying the demon Mahishasura. Mahavira. victory over Nandivarman, the
x Among the masterpieces in Pallava king of Kancheepuram.
x The Lad Khan temple was also built
around the 7th century A.D. It is a unique these rock-cut temples is the x Vikramaditya II brought
temple, with a surprising resemblance to famous eighteen armed artisans from Kancheepuram.
the Parliament House. It is believed that statue of Nataraja (Siva in The Virupaksha temple clearly
this structure was not meant to be a the dancing pose). shows the influence of the
temple but a meeting place. Kancheepuram style.

KAMAKHYA TEMPLE
# HERITAGE & CULTURE
In our society, biological cycle of menstruation among women is usually hushed up or abhorred in
conversation, and also there has been practices of debarring women from certain places of worship due to
it.
Contrary to this, there is a four-day fair, Ambubachi Mela at Kamakhya temple in Guwahati, Assam to mark
the annual menstruation of the goddess at this temple.
It is believed by the devotees that the presiding goddess of the temple, Devi Kamakhya, the Mother Shakti,
goes through her annual cycle of menstruation during this time stretch.

FOCUS | July 2018 | RAU’S IAS 93


@civilminds
History, Heritage & Culture

ABOUT KAMAKHYA TEMPLE

Location It is situated on the Nilachal Hill at Guwahati city in Assam.

x It has association with three major dynasties.


9 The first epigraphic notice of Kamakhya is found in the 9th-century Tezpur plates of
Vanamalavarmadeva of the Mlechchha dynasty, Assam.
9 The temple got destroyed in the 16th century and the ruins of the temple was said to have been
Historical discovered by Vishwasingha, the founder of the Koch dynasty. He revived worship at the site.
facts 9 It was during the reign of his son, Naranarayan, that the temple reconstruction was completed in
1565.
9 By the end of 1658, the Ahoms under king Jayadhvaj Singha had conquered the Lower Assam
and their interests in the temple grew. In the decades that followed the Ahom kings, all who were
either devout Shaivite or Shakta continued to support the temple by rebuilding and renovating it.

RAMANUJA Under Statue of Equality Project, sculpting is on full swing to create a


seated figure of Bhakti saint Ramanujacharya in Hyderabad. Once
# PERSONALITY# CULTURE inaugurated, it will become second tallest seated statue in the world
#MEDIEVAL INDIA after Great Buddha of Thailand.

ABOUT RAMANUJA

x Ramanuja was born in Tamil Nadu in the eleventh century and he was deeply influenced by the
Alvars.
x Ramanuja was the earliest exponent of Bhakti movement.

Pioneer of x Ramanuja gave a philosophic basis to the teachings of Vaishnavism.


the Bhakti x He provided an intellectual basis for the practice of bhakti (devotional worship) in three major
movement commentaries:
9 The Vedartha-samgraha (on the Vedas, the earliest scriptures of Hinduism),
9 The Shri-bhashya (on the Brahma-sutras), and
9 The Bhagavadgita-bhashya (on the Bhagavadgita).

x Ramanuja laid emphasis on Bhakti as the principle means of attaining the supreme realty (God)
or final bliss.
x According to him, the best means of attaining salvation was through intense devotion to
Vishnu.
Main x He propounded the doctrine of Vishishtadvaita or qualified oneness in that the soul even when
philosophy united with the Supreme God remained distinct.
x Ramanuja’s doctrine greatly inspired the new strand of bhakti which developed in north India
subsequently.
x He held that even the Sudras and outcastes could also attain salvation by completely
surrendering to the will of the guru.

Like many Hindu thinkers, he made an extended pilgrimage, covering India from Rameswaram (part of
Extended
Adam’s Bridge), along the west coast to Badrinath, the source of the holy river Ganges, and returning
pilgrimage
along the east coast.

FOCUS | July 2018 | RAU’S IAS 94


@civilminds
History, Heritage & Culture

x Tradition has it that later he suffered from the zeal of King Kulottunga of the Chola dynasty, who
Other adhered to the god Shiva, and withdrew to Mysore, in the west.
important x There he converted numbers of Jains, as well as King Bittideva of the Hoysala dynasty.
facts x This led to the founding in 1099 A.D. of the town Milukote (Melcote, present Karnataka state) and the
dedication of a temple to Selva Pillai (Sanskrit, Sampatkumara, the name of a form of Vishnu).

KABIR
th
On the occasion of 500 death anniversary of Kabir, the Prime Minister
# PERSONALITY # CULTURE of India paid tributes at his final resting place in Sant Kabir Nagar.
# MEDIEVAL INDIA
ABOUT KABIR

Kabir was one of the most influential saints. His time period was the fifteenth-sixteenth centuries. He was brought
up in a family of Muslim julahas or weavers settled in or near the city of Benares (Varanasi). He was a disciple of the
philosopher-reformer Ramananda.

x Being a mystic and poet, he attempted to bridge Hindu and Muslim thoughts and preached the
essential unity of all religions and the essential equality of all people.
Kabirpanthi x His teachings became the forerunner of Kabirpanth. The main element of Kabirpanth was the
veneration of one God.
x Kabirpanthis acknowledges Kabir as its founder.

x Kabir’s ideas were collected from avast collection of verses called Sakhis and Pads.
x It was composed by him and sung by wandering bhajan singers.
x Some of these were later collected and preserved in the Guru Granth Sahib, Panch Vani and
Collection of
Bijak (it was written by him).
verses
x Bijak is the compilation of the verses of Kabir. It presents Kabir's compositions in a simple style
and exhorts his listeners to shed their delusions, pretensions and orthodox views in favour of a
direct experience of truth.

x Kabir’s teachings were based on a complete, indeed vehement, rejection of the major religious
traditions.
x His teachings openly ridiculed all forms of external worship of both Brahmanical Hinduism
Kabir’s and Islam, the pre-eminence of the priestly classes and the caste system.
teachings x The language of his poetry was a form of spoken Hindi widely understood by ordinary people.
x He also sometimes used cryptic language, which is difficult to follow.
x Kabir believed in a formless Supreme God and preached that the only path to salvation was
through bhakti or devotion. Kabir drew his followers from among both Hindus and Muslims.

x Kabir’s iconoclastic/critical ideas are found showing the way forward. His ideas challenged
Ideas against
discriminatory social norms, defied religious orthodoxy.
discriminatory
x It protested against the arbitrariness of political regimes which had no or little regard to the
social norms
idea of justice for all.

Pathfinder of x He was the pioneer of the inclusive cultural and political traditions.
inclusive x This inclusive culture and political traditions were epitomised by intellectuals and rulers like
culture Abul Fazl and Akbar.

FOCUS | July 2018 | RAU’S IAS 95


@civilminds
History, Heritage & Culture

x They followed it with their broad vision and strong emphasis on creating conditions for peaceful
coexistence of a large public in all their spectacular diversity.
x There are similarities between the teachings of Kabir, Mahatma Gandhi and Ambedkar who
have fought against oppressive political regime.
x And they sought justice for those at the bottom of the social order.

x Kabir has been criticised for his depiction of women. Kabir's opinion of women is contemptuous
and derogatory.
Criticism? x Wendy Doniger concludes Kabir had a strongly prejudice against women.
x For Kabir woman is "Kali nagini (a black cobra), kunda naraka ka (the pit of hell), juthani jagata ki
(the refuse of the world)". According to Kabir, a woman prevents man's spiritual progress.

This battle was the game changer for the initiation and spread of colonial power in
BATTLE the sub-continent. From Bengal, the colonialism gradually spread its tentacles far

OF and wide.
This battle was fought on 23rd June, 1757 and now 261 years have passed. One of
PLASSEY the hidden facts about this battle was the help provided to British by (Armenians),
a trading community that had fled persecution in Persia and settled in India in
# MODERN INDIA
large numbers during the Mughal era.

Î THINGS TO UNDERSTAND: BATTLE OF PLASSEY

x In the early 18th century, India was a conflict zone of business interests. There was tussle
among European powers, native rulers, and the local or migrant merchants.
x The major business interest was related to opium, saltpetre, textiles, spices, and bullion.
x In 1756, anticipating French and Dutch fortifications in Bengal, the English began reinforcing
troops at Fort William, their ramparts in Calcutta.
x In this scenario, Siraj ud-Daulah, the last independent Nawab of Bengal, had just succeeded to
the throne, after his grandfather Ali Vardi Khan.
x Infuriated at this activity, he asked the English to stop their fortifications. And when they
Background ignored him, Siraj ud-Daulah attacked the fort and its neighbouring church.
x On June 26, 1756, the British forces surrendered, Calcutta was renamed Alinagar and a
mosque was ordered to be built inside the fort.
x The Nawab captured the British enclave in Cossimbazar, near Murshidabad, and imprisoned
many British officers, including a young Warren Hastings.
x Black Hole Tragedy: In Fort William, about seventy officers and soldiers of an English company,
which included Indian, Portuguese and Armenian soldiers, were herded into the fort’s small
prison. Overnight, 43 of them died due to asphyxiation, in an incident that became infamous as
the Black Hole of Calcutta.

x Exactly a year later, Robert Clive exacted revenge at Plassey.


Battle of
x With the help of the Nawab’s uncle, Mir Jaffar, and local moneylenders, the Jagat Seths, Siraj ud-
Plassey-
Daulah was betrayed.
Revenge by
British x They met for battle on the field of Plassey on June 23, 1757. The fateful battle of Plassey was a
battle only in name. The major part of the Nawab’s army, led by the traitors Mir Jafar and Rai

FOCUS | July 2018 | RAU’S IAS 96


@civilminds
History, Heritage & Culture

Durlabh, took no part in the fighting.


x Only a small group of the Nawab’s soldiers led by Mir Madan and Mohan Lal fought bravely and
well. The Nawab was forced to flee on a camel and was captured and put to death by Mir Jafar’s
son Miran.

SIGNIFICANCE OF THE BATTLE

1) The battle of Plassey exposed the subcontinent’s internal conflicts, destroying the native dynasties then in
power and also the economy of Bengal.
2) It boosted British prestige and raised them to the status of a major power for the Indian empire. They got access
to rich revenues of Bengal which enabled them to organize a large army and meet the cost of the conquest of rest
of India.
3) The company even went to the extent of establishing a mint at Calcutta and introduced its own currency to
prove its sovereign authority in Bengal.
4) The establishment of control over Bengal made it possible for the English to conquer Northern India.
5) The Battle of Plassey saved the British in Bengal from commercial rivalry of other European powers. The French
were forcibly ousted from Bengal for all time to come and no other European powers ever ventured to challenge
the English authority in this part of India.
6) Clive tried to expand the British influence in the South at the cost of the financial resources of Bengal.
7) Britishers got a base to fight other powerful rulers of India like Marathas, Mysore etc.

HOW ARMENIANS COME INTO THE PICTURE?

x There was a mass-exodus of Armenians from Persia, Turkey and Afghanistan into India due to
religious intolerance and forced conversions.
x Emperor Akbar exempted Armenians from taxes on their trade with the Persian Gulf. The
Armenians settled in Surat (Gujarat) in the 16th century, and in Chinsurah (West Bengal) in the
Armenians
late 17th century.
in India
x In 1665, they were allowed to form a settlement in Saidabad, in Murshidabad district of Bengal,
after a royal farmaan was issued by Aurangzeb.
x Besides Murshidabad, Surat and Benares assumed robust identities as towns of silk crafts due to
Armenian trade.

x The rise of the Armenians in Bengal was due to their ability to channelize the trade conflicts and
monopolies between the European and regional powers.
x They also decided to Anglicise themselves to appease the dominant colonial power.
x In 1744, Joseph Emin fled with his family from Persia joined about 4,000 Armenians in Calcutta.
Emin wanted to train in the manners, language, arts and science of the English and found influential
patrons in Mrs. Montagu, Sir William Jones and the Dukes of Northumberland and Cumberland.
Prominent
x There were several pedlars and kingmakers in Clive’s Bengal. One of them was Khoja Wajid, who
Armenians
held business transactions with the French, English and the Dutch. Other noted Armenian
of Bengal
merchants were Avak di Aratoon and Khachik di Khojamal.
x Khoja Wajid was the prince of saltpetre trade and trade negotiations in Bengal.
x Khoja Petrus Aratoon, another leading Armenian merchant, maintained close links with Saidabad
and the Mughal durbar in Murshidabad.
x Later Wajid’s decline coincided with the rise of Petrus Aratoon, who had been strengthening his
English ties for over a decade.

FOCUS | July 2018 | RAU’S IAS 97


@civilminds
History, Heritage & Culture

x All the histories of Plassey usually recount Clive’s coalition with Jaffar, the Jagat Seths and Omichand.
x But another major force to reckon with in Bengal then were the Armenians. Without them, the
victory at Plassey would have been a mirage for Clive and the Company.
x Before and after Plassey, Armenian aid helped dramatically consolidate British trade and military
presence in Bengal. For e.g.
9 After the Black Hole of Calcutta, Petrus Aratoon provided provisions for the East India Company
garrison.
Armenians
9 Petrus Aratoon was employed by Clive as a secret agent during his negotiations with Jaffar for the
and
overthrow of Siraj ud-Daulah in the Plassey Conspiracy.
support to
British 9 Aratoon became a member of the East India Company’s Council in Madras, and turned into an
ambassador for the Armenians in Bengal.
9 Besides shaping the metropolis of Calcutta, the commercial and diplomatic forays of Indian
Armenians also went into rebuilding the colonial epicentre of London.
9 Armenian commercial support, for instance, helped build East India House at Leadenhall Street —
the headquarters for many years of the world’s first multinational company.
x So, there was very real impact of Armenian influence, trade, diplomacy and culture on the course of
events in India before and after Plassey.

RAJA RAMMOHAN Recently, the 246th birth anniversary of Raja Ram Mohan Roy, the
'Maker of Modern India’ was celebrated. He was the one of the
ROY
major pioneers of social reform movement. He fought against
# PERSONALITY # REFORMER orthodoxy, raised issues concerning women exploitation and
worked for their emancipation.
#MODERN INDIA
ABOUT RAJA RAMMOHAN ROY

He was born in 1772 in Murshidabad district of West Bengal. He shunned the orthodox Hindu rituals and idol worship
right from the childhood.

x Rammohan Roy represented a synthesis of the thoughts of East and West. He was a scholar
who knew over a dozen language including Sanskrit, Persian, Arabic, English, French, Latin, Greek
and Hebrew.
x As a young man, he had studied Sanskrit literature and Hindu philosophy at Varanasi and the
Quran and Persian and Arabic literature at Patna.
A great
x To study the Bible in the original, he learnt Greek and Hebrew.
scholar
x In 1809, he wrote in Persian his famous work ‘Gift to Monotheists’. In this book, he put forward
weighty arguments against belief in many gods and for the worship of single god. He held that all
the principal ancient texts of the Hindus preached monotheism or worship of one god.
x He published the Bengali translation of the Vedas.
x He also believed that the philosophy of Vedanta was based on this principle of reason.

Pragmatic x Rammohan Roy insisted on applying rationalism to Christianity too, particularly to the elements
approach of blind faith in it.
towards x In 1820, he published his ‘Precepts of Jesus’. In this book, he tried to separate the moral and
Christianity philosophic message of the New Testament, which he praised, from its miracle stories.

FOCUS | July 2018 | RAU’S IAS 98


@civilminds
History, Heritage & Culture

x He wanted the high moral message of Christ to be incorporated in Hinduism. This earned for
him the hostility of the missionaries.

All his life Rammohan Roy paid heavily for his daring religious outlook. The orthodox condemned him for criticising
idolatry and for his philosophical admiration of Christianity and Islam. They organised a social boycott against him in
which even his mother joined. He was branded a heretic and outcaste. All these motivated him more to fight.
In this regard, let us understand some of his initiatives.

SOCIAL INITIATIVES

x In 1828, he founded a new religious society, the Brahma Sabha, later known as the Brahmo Samaj,
whose purpose was to purify Hinduism and to preach monotheism or belief in one god.
Brahma
x The new society was to be based on the twin pillars of reason and the Vedas and Upanishadas.
Sabha
x It was also to incorporate the teachings emphasis on human dignity, opposed idolatry and
criticized such social evils as the practice of sati.

x He was one of the earliest propagators of modern education.


Modern x In 1825, he established a Vedanta College in which courses both in Indian learning and in Western
education social and physical sciences were offered.
x He compiled a Bengali grammar.

x Rammohan Roy was champion of women’s rights. He condemned the subjugation of women
were inferior to men in intellect or in a moral sense.
x He organised groups of like-minded people to keep a strict check on sati performances and to
prevent any attempt to force the widows to become sati.
Pioneer of
x When the orthodox Hindus petitioned to parliament to withhold its approval of Bentinck’s action of
women’s
banning the rite of sati, he organized a counter-petition of enlightened Hindus in favour of
rights
Bentinck’s action.
x He attacked polygamy and the degraded state to which widows were often reduced.
x To raise the status of women, he demanded that they be given the right of inheritance and
property.

POLITICAL INITIATIVES

x He was also the initiator of public agitation on political questions in the country.
x He condemned the oppressive practices of the Bengal zamindars which had reduced the
peasants to a miserable condition.
x He also protested against the attempts to impose taxes on tax-free lands.
Major actions
x He demanded the abolition of the Company’s trading rights and the removal of heavy
export duties on Indian goods.
x He also raised the demands for the Indianisation of the superior services, separation of the
executive and the judiciary, trial by jury and judicial equality between Indians and Europeans.

International
He was a firm believer in internationalism and in free cooperation between nations.
outlook

FOCUS | July 2018 | RAU’S IAS 99


@civilminds
History, Heritage & Culture

BLAST FROM THE PAST

JAMI MOSQUE, GULBURGA


# MEDIEVAL INDIA #CULTURE
(Courtesy: The Hindu)

Mohammad bin Tughlaq, who ruled from 1321 to 1351 as the Delhi Sultan, captured large parts of the Deccan
including Gulbarga. In 1347, a Tughlaq officer named Alauddin Hasan revolted against Tughlaq and declared his
independence by establishing the Bahmani kingdom (1347-1527) with Gulbarga as its capital. Hasan built Gulbarga
as a fortress city. The Bahmani Sultans ruled from here till the capital of the kingdom was shifted to Bidar in 1424.

When Mohammad bin Tughlaq shifted his capital to Daulatabad in the Deccan, he took artisans and architects with
him. However, the Bahmani Sultans chose to look towards Persia as a source of inspiration, much like Europe looked
towards Greek architecture, impressed with its soaring arches and a lightness of touch. Sometimes this was combined
with existing and developing local styles and at other times the architecture of the Tughlaq style remained, as seen in
the tombs of the Bahmani kings in Gulbarga.

Inside the mosque

Approaching Gulbarga, I could see the irregular fortress walls stretching into the distance for nearly 3 km. They are 50
ft-thick double walls and are surrounded by a moat carved out of living rock. The moat is 30 yards wide in places. I
could see some solid semicircular bastions as we entered through one of the two entrance gateways. I was a little
disappointed when I saw the crumbling wall and gateway, with cattle happily grazing there. But the first sight of the
grand Jami Masjid, or congregational mosque, built inside was enough to set my heart soaring.

The Jami Masjid was completed in 1367. Even now, it stands as a firm testimony to the glory of a bygone kingdom.
Though I have not visited Cordoba in Spain yet, as soon as I entered the Jami Masjid I could imagine why there are
comparisons with the Great Cathedral-Mosque of Cordoba.

The uniqueness of the mosque is that it has no open courtyard and the entire structure is covered by a roof. According
to art historian Percy Brown, “Some of the originality of its design and construction may be due to the [fact] that it was
produced under the direction of a hereditary architect named Rafi, not of India, but from the distant town of Kazvin
in northern Persia. It is possible that this talented descendant of a noted family of architects evolved the scheme of this
mosque from his inner consciousness, that its unusual conformation was the result of his own genius.”

He also says: “He may have looked to the occident for his inspiration, and that at the back of his mind was some idea of
a domed and vaulted hall of the basilica type, an occasional form of Moslem religious edifice in some of the countries
of eastern Europe.”

The mosque is divided into wide arched sections with a spacious area on the west with the Mihrab. This space is
covered by a high central dome. Rows of aisles with 68 bays, each shaped like a cupola, give the pillared hall a
magnificent appearance. There are honeycomb pendentives on the four sides of each cupola.

These rows of wide-spanned single arches with low imposts gave me goosebumps with their perfection and
geometrical symmetry. After I prayed in front of the western Mihrab, I couldn’t resist getting myself photographed
under every arched bay.

FOCUS | July 2018 | RAU’S IAS 100


@civilminds
History, Heritage & Culture

Plain from the outside

In my excitement, I had gone straight into the mosque, without observing it properly from outside. When I had finished
touring the inside, I stepped out. The mosque looked solid and plain, much in contrast to the the one-aisled mosques
of Delhi with their open courtyards and decorated entrances that I was accustomed to. Even the lofty arch on the
northern entrance was simple without any calligraphy.

As the area in front of the mosque was crowded, I got a better view from a nearby bastion on top of which lies one of
the biggest cannons in the world. From this elevation I could see the central dome, the four smaller side domes, and
the arched openings running on three sides of the mosque.

This mosque influenced many other mosques of the Deccan built subsequently. Brown says: “The clerestory
supporting the dome became a feature of the building art in these parts, while the wide span and low imposts of the
cloister arches figured as the keynote to many of the later monuments.” These included the Kali and Khirkee mosques
in Delhi.

FOCUS | July 2018 | RAU’S IAS 101


@civilminds

SECURITY
# GS Paper III (Main)

The Financial Action Task Force (FATF) is an inter-


FINANCIAL ACTION TASK governmental body established in 1989 by the Group of
FORCE (FATF) Seven (G-7) countries in Paris, France. It was formed
with the intention to examine and develop measures to
#MONEY LAUNDERING #EXTERNAL SECURITY combat money laundering.

The objectives of the FATF are to set standards and promote effective implementation of legal, regulatory and
operational measures for combating: Money laundering; Terrorist financing; and threats to the integrity of the
international financial system.
Recently, FATF has placed Pakistan in its Grey List. Moreover, it has made recommendations for India’s Prevention of
Money laundering Act, 2002. Let us understand both the actions of FATF.

MORE TO UNDERSTAND

x Initially, FATF issued report containing a set of (Forty Recommendations), which intended to
provide a comprehensive plan of action needed to fight against money laundering.
x In 2001, the development of standards in the fight against terrorist financing were added to
FATF ‘40+9’
the mission of the FATF thereby further adding 9 Special Recommendations.
recommendations
x Current Status: FATF has formed 40 recommendations against money laundering and 9
special recommendations against terrorist financing, which forms the commonly known
‘40+9’ FATF Standards.

x The FATF conducts peer reviews of each member to assess levels of implementation of the
Mutual FATF Recommendations.
Evaluations x It provides an in-depth description and analysis of each country’s system for preventing
criminal abuse of the financial system.

x FATF issues a list of ‘Non-Cooperative Countries or Territories’ (NCCTs), commonly called


the (FATF Blacklist). These countries or territories are considered to be uncooperative in
international efforts against money laundering and terrorism financing.
FATF Listings
x The (FATF Greylist) is a list of countries or territories with deficiencies in anti-money
laundering and/or countering the financing of terrorism, for which they have developed an
action plan with the FATF.

FATF AND PAKISTAN

x Pakistan was put on the grey list in 2012 after the completion of an earlier Mutual Evaluation by
FATF, and therefore had to follow the action plan suggested by FATF.
Background
x In 2015, it was taken off the grey list, after the FATF was satisfied with Pakistan’s measures
undertaken to counter terror financing.

x The US, along with the UK, France and Germany, started a process at the FATF to co-sponsor a
Current
motion to nominate Pakistan as a country having strategic deficiencies in countering financing of
Scenario
terrorism.

FOCUS | July 2018 | RAU’S IAS 102


@civilminds
Security

x This motion was passed in March, 2018.


x FATF in June, 2018 placed Pakistan on the ‘grey list’ for failing to curb anti-terror financing despite
after submitting a 26-point action plan to FATF.
x The other countries on the list are Ethiopia, Serbia, Sri Lanka, Syria, Trinidad and Tobago, Tunisia
and Yemen.

Indian India has asserted that Pakistan should comply with FATF action plan and take credible measures to
Response address global concerns related to terrorism emanating from any territory under its control.

x FATF has laid out a 10-point action plan for Pakistan for compliance with its guidelines.
Future
x Pakistan’s failure in implementing the Action Plan may result in it being included in the black list the
Scenario
next year.

x FATF uses peer pressure through the age-old technique of name-and-shame which intends to
affect is the sentiment of foreign investors and foreign governments.
x Pakistan was on the FATF grey-list from 2012 to 2015, but the economic impact of the grey-listing
Grey List was minimal.
Impact x Within this period, Pakistan was able to successfully complete an IMF program and rose over $5
billion from the international bond markets. Its imports and exports remained stable and the grey-
listing did not raise any significant barriers to trade.
x But it can definitely malign a country and may impede investments and business flow.

FATF RECOMMENDATION FOR INDIA

x FATF has asserted that Prevention of Money Laundering Act, 2002 is a predicate-offence oriented law. A
predicate offence is a crime that is a component of a more serious criminal offence.
x It means that producing unlawful funds is the main offence and money laundering is a predicate offence, a
component of a more serious offence.
x India needs to make money laundering a (standalone offence and not a predicate offence) to comply FATF
recommendations under 2020 Mutual Evaluations report.

Key focus points (Prelims): FATF; 40+9; Blacklist; Greylist

ONLINE MONITORING The Ministry of Home Affairs has launched an Online


OF FUNDS UNDER FCRA Analytical Tool (OAT) to monitor foreign contributions
under FCRA. Let us further understand about this.
#Internal Security

x The Online Analytical Tool intends to facilitate closer monitoring of the flow and utilisation of
foreign contributions received by various organisations registered or permitted under the
Foreign Contribution (Regulation) Act, 2010.

Objective x It would enable decision-makers in various Departments of Government to:


9 Scrutinise source of foreign funds;
9 Scrutinise actual use of foreign funds in India;
9 Increase capacity to take data-driven and evidence-based decisions on FCRA, 2010.

Implementation x The OAT will have analytical features to conduct big data mining and data exploration.

FOCUS | July 2018 | RAU’S IAS 103


@civilminds
Security

x It will be integrated with the bank accounts of the FCRA-registered entities through the Public
Financial Management System.
x This would provide for updating of transactional data on a real-time basis.

x There are approximately 25,000 active organisations registered under The FCRA, 2010. Foreign
Contributions worth above Rs.18,000 crores were received by such organisations from foreign
donors for various social, cultural, economic, educational and religious activities during financial
Requirement year 2016-17.
x There have been concerns of foreign governments and non-State actors funnelling funds into
India for anti-India activities such as funding protests against nuclear developmental projects,
etc.

Key focus points (Prelims): OAT; FCRA

India participated in the 22nd edition of Exercise


EXERCISE MALABAR - 2018 Malabar at Guam, USA. Let us understand about this
# Military Exercise military exercise.

x Guam lies in Micronesia in the western Pacific Ocean.


x It is the westernmost point (in terms of jurisdiction) and territory of
the United States.
x It is a vital US military outpost and host to strategic bombers and
service members.
x Guam’s importance to America’s Pacific defence capabilities, and its
location as the closest point of American soil to North Korea, just
2,200 miles southeast, have left it vulnerable.
x Last year, it was threatened by North Korea with possible missile
attacks.

Background x Exercise Malabar started as a bilateral exercise between the US Navy and the Indian Navy in 1992.

FOCUS | July 2018 | RAU’S IAS 104


@civilminds
Security

x The Japanese Maritime Self-Defense Force (JMSDF) began to participate from 2007 ,thereby
making Exercise Malabar a trilateral military exercise.

x It is for the first time that the exercise is being conducted in Guam which is a major US Naval
Base in the Western Pacific.
Significance
x Moreover, the US has also recently named their Hawaii-based Pacific Command as the U.S. Indo-
Pacific Command, thereby increasing the relevance of India and Indian Ocean in its purview.

x Australia has shown keenness in joining Exercise Malabar, which is supported by US and Japan.
Australian
x However, India has been reluctant for the membership of Australia as India does not intend to
Membership
showcase Exercise Malabar which primarily focuses on Indo-Pacific region as being against China.

Key focus points (Prelims): Exercise Malabar; Guam

VIEWPOINT

POKHRAN II: TWENTY YEARS LATER


# Nuclear Test # Energy #Security
(Courtesy: The Hindu)

Read and ponder!


Twenty years ago in 1998, India took a leap into the unknown world of nuclear weapon powers with the tests at
Pokhran. Though the decision was taken after great deliberation and with preparation, how the reaction of the world
would affect the future of India was unknown. But today, it is certain that the action was timely and inevitable.

Obstacles removed
India has reason to be satisfied over having accomplished many of the objectives of Pokhran II. Indian diplomacy
triumphed in turning a grave crisis into an opportunity by securing legitimacy for its nuclear arsenal and removing
obstacles in generating nuclear power. But the hasty enactment of a liability law, which inhibited nuclear trade, and
the setback globally to nuclear power on account of the Fukushima disaster stood in the way of India benefitting fully
from Pokhran II and the subsequent agreements reached.
The fact, however, remains that the 1998 tests and the subsequent nuclear deal have brought India to the nuclear
mainstream and opened up the global nuclear market for development of nuclear power without signing the Treaty on
the Non-Proliferation of Nuclear Weapons (NPT) or the Comprehensive Nuclear-Test Ban Treaty (CTBT).
The tests shocked the world, particularly because they were done with utmost secrecy and the India-U.S. ties hit rock
bottom. For nearly two months, the U.S. refused to have any dialogue with India and implemented the Glenn
Amendment for the first time. Newer sanctions were imposed, and at one point it looked that relations would never
recover.
The talks between U.S. Deputy Secretary of State Strobe Talbott and Foreign Minister Jaswant Singh over the next two
years were the most comprehensive dialogues India had with the U.S. on its nuclear policy, including the threat
perception and future plans for security. India was anxious to have the sanctions lifted, but Mr. Singh sought to delink
sanctions from the security dialogue, not to be pressurised to take quick decisions. Mr. Talbott began by insisting that
the objective was to get India to sign the NPT. Then he listed five benchmarks as non-proliferation goals to

FOCUS | July 2018 | RAU’S IAS 105


@civilminds
Security

normalise relations: 1. signing the CTBT, 2. halting production of fissile material, 3. strategic restraint, 4.
strengthening export control regimes, and 5. normalisation of relations with Pakistan. These were strongly
rejected by India, but the talks proceeded on the assumption that India’s security concerns should be fully understood
and that India would take certain measures to suit its new status. But, in effect, India met the U.S. demands more than
half way, leading to an understanding, which led to President Bill Clinton’s visit to India and Prime Minister Atal Bihari
Vajpayee’s visit to the U.S. in 2000.
India refused to sign the CTBT, but declared a moratorium on testing; agreed to join the Fissile Material Cut-off Treaty
negotiations without halting fissile material production; reaffirmed minimum deterrent without giving any number of
warheads; and agreed to strengthen export controls. Additionally, India declared no-first-use and commitment to
disarmament. Though no deal could be struck, the foundation was laid for what became the nuclear deal in 2008.
Though India placed its civilian nuclear facilities under perpetual safeguards, its nuclear assets remained fully insulated
against external scrutiny and interference. India secured rights to receive uninterrupted nuclear fuel supplies as a
trade-off against safeguards. It kept open its right to acquire advanced enrichment and reprocessing technologies,
although it would require bilateral negotiations with the U.S. and others. India’s sovereign right to test a nuclear device
in the future has remained intact, although the deal would be in jeopardy in such an eventuality. Presidents George W.
Bush and Prime Minister Manmohan Singh remained committed to the deal throughout the negotiations and made
decisive interventions at crucial moments. Apart from the specific gains in the nuclear area, the new India-U.S.
partnership, which promised investment and high technology, was a turning point in Indian foreign policy. On the
negative side, the deal generated mistrust in Russia and China, which had to be dealt with in future years.

Reality check
Ten years after signing the deal, its gains and losses have proved much less game-changing than it was hoped in 2008.
Though not a champion of the deal as a Senator, President Barack Obama committed himself to the implementation of
the deal as part of his strategy to build good relations with India. But his personal affinity to the NPT and non-
proliferation made him reluctant to interpret the 123 Agreement liberally. The expectation was that the prospect of
nuclear trade with India would be a great attraction, but in 2009 Mr. Obama gave clear indication to his advisers that
he would not sacrifice his non-proliferation agenda for commercial reasons. Much has happened since then, but the
fact remains that there has been no nuclear trade till today. India’s nuclear liability law, forced on the government by
critics of the deal, became a smokescreen for the U.S. to not supply nuclear material to India. The repeated
declarations about a way out of the liability law and plans to set up American reactors in India after Narendra Modi
became Prime Minister have not changed the lack of enthusiasm in the U.S. on nuclear trade with India. In any case,
the situation has become more volatile after Donald Trump became U.S. President.
Another major event that has shaken confidence in the value of nuclear power in India’s energy mix was the
Fukushima disaster. It has changed the global nuclear power scenario beyond recognition, though India has
maintained that it is “business as usual”. The government’s recent decision to build more indigenous reactors points
to the fact that the dream of imported nuclear reactors dotting India has disappeared. India’s focus has rightly
shifted to solar and other new sources of energy.

FOCUS | July 2018 | RAU’S IAS 106


@civilminds

Miscellaneous
SPORTS ROUND-UP
In one of the most gruelling tests of human endurance, this annual cycle
TOUR DE race of more than 3000 km, crisscrossing mountains, valleys, forests,
FRANCE 2018 villages, towns and cities of France, stands out. Every year, it is conducted in
the month of July. With the other two cycling races--Giro d'Italia and Vuelta
# Cycling a España, they are termed as European Grand Tour.

TOUR DE FRANCE

Starting year 1903

x It is a multiple stage bicycle race,


primarily held in France and
occasionally make passes
through near-by countries like
Location
Germany, Spain etc.
x Last year, it started in Germany
and this year it will make a brief
dip into Spain.

x 105th edition
2018 race x Starts 7th July and ends on 29th
July.

Starting point Noirmoutier, Western France

Ending point Champs-Élysées, Paris

Defending Chris Froome (4 wins), Team Sky, Nationality: British.


champion He will race once again, if he’s successful he’ll claim his fifth victory at the French race.

For leadership positions, there are:


9 General Classification (Leader wears Yellow Jersey)
9 Points Classification (Leader wears Green Jersey)
Classifications
9 Mountain Classification (Leader wears White Jersey with Red dots)
9 Young Rider Classification (Leader wears White Jersey)
9 Team Classification.

Note: Stay connected for the outcome in the forthcoming FOCUS magazine.

FOCUS | July 2018 | RAU’S IAS 107


@civilminds
Miscellaneous

In July, the green grass courts of Wimbledon are beckoning again to relish
WIMBLEDON cool climes of England along with display of high grade tennis by veterans
and some new challengers. All eyes will be on Roger Federer who has shown
2018 immense rejuvenation since last year, and grass court is his favourite. But
# Tennis #Global there is always a challenger in Rafael Nadal and Novak Djokovic.
In women’s category, Serena Williams after her maternity break is again
Tournament
going to impact the chain of events here.

1877, is the oldest tennis tournament in the world, and is


Founded
widely regarded as the most prestigious.

Surface Grass outdoors

Host City London

Venue The All England Lawn Tennis and Croquet Club

It is the third Slam in a calendar year comprising four slams-


Calendar
Australian Open, French Open, Wimbledon and U.S. Open.

Most titles (Men) Roger Federer (8) of Switzerland

Most Titles (Women) Martina Navaratilova (9) of Czech Republic

Note: Stay connected for the winners in the forthcoming FOCUS magazine.

CHAMPIONS Australia won 2018 edition of the Champions Trophy held in


Netherlands by defeating India in the final 3-1 via shoot-off as
TROPHY 2018 both teams were locked at 1-1 in regulation time. It was also the
last edition of the trophy.
# Hockey #Global Tournament

FOCUS | July 2018 | RAU’S IAS 108


@civilminds
Miscellaneous

CHAMPIONS TROPHY

Founded x 1978
x It is an international field hockey tournament held by the International Hockey Federation (IHF).
x It features the world's top-ranked field hockey teams competing in a round robin format. A biennial
women's tournament was added in 1987.

Biennial It changed from an annual to a biennial event from 2014 onwards, due to the introduction of the
Hockey World League (HWL).

2018 edition x It was the last edition of the Champions Trophy and the tournament will be replaced by the
Hockey Pro League (HPL) in 2019.
x The Hockey Pro League is an international field hockey competition organised by the International
Hockey Federation. It was started in 2017.
x The competition, also serves as a qualifier for the Hockey World Cup (men and women's) and the
Olympic Games.

Most titles in Champions Trophy Australia, 15 titles

Indian performance Runner-up in both 2016 and 2018, and on both occasions losing to Australia.

Î RELATED INFORMATION: INTERNATIONAL HOCKEY FEDERATION (IHF)

x It is commonly known by the acronym FIH.


x It is the international governing body of field hockey and indoor field hockey.
x Founded in 1924.
x It is headquartered in Lausanne, Switzerland.
x Current President: Narinder Batra

FIFA WORLD CUP 2018


# Football #Global Tournament
The 2018 edition of the world cup is reaching its climax and has entered the quarter final stage in the first
week of July, 2018. It started with 32 teams, and now 8 remaining in the fray. These include France, Brazil,
England, Russia, Sweden, Croatia, Uruguay and Belgium.
The defending champion of 2014, Germany was knocked out in the group stage, and 2010 champion Spain
lost in the Pre-Quaterfinals.
th
More mystery and climax is hidden in the world cup and the summit clash is on 15 July, 2018.

Note: Stay connected for the further action in the forthcoming FOCUS magazine.

DAUGHTER’S OF THE SUN


# BOOKS AND AUTHORS

FOCUS | July 2018 | RAU’S IAS 109


@civilminds
Miscellaneous

Genre: Non-Fiction, History

Author: Ira Mukhoty

x This represents a parallel history of the Mughal empire and its

women. Many of these were accomplished, educated, articulate

and cultured.

x They were writers, poets, builders, traders, milk-mothers,

providers of sage counsel to brothers and sons, caregivers and

protectors of infant princes, companions in strange lands

accompanying warrior husbands, repositories of dreams and

dynasties, accomplished and better educated than their

contemporaries anywhere in the world.

x They were visible symbols of power and opulence and sometimes

the unseen but politically astute power behind the throne —

Mughal women were all this and more.

Flow of the book: The book:


x Begins from 1500, when a 17-year-old Babur marries for the first time, till the lamp of Mughal glory is extinguished in
1857, so this is a first-of-its-kind parallel history.
x It mentions about accomplished women who could ride into battle with their husbands (like Hamida Begum), or
engage in diplomacy (like Khanzada Begum), write family histories (like Gulbadan Begum), or pen Sufi treatises (like
Jahanara)..
x The most powerful and flamboyant was Noorjahan who has coins issued in her name and is allowed to issue
farmans.
x It is an apt take on role of women in the one of most distinguished empires in the world, the Great Mughals.

FOCUS | July 2018 | RAU’S IAS 110


@civilminds

ETHICS, INTEGRITY & APTITUDE


# GS Paper IV (Main)

ARE WE GETTING THE CIVIL SERVICES THAT INDIA WANTS?


#Ethics #Integrity #Civil Service Reform

Q. Initially Civil Services in India were designed to achieve the goals of neutrality and effectiveness, which
seems to be lacking in the present context. Do you agree with the view that drastic reforms are required in
Civil Services. Comment. (UPSC 2017)

The question above seems melancholic because there is huge contrast between [what is expected] and [what
are we witnessing] in the manner civil servants work.

Do we have a solution?
From where this solution starts?

Let us start by reading, and pondering on the text given below.


----------------------------------------------------------------------------------

SO, EXACTLY........WHAT IS EXPECTED?

A civil servant should act like a (Wisdom Tree)


working relentlessly for promoting resilience,
and wellbeing throughout.

&
This expectation emerge from seven cardinal
principles desired from a civil servant.
These are--selflessness, integrity, accountability,
openness, leadership, objective and honesty.

Cardinal Principle Meaning

Selflessness Selflessness refers to the feeling of serving others without thinking of oneself.

Integrity Integrity refers to the absence of any outside influence from the organization in decision
making.

Accountability Accountability refers to answerability of the civil servant to the public.

FOCUS | July 2018 | RAU’S IAS 111


@civilminds
Ethics, Integrity & Aptitude

Openness Openness refers to transparency in decision making and facilitating flow of information.

Leadership Leadership infers that a civil servant should lead by example by following all the principles.

Objectivity Objectivity refers to the principle of merit where selections would not be based on other
considerations.

Honesty Honesty refers to the act of civil servant in such a way so as to avoid conflict of interest.

In addition to these fundamental principles an ideal civil servant should have empathy i.e. compassion for the
people. He/She should be politically neutral and should have courage to withstand undue pressures. In addition,
he/she should have temperance in order to be patient enough to take decisions. He/she should use public funds
prudently and is the best of the interest of public.

Apart from such virtues, a civil servant must possess intellectual capability to take decisions. He/She acts as an
interface between the people and government; needs to follow the decisions taken at the top and respect
hierarchy; needs to work within the four walls of the law; and if conditions arise that there might be violation of law
in carrying out any instructions from top then it should be clearly brought out to the notice of seniors.

If still insisted on to carry out such instructions, then it should be carried out only through written orders and not
through mere oral orders. It should be mentioned that illegal orders which hinder one’s authority to exercise powers or
is against any statute should be ignored. A civil servant only needs to act according to the law even if he/she finds it
unfair or unjust. The best he/she can do is to put forth his/her reasons about its drawbacks.

In addition, a civil servant should never criticize government policies in public discussions. He/She is a part of the
government and such act reduces the credibility of the government as well as his /her own merit. A civil servant should
never hesitate to put forth his/her views even if it goes against the advice of ministers, and the views should be
coupled with logical reasons and lucid presentations.

Similarly, a civil servant should be open to any new suggestions, open to accept new challenges at work, try to
develop innovative approaches to implement orders, should not hesitate to take tough decisions and should try to
make a note in writing of all his/her actions in order to be transparent.

One of the toughest jobs for a civil servant is to maintain balance in his/her personal and professional life. There
are numerous challenges to a civil servant when there is pressure of family members and close friends. He/She should
be able to maintain such space so as not to allow any influence which hinders improvement in governance and
transparency. He/She should try to include civil society in decision making so as to make governance more
participatory. A civil servant needs to take along with himself/herself private as well public institutions. He/She should
maintain transparency in dealing with private sector and should avoid favouritism. A civil servant should not
blindly follow any law but implement it considering all dimensions. The nature of today’s administration is more people
centric approach. He/she should be able to judge repercussions of allowing more competition or giving space to
private entities on poor sections. This should be clearly brought out at policy discussions. This needs more of
assessing ground level realities which requires open public discussions and bottom up governance.

The globalised world of today makes the task of civil servant more difficult. The increasing role of private sector,
decreasing social sector public investment, high competitiveness, increasing marginalization of lower sections of
society, increased awareness of the citizenry, requirement of flexibility and broad knowledge network has made it
necessary for a civil servant to come out from a narrow outlook and update himself/herself according to changing
needs. He/She needs to be more communicative by developing good inter-personal skills in dealing with
multinational companies. He/She should be well versed with the power of information technology and make its
sufficient use to broaden his/her knowledge and improve service delivery.

The principle of utilitarianism suggests benefit of majority is superior to minority loss. However, this approach
violates the principle of equal justice where each member of the society should be able to enjoy his/her rights. In such

FOCUS | July 2018 | RAU’S IAS 112


@civilminds
Ethics, Integrity & Aptitude

cases, rights based approach that follows non-violation of anyone’s universal rights needs to be followed. However,
such ethical dilemma needs to be sorted out at the individual level according to one’s discretion. Clearly such decisions
need critical analysis and in-depth knowledge in order to be able to take effective decisions which a civil servant should
possess.

The high work pressure and expectations often stresses a civil servant. In such cases, careful handling of situations is
required. A civil servant should be equipped with the ability to maintain his/her cool and calm façade and follow
laid out rules and regulations for maximum effect. He/she should try to include all stakeholders in decision
making and should stand up against illegal directions. It may hamper one’s career growth but in the longer run a civil
servant is respected for his/her uprightness and strong commitment. His/Her attitude must be reflected in the
behaviour which requires strong conscience. For this, he/she needs to have clear objectives, feedbacks from
stakeholders, patient listening and articulation of right thoughts. He/she needs to ensure responsibility for
his/her work and should be ready to accept accountability in case of failures.

Thus an efficient and upright civil servant ensures public service delivery to the best of his/her ability. It leads to a shift
from responsive to proactive administration.

However, inspite of such importance, civil service today is not what an aspiring nation like India would want. It is
marred with inefficiency, corruption, lack of accountability and transparency, apathy towards public. India has
continuously been rated lower in international indexes such as corruption index, world competitiveness index, social
index, education and health indexes, and ease of doing business index etc. However, it is not that government is not
acquainted about such issues as it has taken numerous steps to make civil servants more accountable, and responsive
initiatives such as e-governance, social audits, right to information and mandating citizen charter have also been
instituted to make it more impactful.

Hence only a change in mindsets, where a civil servant should not think himself/herself as a ruler but a provider of
government service, and where public acts as a customer of these services is the key. A shift to the approach where
‘Customer is king’ is echoed in public administration, is required. This needs proper sensitization of civil servants and
effective training and skill enhancement initiatives. Every civil servant should remember Gandhiji’s Talisman before
taking decision:-

“Whenever you are in doubt to take decisions follow this approach. Recall the face of the poorest man you have ever seen and
ask yourself, if the step you contemplate is going to be of any use to him/her. Will he/she gain anything?”

But the question remains....Is this possible?

Are we disillusioned?

What creates this disillusion?

SO, EXACTLY........WHAT ARE WE WITNESSING?

Let us hear from the horse’s mouth. Given below is the right-up shared by Dipali Rastogi. She is a serving IAS officer in
the Madhya Pradesh Cadre.

(Courtesy: Open Page, The Hindu)

The philosophy of power and prestige


The way the steel frame functions and thinks: a critical view from deep within it, from one among the fraternity.

A recent advertisement inviting applications for lateral entry into the bureaucracy at the level of Joint Secretary caused
quite a ripple amongst the Indian Administrative Service fraternity. Social media groups on Facebook, WhatsApp and so
on were awash with opinions and counter-opinions. Some were critical, very few were pro, and some were angry.

But mostly, guarded. Because that’s what most IAS officers are — guarded.

FOCUS | July 2018 | RAU’S IAS 113


@civilminds
Ethics, Integrity & Aptitude

They weigh their words very carefully before they speak. Most choose to not speak at all — on critical matters. On
anything else, they can hold forth forever, to those below them. To those above, well, they are the silent yea-sayers.

And that is what is possibly causing their temporary setback, too.

Irony....

The biggest “quality” attributed to an IAS officer is that he or she will anticipate political will and run to obey/ comply
before words are even out of his political masters’ mouth. The best IAS officer is one that has no opinion of his own, or
even if he does, keeps it to himself. As the years go by and the lines between bureaucracy and politics, between right
and wrong, between truth and lies become fainter and fainter, an IAS officer will have lost all original thought. In
matters of governance. And since by then he has almost stopped listening to anybody except his pompous self, his
opinion or knowledge becomes all but redundant — it is neither real nor current. His figures are all fudged to show
what the ones in power want to see and project. His opinions are only a confused muddle of what “appears correct” or
“conveys the right message.”

Far from reality. But he doesn’t care. He’s got his salary, perks, club membership, car, driver, domestic help, spruced-up
home, dinner parties, discounted vacations, foreign trips, and the silly, mean-minded “power” to dole out small favours
to friends and relatives.

And now probably the politician, always answerable to the people, has a nagging suspicion of being taken for a ride.
Fed with the belief that the real experts are those that come from years of working in the “field” — a euphemism used
by IAS officers as an indication of their superior experience over their counterparts in other central services — he may
have realised he is not getting his money’s worth. So why not get in some real experts who actually know something
and are not scared to voice it.

The IAS officer feels cheated. He who let his spine to rot and said only what the masters wanted to hear, being moved
out because the masters no longer want a rebound of their thoughts but something else? What a sheer waste! All
those years of realisation that his “expertise” was a prerequisite only till he got in — later it was to hang at the window
till it ran dry. Nobody asked for it, nobody wanted it. What was required to nab those coveted postings was a whole
different set of skills — and he spent valuable years cultivating them.

Pun....will bounce back with vengeance

What now? Is he irrelevant? Well, one thing the IAS has cultivated in good measure is resilience. He will wait quietly in
the wings. He knows that truths are much more inconvenient than post-truths. He will wait till the masters get
uncomfortable with the opinions and the barefacedness of the experts. They’ll look around for someone comfortable
to work with. Someone who can tell them the sun shines brightly and all’s right with the world. They’ll wait for the
masters’ discomfort to set in.

And then they’ll move in again quietly. I told you so, they’ll say. And this time they’ll fortify their citadel better.

Never is it easy to dislodge the steel frame in half measure.

-----------------------------

FOCUS | July 2018 | RAU’S IAS 114


@civilminds
Ethics, Integrity & Aptitude

IAS: The belief system, and some truths


These are the assumptions an IAS officer lives with, and the reality there:

™ We are in this service to serve. The truth is, we scarcely behave as servants.

™ We handle vast sums of money and human resources; we do not possess any expertise for this task. We are not
trained accordingly.

™ We have a very high opinion of ourselves and our “intelligence” and “experience”, and think people respect us for
what we are. In reality, people genuflect before us due to the power we wield to either do benefit or damage.

™ Over the years we have developed the tendency to distribute largesse, whether in kind or in ideas. In reality, we do
not own what we distribute.

™ We are paid to manage things efficiently and create systems. In actuality, we thrive on mismanagement and chaos
because that gives us the power to choose some over others.

™ We are supposedly the steel frame. In reality, we have no long-term vision. We take ad hoc decisions, looking to
what the authority above us wants.

™ We exploit the system for preferential treatment — for ourselves and people known to us. We are hypocritical
enough to say we do it to “help” people.

™ We know if we create systems where everyone has easy access to services, we shall become superfluous. So we let
things be.

™ We love to expand and enhance our sphere of work. We do not bother to place systems to bring in the needed
efficiency.

™ Worst of all, we are the most pompous, officious and ill-bred set of people. And we have the nerve to say we work
for the people of this country.

™ In reality we have no stakes in this country — our children often study abroad and we have created a niche cocoon
of the luxuries this system can give us.

™ We have no empathy with the larger populace, though we are always careful to make the right noises.

™ If there were any justice, we would have long been extinct. But we are too powerful to let ourselves be annihilated.

™ We are the IAS.

Q. What is the importance of a neutral, effective and well managed Civil Services to India?
A well-managed Civil Services strikes to achieve an egalitarian and just society. It removes inequality and
marginalization and makes economic growth inclusive. India, with high demographic dividend needs to take up the
challenge to utilize this dividend. This needs proper imparting of skills, medical and education infrastructure and right
legislations. Clearly, the role of civil servant is preeminent in the facilitation of such national objectives as it would help
in elimination of social and economic injustice, improve standards of living and make India economically developed. To
achieve these, the key attributes that are of utmost importance are:

Key attributes of a Civil Servant

Professional face Should have empathy for the people

Should act as an interface between the people and government

Should be politically neutral

FOCUS | July 2018 | RAU’S IAS 115


@civilminds
Ethics, Integrity & Aptitude

Upright and Should never hesitate to put forth his/her views


proactive
Views should be coupled with logical reasons and lucid presentations

Should more communicative by developing good inter-personal skills

Open to any new suggestions

Open to accept new challenges at work

Approach Develop innovative approaches to implement orders

Should have patience to take non-judgemental decisions.

Include all stakeholders in decision making

Promotes and support team work

Well versed with the power of information technology

Promoting Should use public funds prudently


transparency and
Transparency in dealing with private sector and should avoid favouritism
accountability
Upholds Rule of Law

Balanced life Maintain balance in his/her personal and professional life.

ASSIGNMENT FOR SELF-PRACTICE

Q. “Traditional bureaucratic structure and culture have hampered the process of socio-economic
development in India.” Comment.

Q. Suggest drastic reforms that are required to install neutral, effective and well managed Civil Services in
India?

Q. Do you possess the key attributes of a civil servant? If not, then what are the steps that you plan to
undertake to develop those attributes.

GET STARTED !

----------------------------------------------------------------------------------

FOCUS | July 2018 | RAU’S IAS 116


@civilminds

Glossary
# Terms

Enrich Your Concepts

x This refers to a seismic law.


x It states that there is a constant empirical relationship between the frequency of
earthquakes in a region and their magnitudes.
Gutenberg- x That is for every 1,000 earthquakes of magnitude 4, for instance, there are 100
Richter law earthquakes of magnitude 5, 10 earthquakes of magnitude 6 and so on.
x The law has been used to argue that seemingly unpredictable events actually follow a
(# Geology)
simple pattern.
x It was formulated by American seismologists Charles Francis Richter and Beno
Gutenberg in their 1956 paper, “Magnitude and energy of earthquakes”.

x This refers to the changes in the physical characteristics of an organism caused by


deliberate human intervention in its genetic makeup.
x Volitional evolution is made possible by recent advances in the science of
Volitional biotechnology which allows human beings to engineer the genetic composition of
evolution various organisms according to their own desires.
x This is in contrast to the predominant form of evolution throughout history, facilitated
(#Human Biology)
by the process of natural selection.
x Infusion of Genetically modified technology is increasing and this may also pose ethical
problems by the deliberate manipulation of genes by human beings.

x This refers to the proposition that a person who takes up a task without being
completely aware of its level of difficulty may still decide to complete it because it is
too late to abandon it.
Hiding Hand x Being unaware of the obstacles that lie in the way of completing a task can thus cause

principle people to complete difficult tasks that they would not otherwise.
x It has been criticised by behavioural economists who contend that being unaware of
(#Economy)
the actual costs of a project can cause people to abandon ongoing projects.
x To avoid this most of the projects are carried forward on the basis of Detailed Project
Reports (DPRs).

x This refers to the tendency to form friendships and other forms of interpersonal
Propinquity relationships with people we come across often in our daily lives.

Effect x People who live near each other or work in the same places are more likely to form a
bond with each other when compared to others who are not exposed to each other as
(#Psychology)
often. This is because constant exposure can lead to increase in trust.

FOCUS | July 2018 | RAU’S IAS 117


@civilminds
Glossary

x Even this also happens frequently during journeys…where co-passengers develop


friendship (mostly temporary) but sometimes permanent also.

x Anthropocentrism refers to a philosophical world view where human beings are


seen as superior to other living and non-living things.
x It justifies the exploitation of nature for the sake of human welfare.
x Enlightened anthropocentrism is a world view that says humans have ethical
obligations towards the environment but those can be justified in terms of
obligations towards other humans.

Anthropocentrism x For instance, environmental pollution can be seen as immoral because it


negatively affects the lives of other people. Similarly, the wasteful use of natural
(# Philosophy) resources is viewed as immoral because it deprives future generations of those
resources.
x Contrast to it is “Eco-centrism”. According to eco-centrism, there are no
existential differences between the human and non-human entities in nature,
which means humans are not more valuable than any other component of the
environment. Humans as well as plants, animals, and other constituents of
nature have an inherent value.

x This refers to a scientific hypothesis which states that the earth is a complex living
entity, with the sustenance of life dependent on the self-regulating interactions among

Gaia organisms and their inorganic surroundings.


x For instance, climatic conditions depend on the interactions among living organisms
hypothesis like human beings and their non-living atmosphere, all of which regulate each other
(# Ecology) constantly. The Gaia hypothesis is named after the mythical Greek goddess Gaia who
personifies the earth.
x For e.g. human beings and the usage of metal and mineral resources.

x This refers to an economic theory which views resources like oil and other minerals as
assets that are created through the application of human effort.
x This is in contrast to the typical view of environmentalists who see resources as that
which are provided by nature, limited in their overall supply, and prone to be used till
Resourceship they are no longer available.

(# Economics) x Resourceship considers the supply of resources to be dependent on the capability of


human beings to tap them, which in turn depends on factors like property rights which
shape human incentives.
x It also depends on human regulations, for example: - shift from fossil fuels to clean
energy will prevent complete exhaustion of fossil fuels.

x This refers to a scientific hypothesis which states that obesity in some human beings
can be explained by the existence of a ‘thrifty gene’ that aided the storage of excess
Thrifty Gene energy as fat.

(# Biology) x According to the hypothesis, certain genes, by helping in the storage of energy as fat,
might have enabled human ancestors live through periods of famine, thus enhancing
their chances of survival.

FOCUS | July 2018 | RAU’S IAS 118


@civilminds
Glossary

x The same genes, however, turned into a biological liability that led to obesity in some
people. The hypothesis was proposed as an explanation for the increased prevalence
of type-2 diabetes in certain populations by American geneticist James V. Neel.

x This refers to the scientific problem of estimating the different causes that led to a
particular outcome.
x Scientists trying to understand an observation may at times have to work backwards to
Inverse estimate the causal factors behind it. This is in contrast to the forward or the direct
problem problem where scientists try to find the outcome that results from a series of causes.
x A scientist observing a photograph, for instance, would be faced with the inverse
(# Science)
problem while trying to figure out the various causal factors that combined to form the
final photograph. The inverse problem is applied to understand processes in a wide
variety of sciences.

x This refers to any business that is focussed primarily on the purchase of patent rights
for products with the sole intent of profiting from it by stifling competition or filing
patent infringement claims.
x This is in contrast with other more genuine businesses that purchase patents to
Patent troll manufacture their own products for the market.
(#Economics) x A company engaged in patent trolling does not care much about developing its own
products using the patent.
x Instead, it is focussed on taking advantage of loopholes in the patent law to sue
companies with similar patents for patent infringement, thereby profiting from it.

FOCUS | July 2018 | RAU’S IAS 119


@civilminds

Part TWO

CONTRIBUTORS
ZONE
articles .opinions .essays .notes
by rau’s professors & students
@civilminds

lEAD ARTICLES

DOUBLING OF FARMERS INCOME BY 2022: IS


IT A REALISTIC TARGET?
#Economy
DR. J. C. SHARMA
FACULTY (GS: ECONOMY AND OPTIONAL: ECONOMICS)

Agricultural performance is relevant for inclusive growth ¾ This process of gradual improvement continued
as with almost 47 per cent of the workforce in India during the 10 years, i.e. during 2004-05 and 2013-14,
engaged in agriculture, unless this sector performs well, agri-GDP registered a growth rate of 3.7 per cent,
“sabka saath, sabka vikas” will not be possible. Moreover, while the overall GDP grew at an average rate of 8 per
as the World Development Report (2008) revealed, cent.
growth in agriculture is at least two to three times more
effective in reducing poverty than the same quantum of ¾ The agri-GDP growth has plunged to just 1.9 per cent
growth in non-agricultural sectors. So, from the during 2014-15 to 2017-18 (along with the overall GDP
standpoint of poverty alleviation, and hence inclusive growth of 7.2), half of what was achieved in the
growth, not only development of agriculture sector is previous ten years period.
necessary but improvement in wellbeing of 47% people
During the last four years if agriculture hit by back-to-
engaged in the sector is rather more relevant.
back droughts, it also got a windfall of thousands of
crores as savings in the import bill of crude oil with prices
Agriculture Growth – Past falling by more than 50 per cent. Moderating global
commodity prices also helped in taming inflation at

Trends home. Poor agri-performance has also reflected in


shrinking agri-trade surplus (exports minus imports)
from $25.5 billion by 2013-14, to just $8.2 billion by 2016-
The advance estimates for 2017-18 put agri-GDP
17. This does not go well for Indian agriculture as well as
growth at 2.1 per cent, down from 4.9 per cent in the
farmers.
preceding year. The comparison with agriculture growth
since 1991 will give a long term perspective:

¾ During 1991-92 to 1995-96, agri-GDP grew at a


Why Doubling of Farmers
modest annual rate of growth of 2.4 per cent, while
the overall GDP growth was 5.2 per cent per annum.
Income?
¾ There was a marginal improvement in the agri-growth According to Ramesh Chand, Member NITI Aayog, past
rate during 1999-2003-04, i.e. it grew at 2.9 per cent. strategy of the agriculture sector in India has focused
The overall GDP growth, in the same period, was 6 per primarily on raising agricultural output and improving
cent per annum. food security. This strategy involved (a) an increase in

FOCUS | July 2018 | RAU’S IAS 121


@civilminds

productivity through better technology and varieties, and farmer income increased from 4 per cent to 12 per cent.
increased use of quality seed, fertilizer, irrigation and agro- The share of income from non-farm sector declined from
chemicals, (b) incentive structure in the form of 11 per cent to 8 per cent.
remunerative prices for some crops and subsidies on farm
product, (c) public investments in and for agriculture, and (d) Real income of farmers, calculated on the basis of GDP
facilitating institutions. This strategy paid dividends as the deflator showed a real growth rate of 5.24 per cent
country was able to address severe food shortage that during the period 2002-03 to 2012-13. This implies that
emerged during 1960s. During the period 1965-2015, while farmers’ nominal income doubled in 6 years, real
since the adoption of green revolution, India’s food income took 14 Years to double. The growth rates in
production multiplied 3.7 times while the population nominal income of farm households across major states
multiplied by 2.55 times. The net result has been a 45% of the country varied from 6.71 per cent in West Bengal
increase in per person food production, which has made to 17.48 per cent in Haryana. Nominal income doubling
India not only self-sufficient in foodgrains at aggregate time is 8 to 11 years for states like Assam, Bihar, J&K,
level but also a net food exporting country. This strategy Jharkhand, and West Bengal. For all other states doubling
however did not recognize the need to raise farmers’ time for nominal income is around 6 years or less.
income. As per Chand’s estimate, the real income grew at However, the lowest real growth rate recorded was less
the compounded rate of 3.94 per cent per annum during than one per cent in Assam and the highest was 9.81 per
2004-05; to 2011-12, which is the fastest compared to cent for Madhya Pradesh. The doubling time is beyond
previous two decades. Based on the trends in farm 10 years for all states barring Andhra Pradesh, Madhya
income from 1983-84 till 2011-12, he concluded that: Pradesh, Odisha, and Rajasthan.

x The income earned by farmers net of input cost and Further, if the Consumer Price Index for Agricultural
wage bill has seen low and high growth paths in Laborers is used, the annual growth rate of real income
different periods; for farmers falls to 3.5 percent. With this growth rate
x The growth in farm income accelerated towards farmers' income will double in 20 years. Therefore, in
recent period ending 2011-12; order to achieve the target of doubling farmers' income
x Decent growth in farm income requires high growth in 5-6 years there is a need for much greater effort and
in output, favourable farm produce prices, and some focused attention.
cultivators moving out of agriculture;
A high growth in agriculture can reduce income
x
disparities and promote inclusive growth;
Doubling of Framers’ Income
x Low growth of farm income seems to have been
associated with agrarian distress; and, – Perception vis-à-vis
x More than half of farm households in the country will
remain below poverty level unless they adopt high- Reality
income earning avenues and augment their incomes
through non-farm activities. The Prime Minister had shared his dream of doubling
farmers’ income (DFI) for the first time at a kisan rally in
The major Source of information on income of farmers Bareilly (Uttar Pradesh) on February 28, 2016. A day later,
based on large sample survey is Situation Assessment the finance minister talked about this goal in his budget
Survey (SAS) by NSSO conducted during 2002-03 for the speech. Thereafter, this goal attracted the attention of
first time and repeated during 2012-13 These surveys policymakers, economists and most importantly,
indicate that the average total income of farmer farmers. Initially, it was not clear if the government
households increased by 11.75 per cent per annum from intended to double the real income of farmers or their
Rs 25320 in 2002-03 to Rs 77,112 in nominal terms. The nominal income.
largest share of farmers’ income was from cultivation,
which increased from 46 per cent in 2002-03 to 48 per On April 13, 2016, the government set up a committee in
cent in 2012-13. This was followed by wages and salaries, the Union Ministry of Agriculture and Farmers; Welfare,
the share of which declined from 39 per cent to 32 per to prepare a report on DFI. The committee seems to have
cent during the same period. Contribution of livestock to

FOCUS | July 2018 | RAU’S IAS 122


@civilminds

prepared a 14 volume report of which some have already likely reality is that investments are going to shrink
been uploaded on the Ministry’s website. Since there was further. Going by the present trends, the farmers’ real
no updated estimate of actual level of farmers’ incomes incomes cannot be doubled even by 2025 and the target
in 2015-16, the Committee arrived at estimates by of achieving it by 2030 seems more plausible
applying net state domestic product (NSDP) growth rates
on estimated state-wise farmers’ incomes of 2012-13
(from the NSSO). It found that an average Indian farmer
Strategy Proposed by
household earned Rs 96,703 in 2015-16 (this was Rs
77,112 in 2012-13 according to the NSSO). Doubling this Committee
to Rs1,93,406 (at 2015-16 prices) by 2022-23, needed real
incomes to grow at CAGR of 10.4 per cent at the all India- The Committee’s strategy focused on three areas —
level, and at differentiated rates at state-level. For productivity gains, reduction in cost of cultivation,
example, States like Bihar where real incomes of farmers and remunerative prices. Strategic framework has four
was at (-)1 per cent (CAGR) between 2002-03 and 2012-13 concerns — sustainable agri-production,
would require a CAGR of 12.3 per cent between 2015-16 monetization of farmers’ produce, re-strengthening
and 2022-23 to achieve the government’s target which extension services, and recognizing agriculture as an
seems very unlikely. There are similar unrealistic enterprise. Government have taken several initiatives in
expectations from other moderate-performing agriculture — providing soil health cards to more than 10
agricultural states like West Bengal, UP and Jharkhand. crore farmers and the enhanced crop insurance scheme,
targeting to complete 99 projects under the Pradhan
The report also uses an econometric model to work out Mantri Krishi Sinchayi Yojana by 2019, encouraging FDI in
the investment needed in agriculture, irrigation, rural food processing, ensure supply of inputs to farmers
roads, rural energy and rural development to attain assisting them in marketing their produce. These steps
10.41 per cent annual growth in real incomes for DFI by together likely to help in building India where the farmers
2022-23 over the base of 2015-16. The point to be noted by 2022 will earn double of what they earn in 2015.
is that farmers’ real incomes have increased by only 3.5
per cent per annum during 2002-03 to 2012-13. So, DFI But even if one makes the assumption that targeted
means three time’s higher effort and resources. That investment will somehow be made in agriculture, there
means an additional investment of about Rs 6,40,000 are two questions that need to be addressed. How much
crore at 2011-12 prices. And this does not include will agri-production increase as a result of this
investments in agri-logistics, cold chains, etc. Eighty per investment? Where will that increased production be
cent of this investment has to come from the absorbed? The past experiences reveal when production
government. The investments in and for agriculture need increases somewhat significantly, prices crashed, for
to rise by 22 per cent per annum in real terms if the example during 2016-17 and 2017-18, in several states,
dream of DFI is to be realised. prices of onions, potatoes, pulses and oilseeds crashed
when production increased. If domestic consumption
As two of the seven years have already passed and given can’t absorb increased outputs, can we export
the performance of agriculture in 2016-17 and 2017-18, competitively in global markets? The report does not
the required CAGR in the remaining five years will have to answer any of these fundamental questions. Instead, we
be much higher than these rates. For example, instead of have a list of hundreds of recommendations, ranging
10.4 per cent, the average incomes of farmers should from implementation of the Agriculture Produce and
grow at 13 per cent nationally if the goal is to be achieved Livestock Marketing, (Promotion and Facilitation) Act to e-
by 2022. To leapfrog from 2.5 per cent to 13 per cent and NAM to negotiable warehouse system to price deficiency
to mobilize the targeted amount of resources for payments to re- organizing KVKs and setting up a
investment are the real challenges before the sector as secretariat for DFI.
well as before the planners and policy makers. But the
report is totally silent on how, and from where, these
resources will be generated. In a climate of loan waivers, Farmers continued to Suffer
subsidies, and welfare programmes that dominate the
budget amounting to more than Rs 3.26 lakh crore, the

FOCUS | July 2018 | RAU’S IAS 123


@civilminds

It is well documented that farmers have suffered during 5,000 crore, and the dairy development fund of Rs 8,000
the last three-and-a-half years of the government, first crore, all with NABARD. A model Agricultural Produce and
from two successive droughts and then from tumbling Livestock Marketing (Promotion and Facilitation) Act,
agri-prices. In the first four years of the current 2017, was also circulated. Out of the 99 major and
government, agri-GDP is going to register an average medium irrigation projects identified for completion by
annual growth rate of around 2 per cent, which is almost 2019 through the LTIF, only few have been completed so
half of what was achieved during the previous 10 years far and that too many without field channels. The micro-
rule. So, without lifting its performance, and the incomes irrigation fund has not taken off yet, and the dairy fund
of farmers, it will not be feasible to achieve either “sabka was approved recently. With such a lackluster
sath, sabka vikas” or “doubling farmers’ incomes by performance, one cannot expect much improvement in
2022”, as committed by the Government. agriculture or farmers’ incomes.

The year 2107 has been a puzzling and painful year for
farmers. The monsoon was reasonably good, and so was Re-emphasis on Proposed
kharif production. Still, several states saw farmers’
agitations triggered primarily by very low prices for their Strategy
produce, be it onions, potatoes, pulses, oilseeds or
cotton. The profitability of major kharif crops for major On February 19-20, the Ministry of Agriculture and
producing states, based on market prices and projected Farmers’ Welfare organized a conference, “Agriculture
costs estimated by the CACP, has gone down 2022-Doubling Farmers’ Income”. The PM shared his aim
dramatically to less than 5 per cent or negative in kharif to overhaul the agriculture sector and double farmers’
2017 as depicted in the following graph: incomes through a four-pronged strategy. This involves
reducing costs of cultivation (through neem-coating urea
and soil health cards); ensuring better prices for produce
(through new cost plus-pricing MSP formula, reforming agri-
markets through e-NAM and upgrading 22,000 village-
markets); reducing wastage in the value-chain (through
proper storage, logistics, food processing, and implementing
Operation Greens-TOP (tomatoes, onions and potatoes); and
diversifying sources of farmer incomes (through blue,
organic, sweet revolution).

In order to take dream of DFI by 2022 closer to reality,


one may look at the Chinese experience during 1978-84,
when the country doubled farmers’ real incomes in six

Net Margin as percentage to years and reduced poverty by half (India took 18 years,
from 1993 to 2011, to cut poverty by half). China focused
primarily on incentives for farmers by moving from the
Projected C2 commune system to the household responsibility system
in land, and ensured higher prices for farmers. To cite
The worst, however, happened in Madhya Pradesh, one example, China’s MSP for wheat in 2014-15 was $
where the farmers’ agitation became violent and many 385 per tonne against India’s $ 226 per tonne. Similar
farmers died in the ensuing police firing. This led to a differences exist for other crops. The upshot of this
knee-jerk reaction in policies, from farm loan waivers in example is that India needs to focus on incentives for
many states to a drastic increase in import duties on farmers. Unfortunately, our policy is biased in favor of
pulses and edible oils. The Union budget for 2017-18 had the consumers and that inadvertently makes it anti-
also announced several measures for farmers, e.g. farmer. The government can address that issue by using
example, the augmentation of the Long Term Irrigation an income policy to protect the poor, and free up prices
Fund (LTIF) with NABARD by Rs 20,000 crore taking its for farmers, allow private trade to stock and operate
total to Rs 40,000 crore; the micro-irrigation fund of Rs

FOCUS | July 2018 | RAU’S IAS 124


@civilminds

freely and have unhindered exports. India can, then, ¾ Substituting the inherent consumer-bias in country’s
raise farmers’ incomes significantly, if not double them marketing and trade policies by outward looking
by 2022. Besides, the government may consider the marketing policy directly benefitting the farmers, e.g.
following suggestions: massive and growing food subsidies (marred with
high inefficiencies) is an example of such a bias.
¾ Expedite the implementation of major flagship
programmes by removing glitches, especially in crop ¾ Revisit the archaic ECA, revitalize inefficient APMCs,
insurance (PMFBY), irrigation (LTIF and micro- put an end to ad hoc export bans and ensure a stable
irrigation), and the dairy development fund. policy-environment for domestic agri-trade and
exports to flourish.
¾ Ensure an effective monitoring and dovetailing of agri-
trade and tariff policy with MSP policy through an ¾ Make policy to absorb the production boom either by
Empowered Committee to take quick decisions. making consumption to keep pace with production or
by allowing processing, exports or in augmenting
¾ Give high priority to agri-marketing reforms to create demand.
a seamless movement of agri-produce all over India.
So far, e-NAM has not delivered and will not do so ¾ Subsidy model to be replaced by an investment
unless the basics of agri-marketing from assaying, model. Of the total resources (Rs 3.66 lakh crores)
grading, storage, to dispute settlement are put on going to agriculture (and food), in the form of
track. A major impetus needs to be given to link subsidies on food and farm inputs, and other
farmer producer organizations (FPOs) to agri-markets programmes such as MNREGA, ensure the share of
NABARD has more than 2,000 FPOs and SFAC has investments in/for agriculture to be at least 75 per
created about 700 FPOs. This however requires cent.
significant amendments in APMC Acts permitting
FPO’s direct sale to consumer markets as well as in Unless these bold and rational policy decisions, chances
Essential Commodities Act removing restrictions on of augmenting farmer incomes are bleak, let alone
storages of essential commodities. doubling them by 2022-23.

THE WATER USE EFFICIENCY IN AGRICULTURE


#Economy #Environment
HIRALAL MANDAL, EDITORIAL TEAM
& EDUCATOR
resource is going to be altered significantly in future.
Coupled with this, limited availability of surface and sub-
Water being the necessity for crop production is one of
surface water resources in large tracts of arid and semi-
the most demanded natural resources in agriculture. So
arid land, over-exploitation of water at critical rates even
much so that almost 82 percent of the total supply of
in water abundance regions and continuing importance
water is consumed by agriculture sector along leaving
of agriculture, highlights the critical need for improved
a meagre 10 percent and 8 percent to the industrial and
water use efficiency in India.
domestic users respectively. However, with the rapid
development and urbanisation, and increasing The essay discusses the critical issue of water use
consumption needs of middle class driving the domestic efficiency in India. The essay is broadly divided into two
and industrial demand, the demand for this natural parts. First part emphasises upon the need for water

FOCUS | July 2018 | RAU’S IAS 125


@civilminds

use efficiency in the country while the second part China or the USA, it can save at least half of water
suggest the ways to enhance water efficiency in presently used for irrigation purposes.
agriculture.
Moreover, close to 55 percent of the cultivated area is
not covered under irrigation. This results in low
Need for water use productivity and high risk of production due to erratic
rainfall. Ways and means need to be devised to expand
efficiency irrigation and enable dryland agriculture to have access
to water to address at least critical water shortages.
Increasing incomes, growing urbanization and, rising Further, with irrigation predicted to remain the
prosperity are rapidly changing the composition of food dominant user of water, efficiency in water usage is
basket away from cereals towards high-value imperative.
agricultural commodities such as fruits, vegetables, milk,
Irrigation infrastructure in the country has expanded
poultry, fish and meat. These fruits, vegetables and
significantly over the years. The total irrigation potential
livestock products are more water intensive as
created (IPC- gross area that can be irrigated annually by
compared to cereals other than rice. Moreover, the
the quantity of water that could be made available by all
changing lifestyle, and dietary habits, increasing intake
connected and completed works up to the end of the
of fresh fruits and vegetables among the new
water courses) from major, medium and minor irrigation
prosperous population are creating pressure for their
schemes has increased from 22.6 million hectares
production in offseason requiring the much higher use
during pre-plan period to 113 million hectares at the end
of water. Besides, the amount of water required to
of the 11th Plan. Since, this represents more than four-
produce a unit of animal origin products, such as
fifths of India’s ultimate irrigation potential, the scope for
chicken, mutton and eggs, is much higher than plant
further expansion of irrigation infrastructure on a large
origin products, such as cereals, pulses and oilseeds.
scale is limited. Therefore, priority must be given to
These developments point to the growing demand for
improving the utilization of irrigation potential utilized
and therefore rising pressure on India’s limited water
(IPU- the total gross area actually irrigated by a project
resources.
during the year under consideration) of the existing IPC.
India accounts for about 17 percent of the world’s This requires effective utilisation of Command Area
population but has only 4 percent of the world Development Programme (initiated in 1973–74 to bridge
freshwater resources. Distribution of these water the gap between IPC and IPU).
resources across the vast expanse of the country is also
Over the years, there has been a significant shift in the
uneven. Further, as per the international norms, India is
sources of irrigation. The share of the canal in the net
a water stressed country and moving towards a water
irrigated area has declined from 39.8 % in 1950-51 to
scare due to its low per capita water availability which
23.6 % in 2012-13 whereas the share of groundwater has
stands at 1444 m3. Worth to mention here is that a
surged from 28.7% to 62.4% during the same period.
country is classified as Water Stressed and Water Scarce
Even though it reflects the efficiency and reliability of
if its per capita water availability goes below 1700 m3
groundwater irrigation, it raises several sustainability
and 1000 m3 respectively. The limited availability of
issues due to the injudicious utilisation of groundwater
water and uneven distribution coupled with the rising
through the explosion of tube wells. Increasing use of
income and water need, the stress on water use will only
the groundwater has resulted in over-exploitation of
increase in future.
water resources leading to significant decline in the
While the stress on limited water resources in the groundwater table, especially in northwest India. This
country is rising the scarcity is not reflected in use of unsustainable groundwater consumption compels an
water. India uses 2-4 times water to produce one unit of effective demand management and supply
major food crops as compared to other major augmentation measures in order to ensure water use
agricultural countries like China, Brazil, and the USA. efficiency in the agriculture sector.
Such differences in water use hints that India has huge
Furthermore, various regions including the districts of
scope for improvement in water efficiency. In other
South and North Interior Karnataka, Rayalseema in
words, if India can match the water use efficiency of
Andhra Pradesh, Vidarbha and Marathwada in

FOCUS | July 2018 | RAU’S IAS 126


@civilminds

Maharashtra, western Rajasthan and Bundelkhand irrigation technologies such as sprinkler and drip
region of Uttar Pradesh and Madhya Pradesh face irrigation system can be very helpful. Globally, it’s well-
serious water stress in the country. Low and erratic established fact that micro-irrigation technologies increase
rainfall, ineffective water-harvesting structures, depleted crop yield, save water, improve crop quality, enhance the
water storage in reservoirs etc. have affected the fertilizer/ chemical application efficiency, conserve energy,
agriculture production in these regions leading to reduce labour cost, improve pest management, increase the
farmers suicides. There is a strong case to provide feasibility of irrigating difficult terrains, improve the
perennial water source connectivity to these regions suitability of problem soils, increase tolerance to salinity etc.
along with the effective implementation of water Further, the new agronomic practices like raised bed
efficiency techniques. planting, a ridge-furrow method of sowing, sub-surface
irrigation, precision farming offer vast scope for
Ways to improve water economising water use.

Experts also suggest that promotion of alternative


efficiency methods of planting such as System of Rice
Intensification and Direct Seeded Rice can
A comparative analysis of the water use efficiency across significantly lead to water saving and water productivity.
countries reflects that India uses double or sometimes Further, application of the concept of multiple uses of
thrice the level of water to produce the same amount of water can enhance water productivity. Apart from
grain or cereals. This means that India needs to enhance enhancing the water productivity, it also helps in
its water efficiency not only to increase production but generating more income benefits, decreasing
also to meet the future demand of water security in the vulnerability by allowing more diversified livelihood
country. Accelerating water efficiency in the country will strategies and increasing the sustainability of the
require the application of the multiplicity of instruments. ecosystem.
Pradhan Mantri Krishi Sinchai Yojana (PMKSY) Watershed development is suggested another way to
provides a sound framework for the expansion as well bring efficiency in water usage. This is done through the
as effective use of water in irrigation. Under this introduction of water resources conservations method
programme, the Government has committed to accord in suitable areas and development of micro-water
high priority to water conservation and its management. structures for rainwater harvesting. It has a direct effect
To this effect, the scheme has been formulated with the on water resources availability, groundwater recharge
vision of extending the coverage of irrigation ‘Har Khet and socio-economic conditions of the population.
ko pani’ and improving water use efficiency ‘More crop
The specialised solution is required in chronically water-
per drop' in a focused manner with an end to end
stressed areas where the normal measures may not be
solution on source creation, distribution, management,
effective. Connecting highly water-stressed areas with
field application and extension activities. It is important
the perennial source of water through linking of rivers or
to note that the scheme has been formulated
water grids is one such option. The value added agri-
amalgamating ongoing schemes viz. Accelerated
horti-pastoral agro-forestry systems and alternative
Irrigation Benefit Programme (AIBP) of the Ministry of
source of livelihood are required in those water-stressed
Water Resources, River Development & Ganga
regions of the country.
Rejuvenation (MoWR, RD&GR), Integrated Watershed
Management Programme (IWMP) of Department of Land The effective water management is critically linked with
Resources (DoLR) and the On-Farm Water Management the performance of local level water institutions.
(OFWM) of Department of Agriculture and Cooperation Therefore, institutional restructuring in favour of
(DAC). participatory irrigation management and water users
associations (WUAs) needs to be strengthened. The
Besides, the method of irrigation followed in the country
National Water Policy stresses on the concept of
is flood irrigation, which results in a lot of water loss.
Participatory Irrigation Management and WUA. These
Greater efficiency in irrigation can be achieved
will help in actively involving the people in the
through proper designing of irrigation system for
implementation of the irrigation project and make the
reducing water conveyance loss. In this context, micro-

FOCUS | July 2018 | RAU’S IAS 127


@civilminds

concept of water efficiency more acceptable among the of water and encouraging efficient use of them in
common people. agriculture. Along with these, imposing strict regulatory
measures for misusing the water and generating
Conclusion awareness among the common masses will help not
only to conserve water but in the long run facilitate
Climate change along with severe neglect and lack of country to tide over the water crisis. The challenge is
monitoring of water resources can lead to water scarcity manageable provided we have favourable policies and
in the country in the next one-two decades. It is mechanisms to persuade our people to change their
therefore essential to thwart this crisis by using the lifestyle.
technologies and resources to conserve the existing
water resources, ensuring efficient management of
water across the sectors, promoting reuse and recycling

INDIA AND OBOR


#India and the World
RITESH KUMAR SINGH
FACULTY: G.S. CONTEMPORARY ISSUES, ANSWER WRITING & ESSAY,
AND EXECUTIVE EDITOR

China’s ‘One Belt, One Road’ could potentially allow of time. The Silk Road derives its name from the lucrative
India a new track in its own attempt to integrate Chinese silk trade, a major reason for the connection of
South Asia. trade routes into an extensive transcontinental network.
Trade on the Silk Road was a significant factor in the
In ancient times, diligent and courageous people of
development of the civilizations of China, the Indian
Eurasia explored and opened up several routes of trade
subcontinent, Persia, Europe, the Horn of Africa and Arabia,
and cultural exchanges that linked the major civilizations
opening long-distance, political and economic interactions
of Asia, Europe and Africa, collectively called the *Silk
between the civilizations.
Road by later generations. For thousands of years, the
Silk Road Spirit - "peace and cooperation, openness and ___________________________________________________
inclusiveness, mutual learning and mutual benefit" - has
been passed from generation to generation, promoted
the progress of human civilization, and contributed
CHINA’S NEW INITIATIVE
greatly to the prosperity and development of the To revive the erstwhile spirit of the old Silk Route,
countries along the Silk Road. China’s National Development and Reform Commission
(NDRC) released an expansive blueprint on “One Belt,
-----------------------------------
One Road (OBOR)” initiative. This initiative is a Chinese
*The Silk Road, or Silk Route, is a network of trade and framework for organizing multinational economic
cultural transmission routes that were central to cultural development through two component plans, the land-
interaction through regions of the Asian continent based "Silk Road Economic Belt" (SREB) and
connecting the West and East by merchants, pilgrims, oceangoing "Maritime Silk Road" (MSR).
monks, soldiers, nomads, and urban dwellers from China
and India to the Mediterranean Sea during various periods

FOCUS | July 2018 | RAU’S IAS 128


@civilminds

In OBOR, the SREB initiative was announced by Xi At the same time, India has begun its own long overdue
Jinping, the President of China, on a visit to Kazakhstan. initiatives in the Indian Ocean to counter the Chinese
Essentially, the 'belt' includes countries situated on the and ensure that China does not start to dominate the
original Silk Road through Central Asia, West Asia, the Indian Ocean. India’s maritime initiative on the Indian
Middle East, and Europe. The initiative calls for the Ocean has been variously presented as **“Project
integration of the region into a cohesive economic area Mausam”, and ***“Spice Route”. The initiative
through building infrastructure, increasing cultural envisions India as the centre of the “Indian Ocean
exchanges, and broadening trade. Apart from this zone, world,” which stretches from Africa in the west to
which is largely analogous to the historical Silk Road, Southeast Asia in the east. Like China’s Maritime Silk
another area that is said to be included in the extension Road, Project Mausam would boost regional commercial
of this 'belt' is South Asia and Southeast Asia. and cultural linkages – but where the MSR would have all
roads leading back to China, Project Mausam seeks to
The MSR, also known as the "21st Century Maritime
return India to its role as the centre of Indian Ocean
Silk Route Economic Belt" is a complementary initiative
trade.
aimed at investing and fostering collaboration in
Southeast Asia, Oceania, and North Africa, through ___________________________________________________
several contiguous bodies of water – the South China
**Project ‘Mausam’ focuses on how monsoon as natural
Sea, the South Pacific Ocean, and the wider Indian
phenomenon has shaped interactions between countries
Ocean area.
and communities connected by the Indian Ocean. Project
Mausam, would endeavour to position itself at two levels :
OBOR AND INDIA at the macro level where it would reconnect and re-establish
communications between countries of the Indian Ocean,
India’s hesitation in embracing the China-led OBOR is
leading to an enhanced understanding of cultural values
the projection of the China-Pakistan Economic
and concerns; while at the micro level, the focus would be to
Corridor (CPEC) as a flagship project. The CPEC is
understand national cultures in their regional maritime
expected to connect Kashgar in China’s restive province
milieu. Thus Project Mausam is an exciting, multi
of Xinjiang with the Gwadar port in the volatile
disciplinary trans-national project that tries to rekindle long
Baluchistan province of Pakistan. It will pass through
lost ties across the Indian Ocean Littoral and forges new
Pakistan-occupied Kashmir (PoK) and Gilgit-Baltistan—
avenues of cooperation and exchange between India and
both Indian territories occupied by Pakistan. This is at
states of the Indian Ocean.
the core of India’s doubts about China’s sincerity in
seeking its collaboration. How can this initiative navigate
the irreconcilable geometries of South Asia that prevent ***The Spice Route initiative works to establish itself as a
India from providing full backing to OBOR? A formal nod symbol of peace and camaraderie by bringing places and
to the project will serve as a de-facto legitimisation to people of the world closer. The Spice Route initiative intends
Pakistan’s rights on Pakistan-occupied Kashmir and to follow the route which once existed between the countries
Gilgit-Baltistan under the China-Pakistan Economic and to bring back the cultural rapport which used to exist
Corridor (CPEC) that is “closely related” to OBOR. and become a cultural rendezvous. This thought led the
Further, there are conflicting views in India on whether Government of India and the State Government of Kerala
China’s OBOR strategy represents a threat or an joining hands to create the Muziris Heritage Project (MHP).
opportunity. Some view it as a strategy which China will use The main goals of the Spice Route Initiative are to: (1)
to encircle India. Some consider it as a great opportunity to connect all 31 countries of Asia, Far East, Africa and Europe,
attract the much-needed infrastructure finance into India to (2) work for protection and promotion of heritage cities/sites
fill its infrastructure growth and boost growth and in Spice Route countries, (3) create ‘One Destination’ to
employment. And some regard it as a fait accompli in which trigger voyages and excursions for the traveller, and (4) use
India must engage to derive as much benefits from it as Spice Route as a platform to work for peace in and around
possible. What is emerging is a competitive yet cooperative the regions of the route.
approach from both China and India, in the Indian Ocean ___________________________________________________
and in the South China Sea and parts of East Asia.

FOCUS | July 2018 | RAU’S IAS 129


@civilminds

One view is that China’s actions are not directed at India China which saw almost 3 decades of spectacular growth
but is China’s response to reduce its vulnerability on oil rates. India also shed its slow growing so called “Hindu”
and trade shipments through the Straits of Malacca. If growth rate and began to grow much faster. If both
China’s oil supplies are threatened through the Straits of China and India could continue to grow rapidly they are
Malacca it needs alternative options, with access to the projected to become more than 50% of the world
Indian Ocean and the Gulf. India also needs to overcome economy by 2030 – roughly where they were two
infrastructure-related constraints to enhance centuries ago. China and India have much to gain if they
connectivity for its overseas trade, which contributes cooperate and much to lose if they compete in a manner
substantially to the national economy. The MSR could be that pulls each other down.
an effective maritime supplement to the land-based
Bangladesh-China-India-Myanmar (BCIM) Economic
Corridor under active consideration by New Delhi. It
Options for India
could be dovetailed with India’s own ****‘Sagarmala’ Fundamentally, New Delhi needs to resolve for itself
project, and thereby contribute to the nation’s efforts to whether OBOR represents a threat or an opportunity.
enhance sea trade connectivity, while also progressively The answer undoubtedly ticks both boxes. Chinese
leading to ‘port-led development’ of the hinterland, and political expansion and economic ambitions, packaged
the SEZs. as OBOR, are two sides of the same coin. To be firm
while responding to one facet, while making use of the
___________________________________________________
opportunities that become available from the other, will
****Sagar Mala project is a strategic and customer- largely depend on the institutional agency and strategic
oriented initiative of the Government of India to modernize imagination India is able to bring to the table.
India's Ports so that port-led development can be
First and foremost, India needs to match ambition with
augmented and coastlines can be developed to contribute in
commensurate augmentation of its capacities that
India's growth. It looks towards "transforming the existing
allows it to be a net security provider in the Indian Ocean
Ports into modern world class Ports and integrate the
region. This will require New Delhi to not only overcome
development of the Ports, the Industrial clusters and
its chronic inability to take speedy decisions with respect
hinterland and efficient evacuation systems through road,
to defence partnerships and procurement, but will also
rail, inland and coastal waterways resulting in Ports
necessitate a sustained period of predictable economic
becoming the drivers of economic activity in coastal areas.
growth; OBOR can assist in the latter.
___________________________________________________
Therefore, just as U.S. trade and economic architecture
Further, according to China, OBOR initiative can also be underwrote the rise of China, Chinese railways,
linked with India’s ‘Mausam’ and ‘Spice Route’ projects, highways, ports and other capacities can serve as
thus forming a new starting point and a new bright spot catalysts and platforms for sustained Indian double-digit
in China-India cooperation. On the surface, the projects growth. Simultaneously, India can focus on developing
do have much in common – both seek to expand last-mile connectivity in its own backyard linking to the
regional integration, especially when it comes to trade OBOR — the slip roads to the highways, the sidetracks to
and commerce. But on a deeper level, both the MSR and the Iron Silk Roads.
Project Mausam are about expanding influence –
Arguably, OBOR offers India another political
culturally, economically, and even strategically.
opportunity. There seems to be a degree of Chinese
Practically and going by the changing dynamics of the eagerness to solicit Indian partnership. Can India seek
21st century, China and India must cooperate rather reworking of the CPEC by Beijing in return for its
than compete if an Asian century is to be realized. active participation? Furthermore, for the stability of
According to Angus Maddison, a British economist, till the South Asian arm of OBOR, can Beijing be motivated
early 19th century, India and China together constituted to become a meaningful interlocutor prompting rational
almost 50% of the global economy. But by 1950 their behaviour from Islamabad? OBOR could potentially
share of the global economy had dropped to only allow India a new track to its own attempt to integrate
around 15%. Since 1980’s both economies saw a step South Asia.
increase in their growth rates – more so in the case of

FOCUS | July 2018 | RAU’S IAS 130


@civilminds

Further, India wants better connectivity with Afghanistan So, India will soon have to articulate a definite stand on
and the rest of Central Asia to its west, and with OBOR by assessing its security concerns along with the
Myanmar and rest of South East Asia to the east. Both benefits of greater engagement with China. If OBOR
regions suffer from intermittent connectivity due to does generate substantial socio-economic benefits of
insurgency, and terrorism has stalled several infrastructure and connectivity, India will gain from a
infrastructure projects undertaken by India in these stable neighbourhood and a prosperous Asia.
regions. Any financial and engineering inputs arising out
of OBOR will take this regional connectivity forward.

FOCUS | July 2018 | RAU’S IAS 131


@civilminds

lEAD ESSAYS

MARITIME SECURITY
TUSHAR
GS-2A

there are also some countries like Australia whose whole


Maritime Security is a very general word, but the exact
boundary is a coast line. Thus, countries like Australia
meaning of the word is very deep and definitely very
should really seek to ensure maritime security.
important for overall security of any nation. Water being
one of the 3 major pillars of armed forces. So, let us first Now, if we talk about India, India is somewhere in
understand what do the term “Maritime Security” between the two extremes. We (Indians) have a very long
means? Why it is important for India? And, finally, International border on land but at the same time, we
what has India done to secure it? also have one of the longest coastlines in the world. Our
position is so important in the sea that we are the
‘Maritime Security’ is amalgamation of two words
country to have an ocean named after it. Indian Ocean!
“maritime” and “security”. “Maritime Security” is derived
Now we have understood the meaning of maritime
from the Latin word “Maritime” meaning “Related to Sea”.
security, its importance for a nation and its importance
Therefore, “maritime security” means security related to
for India. But before I get into the steps taken by India to
sea. But the term “Security” is a deep one. While
secure its seas, I would like to go into a bit of History.
understanding it, one should ask oneself why does one
need security? What are the threats? To what extent History, as we know, is written by victors, and victors are
should one strive for security? Is it multifaceted or there never forgetful to mention the reasons why they were
is only one side? These are few questions that would help always victorious. From the start of civilization, Indus
us understand the term “Security” in its entirety. Valley Civilization – sea routes, sea transport and in
general “sea” have been important for a civilization. If
“Security” is meaningless with a threat or potential
there were no sea or no safe sea, then Indus Valley
threats. Therefore, we should understand all the threats
Civilization’s trade with west would not have happened
that may arise. Threats can be of damage to economy,
or the least be limited. Coming to Medieval India – the
defence and integrity, social, terrorism, full-blown war,
great Chola King Raja Raja Chola I (VI 1000 AD) was
etc. These are the few dimensions to threat and thus,
expert in navy. He expanded his kingdom from
security should be also multi-dimensional and must
Madagascar in the west to Indonesia in the west. This
cover all these.
would have never happen if he would not have a mighty
Now, we can attempt to understand the term “maritime navy. Thus, “History” is evident that a navy can make a
security”. It is securing our (a nation’s) seas from threats kingdom to an empire or vice-versa. Therefore, for India
to economy, defence and integrity, social, terrorism, full- Navy means a lot and with Navy, comes the maritime
blow war etc. Bur we are still left with the question – Why security.
should a nation care about maritime security? Is it really
India has many maritime threats and in all the
the important? The answer to this depends from nation
dimensions discussed earlier. The threat in economy. In
to nation.
today’s world, the cheapest mode of transportation is not
A nation that does not have any threat from Sea on any railway or road way but water ways. Most of the items
dimension should not really bother about maritime are imported or exported travels through a ship.
security. For example, Mongolia - It is a land locked Therefore, maritime security is important for economy.
country, and thus, has no sea – let alone any threat from But at the same time there are few threats in Indian
sea. So, countries like Mongolia should not bother. But,

FOCUS | July 2018 | RAU’S IAS 132


@civilminds

Ocean. For example, Somalian Piracy in the west and at Ocean Naval Symposium (IONS) and Indian Ocean
the bottle neck of Gulf of Aden and Gulf of Omar. Commission (IOC). These associations help India to
Towards, the south of Indian Ocean we have two bottle- collaborate with various littoral states of Indian Ocean
necks – Cape of good-hope and the Mosambiq-Channel. and to resolve disputed peacefully.
India know this and has also taken steps to counter
With this we can see that India faces many threats in
them.
Indian Ocean and it is also clearly visible that India has
India has engaged and secured a strategic alliance on taken many steps to counter these threats to promote
Naval cooperation on with Oman in the Duqm – island, maritime security. But, again we should ask our self “Is
with Iran in the Chabahar Port. These two are counter this enough?” And, I will tell you that we have over looked
steps taken to control Somalian piracy and secure seas in a chief issue in our discussion. The issue is Science and
West Indian Ocean. We are also in talks with France for Technology.
Comros Island (middle of Mozambique Channel), with
If we talk about Navy, India though not very far behind
Madagascar, with Mauritius, and with Seychelles – a
other nation, yet, lacks in some advanced technologies
group of islands towards South – West Indian Ocean. All
know to human kind. They are submarines, and war –
these are to secure the South Indian Ocean. We have
ship along with supporting technology of navigation
also signed LEMOA agreement with USA for Cooperation
required by both of them.
in Diego Garcia – an island in the middle of Indian Ocean
– a very strategic forward post. Our Naval fleet has some more problem like dependence
on other nations, old and aging fleet and slightly less
Now, if we assume that we have secured Indian Ocean
advanced and less in quantity. We fall behind other
then it would be a blunder on our past. Why? Because,
nations – mainly China – in these terms. But, as with
we have only secured Indian Ocean at a macroscopic
other issues, we have also taken steps to resolve them.
level at an International Level – but, not at an regional
level. As we might be aware of 26/11 attacks on Mumbai. INS Arihant class submarines is one such step. It is a
Post the attacks, we were hit by reality that Navy does nuclear submarine indigenously developed by India
not protect the coast line, it only protects the seas. For (DRDO) and is capable of shooting Submarine launched
coastline, we have another branch of paramilitary forces Ballistic Missiles (SLBMs). I am proud to say that we are
known as “Coast-Guard”. It is the duty of Coast-guards to the only nation outside the elite groups of P5 nations
protect seas that are too-shallow for Navy. And, this is that have this capability and indigenous technology.
the reason India missed detecting the incoming terrorist Our recent launches of satellites of “Indian Regional
from Pakistan. But, India has learnt from the mistakes. Navigation Satellite System” (IRNSS) is another step
India has improved Intelligence Bureau (IB), improved towards the same goal. A goal to have independent Navy
National Security Guards (NSG), improved the Coast and for that to have Independent Navigation. With these
Guards, have set up anti-terrorism and counter satellites India has set up a regional navigation system
insurgency schools and the most important identified the known as Navigation through Indian Constellation
mistake. (NAVIC). NAVIC will truly make our Navy independent and
Now, if we talk about a full-blown war scenario then a is of our strategic interest in maritime security.
term called “String – of – Pearls” is common among the Thus we can say that we the people of India are moving
elites on Indian Nation Security. “String of Pearls” refers towards maritime security. Although, we should also note
to China’s strategic Points or locations in Indian Ocean here that all these are step towards maritime security
and also in Land-border. These are the points where and not the destination itself. These systems have their
China has a strategic agreement with various nations own limitations like we have only 1 INS Arihant class
starting from Myanmar, to Maldives to Pakistan to submarine, the next one in the same class INS Aridaman
Djibouti in Red. It has covered India from all sides and is due to 2019. Our NAVIC is a limited navigation system,
pose a serious threat to maritime security interested of limited to Indian region and Indian ocean Region. Our
India in Indian Ocean. fleet is still composed of other nation’s technologies and
India has taken steps to counter it. Some of the step have is small in size compared to other major power in the
already been discussed like the strategic agreement with world.
various nations in South, South West and Central Indian But before I end, I would like to stress on another
Ocean Region. On the top of all these India is also a dimension of maritime security – the social dimension.
member of Indian Ocean Rim Association (IORA), Indian

FOCUS | July 2018 | RAU’S IAS 133


@civilminds

India is a country of socio – economic inequalities. A marginalised section of society. It covers all the three
majority of fishermen are marginalised and like in poor parts of health care namely primary, secondary and
living condition. They are vulnerable to both poverty and territory GOI is a welfare oriented government with
adverse calamities. They are susceptible to Tsunamis, numerous welfare schemes to protect. The interest of all
bad catch and poor health. And when people of our the sections of the society. GoI has also improved its
coasts are secured then what is meaning of this maritime weather forecasting and early warning system to protect
security. The more the people are marginalised the more people from adverse natural forces. Thus, the GoI is on
they are vulnerable to crime and specifically speaking the right track to ensure social and economic needs of
cross-border crimes. This social vulnerability forces them the people of India including the people of the coasts.
to allow drug trade in India. India’s drugs are imported
Thus, I would say that India has a lot of threats in
from various countries via small fishermen (also from
maritime security and till now India has proved to the
land but for current discussion not relevant). This illegal
world that it is capable of withstanding all the threats
trade is the purse of almost all terror outfits and a major
that have come to it. India has ensured maritime security
problem in Indian society. If we cannot guarantee the
and with all the various programmes, technologies,
people of coasts – a secure life, a life of dignity, a life of
agreements etc. discussed before, India will continue to
literacy and a life of health back up then we would fail in
ensure maritime security. And I would like to conclude
maritime security at the deepest level of social security.
with Sh. Hari Vansh Rai Bachchan’s famous lines “Lahero
But the same issues I would also like to point out some of
say darkar Kabhi nauka paar nahi hoti, Himmat walon ki
Governments initiatives.
kabhi haar nhi hoti” (No ships sails fearing the waves and
The Government of India (GoI) have launched Ayushman similarly, no brave person has ever been defeated).
Bharat with the aim to solve healthcare problems of the

FOCUS | July 2018 | RAU’S IAS 134


@civilminds

Part Three

practiCe
ZONE
mcqs . descriptive questions .
questions based on editorials
@civilminds

Here are given 50 Multiple Choice Questions (MCQs)

Multiple Choice Questions for self-practice. These questions have been framed
from this issue itself. So, a reader is expected to
attempt the questions, and also refer to the Answer
(MCQs) for prelims gs paper i Key given after the MCQs. These are helpful for General
Studies- Paper I (Preliminary Examination).

Q1. Consider the following statements regarding (d) Neither 1 nor 2


Raja Rammohan Roy:
1. He demanded the abolition of the Company’s Q4. Consider the following statements:
trading rights and the removal of heavy export 1. Ramanuja laid emphasis on Bhakti as the principle
duties on Indian goods. means of attaining the supreme realty (God) or
2. In 1825, he established a Vedanta College in which final bliss.
courses both in Indian learning and in Western 2. Ramanuja propounded the doctrine of
social and physical sciences were offered. Vishishtadvaita.
Which of the statements given above is/are Which of the statements given above is/are
correct? correct?
(a) 1 only (a) 1 only
(b) 2 only (b) 2 only
(c) Both 1 and 2 (c) Both 1 and 2
(d) Neither 1 nor 2 (d) Neither 1 nor 2

Q2. Consider the following statements regarding Q5. Ambubachi Mela is related to which of the
Raja Rammohan Roy: following states?
1. In 1818, he founded a new religious society, the (a) Assam
Brahma Sabha.
(b) Bihar
2. He published ‘Precepts of Jesus’.
(c) Manipur
Which of the statements given above is/are
(d) Tripura
correct?
(a) 1 only
Q6. Consider the following statements:
(b) 2 only
1. The first epigraphic notice of Kamakhya is found
(c) Both 1 and 2
in the 9th-century Tezpur plates of
(d) Neither 1 nor 2 Vanamalavarmadeva of the Mlechchha dynasty.
2. The Kamakhya Temple is situated on the Chail
Q3. Consider the following statements: Hill.
1. Ramanuja was born in Tamil Nadu in the eleventh Which of the statements given above is/are
century and he was deeply influenced by the correct?
Nayanars. (a) 1 only
2. Ramanuja gave a philosophic basis to the (b) 2 only
teachings of Vaishnavism.
(c) Both 1 and 2
Which of the statements given above is/are
(d) Neither 1 nor 2
correct?
(a) 1 only
Q7. Consider the following statements regarding
(b) 2 only
significance of the battle of Plassey:
(c) Both 1 and 2

FOCUS | July 2018 | R AU’S I AS 136


@civilminds

1. The battle of Plassey exposed the subcontinent’s Which of the statements given above is/are
internal conflicts, destroying the native dynasties correct?
then in power and also the economy of Bengal. (a) 1 only
2. Britishers got access to rich revenues of Bengal (b) 2 only
which enabled them to organize a large army and (c) Both 1 and 2
meet the cost of the conquest of rest of India.
(d) Neither 1 nor 2
Which of the statements given above is/are
correct?
Q11. Which of the following is incorrectly matched?
(a) 1 only
1. Rail Madad- An App to expedite and streamline
(b) 2 only
passenger grievance redressal
(c) Both 1 and 2
2. RPGRAMS- Railway Passenger Grievance
(d) Neither 1 nor 2 Redressal and Management System
3. Menuonrails- Creating awareness to the Railway
Q8. Consider the following statements: Passengers for the items served to them on their
1. Siraj ud-Daulah was the last independent Nawab Rail Journey
of Bengal who had succeeded to the throne after Select the correct answer using the code given
his grandfather Ali Vardi Khan. below.
2. The battle of Plassey was fought on December 23, (a) 1 only
1757. (b) 2 only
Which of the statements given above is/are (c) 2 and 3
correct?
(d) None of the above
(a) 1 only
(b) 2 only
Q12. Consider the following statements:
(c) Both 1 and 2
1. Indian National Strategy for Standardization (INSS)
(d) Neither 1 nor 2 is the result of the combined efforts of the
Ministry of Commerce and Industry, Ministry of
Q9. Consider the following statements regarding Consumer Affairs and industry stakeholders.
Kabir: 2. It provides a vision for the country to achieve the
1. His teachings became the forerunner of highest quality standards in production and
Kabirpanth. distribution of goods and services in an attempt
2. The main elements of Kabirpanth is the to reclaim Brand India.
veneration of one God. Which of the statements given above is/are
Which of the statements given above is/are correct?
correct? (a) 1 only
(a) 1 only (b) 2 only
(b) 2 only (c) Both 1 and 2
(c) Both 1 and 2 (d) Neither 1 nor 2
(d) Neither 1 nor 2
Q13. Which of the following is/are the potential
Q10. Consider the following statements regarding benefits of ‘standardisation’ of products and
Kabir: services?
1. He was the pioneer of the inclusive cultural and 1. Boosting exports
political traditions. 2. Domestic economic growth
2. Bijak is the compilation of the verses of Guru 3. Benefits to consumers
Nanak.

FOCUS | July 2018 | R AU’S I AS 137


@civilminds

Select the correct answer using the code given 2. NBFCs can offer banking services such as loans
below. and credit facilities, retirement planning, money
(a) 1 only markets, underwriting and merger activities.
(b) 1 and 2 Which of the statements given above is/are
(c) 2 and 3 correct?

(d) 1, 2 and 3 (a) 1 only


(b) 2 only

Q14: Which of following statements is/are correct (c) Both 1 and 2


regarding ‘Adopt a Heritage’ scheme? (d) Neither 1 nor 2

1. The scheme is being implemented by the Ministry


of Tourism, Ministry of Culture and Archaeological Q17: Which of the statements given above is/are
Survey of India. correct?
2. The scheme aims to bring synergy among public 1. Stressed assets are loans on which the borrower
sector companies, private sector companies and has defaulted or it has been restructured.
corporate citizens/individuals to effectively 2. An NPA is a loan or advance for which the
promote responsible tourism. borrower has failed to repay the principle or
3. The project primarily focuses on providing basic interest for a period of 90 days.
amenities that includes cleanliness, public 3. Restructured asset or loan are that assets which
conveniences, drinking water etc. in tourist sites. got an extended repayment period, reduced
Select the correct answer using the code given interest rate, converting a part of the loan into
below. equity, providing additional financing, or some
(a) 1 only combination of these measures.

(b) 1 and 2 Select the correct answer using the code given
(c) 2 and 3 below:

(d) 1, 2 and 3 (a) 1 only


(b) 1 and 2

Q15: Consider the following statements regarding (c) 2 and 3


Asian Infrastructure Investment Bank (AIIB): (d) 1, 2 and 3
1. It is a multilateral development bank.
2. It aims to improve social and economic outcomes Q18: Which of the statements given above is/are
in Asia only. correct?
Which of the statements given above is/are 1. The Liberalized Remittance Scheme (LRS) is a
correct? facility provided by the RBI for all resident

(a) 1 only individuals including minors to freely remit upto a


certain amount in terms of US Dollar.
(b) 2 only
2. The LRS was launched in 2004 and regulations for
(c) Both 1 and 2
the scheme are provided under the FEMA Act
(d) Neither 1 nor 2
1999.
Select the correct answer using the code given
Answer- a
below:
(a) 1 only
Q16: Consider the following statements:
(b) 2 only
1. Non-banking financial companies (NBFCs) are not
(c) Both 1 and 2
allowed to take deposits from the public.
(d) Neither 1 nor 2

FOCUS | July 2018 | R AU’S I AS 138


@civilminds

Q19: Which of the following may help in controlling Q22. Consider the following statements about
appreciation of rupee? Aspirational District Programme:
1. Raising interest rate by RBI. 1. Districts will be ranked on their "incremental
2. Using forex reserve by RBI in exchange market. progress" through Delta Ranking
3. Providing incentives to exporters. 2. It will be monitored by NITI Aayog in collaboration
4. RBI raising funds through foreign currency non- with other ministries

repatriable (FCNR) deposits. 3. The programme focuses on certain key themes


Select the correct answer using the code given like education, skill development, health etc.
below: Which of the statements given above is/are
(a) 1 and 2 correct?

(b) 1, 2 and 3 (a) 1 only

(c) 2, 3 and 4 (b) 2 only

(d) 1, 2, 3 and 4 (c) 2 and 3


(d) 1, 2 and 3

Q20. Consider the following statements regarding


Gross Fixed Capital Formation: Q23. Which of the following statements about Delhi

1. It refers to the net increase in physical assets Agreement of 1952 between Jawaharlal Nehru
(investment minus disposals) within the financial and the then Prime Minister of Jammu and

year. Kashmir, Sheikh Abdullah is/are correct?

2. It accounts for the consumption (depreciation) of 1. Residuary power to be vested in the state of
fixed capital, and also includes land purchases. Jammu & Kashmir and not with Union
Government
Which of the statements given above is/are
correct? 2. The state of Jammu & Kashmir not to have their
own flag
(a) 1 only
3. The Supreme Court of India will have no
(b) 2 only
jurisdiction with respect to Jammu & Kashmir.
(c) Both 1 and 2
Select the correct answer using the code given
(d) Neither 1 nor 2
below?
(a) 1 only
Q21. The Union Cabinet has approved certain
(b) 2 and 3
official amendments to the National Medical
(c) 3 only
Commission (NMC) Bill, 2017. Which of the
following statements with respect to (d) 1 and 3

amendments is/are correct?


1. Addition of Bridge Course Q24. Article 35A of the Indian Constitution has been
2. Increased Representation from States recently in news. Which of the statements
given below about Article 35A is/are correct?
3. More stringent punishment for unqualified
practitioners 1. It was added through a Presidential Order of 1954

Select the correct answer using the code given 2. It defines the concept of “Permanent Citizens”
below? 3. It pertains to north-eastern states of India.
(a) 1 and 2 4. It has been challenged in a Supreme Court

(b) 2 and 3 Select the correct answer using the code given
(c) 1 and 3 below?

(d) None of the above (a) 1, 2 and 3


(b) 1 and 3
(c) 1, 2 and 4

FOCUS | July 2018 | R AU’S I AS 139


@civilminds

(d) 2, 3 and 4 Q28. Which of the following is/are members of


Governing Council of NITI Aayog?
Q25. The Union Cabinet has approved DNA 1. Prime Minister
Technology (Use and Application) Regulation 2. Ex officio Members of NITI Aayog
Bill, 2018. Which of the following statements 3. Full time members of NITI Aayog
can be said to be the mandate of the Bill? 4. Chief Minister of Delhi
1. To expand the application of DNA-based forensic Select the correct answer using the code given
technologies to support and strengthen the below?
justice delivery system.
(a) 1, 2 and 3
2. It allows law enforcement agencies to collect DNA
(b) 2 and 3
samples, create DNA profiles and create special
(c) 2, 3 and 4
Databanks for forensic-criminal investigations.
(d) 1, 2, 3 and 4
3. It provides for mandatory accreditation and
regulation of DNA laboratories.
Q29. The Union Cabinet has approved the proposal
Select the correct answer using the code given
for introduction of Dam Safety Bill, 2018 which
below?
establishes a National Dam Safety Authority
(a) 1 and 3
(NDSA). Which of the following statements
(b) 1 and 2
about NDSA is/are correct?
(c) 2 and 3
1. To maintain liaison with the State Dam Safety
(d) 1, 2 and 3 Organisations and the owners of dams for
standardisation of dam safety related data and
Q26. Consider the following statements about practices
Higher Education Commission of India (Repeal 2. It shall maintain a National level Data-Base of all
of University Grants Commission Act) Bill 2018. dams in the country including records of dam
1. It aims to establish Higher Education Commission failures.
of India (HECI) by replacing University Grant 3. It shall accord recognition or accreditations to the
Commission. organisations which works on investigation,
2. It allocates grant function to HECI design or construction of new dams.
3. It grants power of enforcement to HECI Select the correct answer using the code given
Which of the statements given above is/are below?
incorrect? (a) 1 only
(a) 1 and 2 (b) 2 only
(b) 3 only (c) 2 and 3
(c) 1 and 3 (d) 1, 2 and 3
(d) 2 only
Q30. In 2017, government constituted an Expert
Q27. The Department of Personnel and Training has Group under the Chairmanship of former
published invitation of applications for senior Finance Secretary Shri Sumit Bose to
positions in Government of India at the level (a) Define formula for Minimum Support Price for
of different crops
(a) Additional Secretary (b) Devise a formula of disinvestment of Air India
(b) Joint Secretary (c) Identify and prioritize beneficiaries under various
(c) Principal Secretary programme using data of Socio Economic and

(d) Deputy Secretary Caste Census.


(d) Identify challenges regarding slow growth of
manufacturing in India

FOCUS | July 2018 | R AU’S I AS 140


@civilminds

(d) WTO
Q31. Which of the following are correctly matched?
1. Sentosa – South Korea Q37. Which of the following are participants in
2. Hodeidah – Yemen Exercise Malabar – 2018?
Select the correct answer from the codes given 1. India
below: 2. USA
(a) 1 only 3. Japan
(b) 2 only Select the correct answer from the codes given
(c) Both 1 and 2 below:
(d) Neither 1 nor 2 (a) 1 and 2
(b) 1 and 3
Q32. Peace Mission - 2018 is associated with which (c) 1 only
of the following international organisation? (d) 1, 2 and 3
(a) Shanghai Cooperation Organisation
(b) SAARC Q38. Ghouta was recently in the news. It is in which
(c) BIMSTEC of the following countries?
(d) IORA (a) Lebanon
(b) Sudan
Q33. Sabang Island was recently in the news. It is in (c) Syria
which of the following countries? (d) Qatar
(a) Malaysia
(b) Bangladesh Q39. C. Chandrasekharan Task Force is associated
(c) Indonesia with which of the following field?
(d) Myanmar (a) Artificial Intelligence
(b) Border fencing
Q34. Assumption Island was recently in the news. It (c) APMC reforms
is in which of the following countries? (d) Transgender rights
(a) Mauritius
(b) Sri Lanka Q40. Which of the following has the highest
(c) Maldives hydroelectric power potential in India?

(d) Seychelles (a) Arunachal Himalayas


Q35. Which of the following is the implementing (b) Kashmir Himalayas
agency of the Chemical Weapons Convention? (c) Eastern Ghats
(a) Organisation for the Prohibition of Chemical (d) None of the Above
Weapons
(b) UN Security Council Q41. Recently Konark coast of Odisha was accorded
(c) UN General Assembly the ‘Blue Flag, tag. Which of the following
(d) Chemical Weapons Ban Coalition organisations accords the ‘Blue flag’ tag?
(a) UNEP
Q36. Grey Listing is associated with which of the (b) UNFCCC
following International Organisations? (c) Foundation for Environmental education(FEE)
(a) United Nations Security Council (d) Ministry of Environment and Forest
(b) FATF
(c) NATO

FOCUS | July 2018 | R AU’S I AS 141


@civilminds

Q42. ‘Atal BhujalYojana’ will be implemented with (b) Beat the Air pollution
the assistance of (c) Beat the water pollution
(a) Asian Development (d) Beat the Noise pollution
(b) World Bank
(c) Japan International Cooperation Agency (JICA) Q47. Which of the following states has completely
(d) BRICS Bank banned the use of plastics?
(a) Gujrat
Q43. Which of the following are likely benefits of (b) Rajasthan
Zero Budget Natural Farming(ZBNF)? (c) Tamil Nadu
1. Near zero input cost (d) Maharashtra
2. Increased profits
3. Improvement in local biodiversity Q48. Which of the following are related to
Select the correct answer using the code(s) given biodiversity Conservation ?
below? 1. Cartagena protocol
(a) 1 and 2 2. Nagoya protocol
(b) 2 and 3 3. Montreal protocol
(c) 1 and 3 Select the correct answer using the code(s) given
(d) 1,2 and 3 below?
(a) 1 and 2
Q44. Which of the following is a principle of FAO’s (b) 2 and 3
‘Conservation Agriculture’? (c) 3 and 4
1. Zero Tillage (d) 1,2 and 3
2. Permanent Soil Organic cover
3. Increase in plantation crops Q49. Seed Treaty is related to which of the following
Select the correct answer using the code(s) given organisations?
below? (a) UNEP
(a) 1 and 2 (b) UNFCCC
(b) 2 and 3 (c) FAO
(c) 1 and 3 (d) WTO
(d) 1,2 and 3
Q50. Which of the following are Indian Permanent
Q45. Uranium Contamination is associated with stations in Antarctica?
which of the following health risks? 1. Dakshin Gangotri
(a) Cardiac disease 2. Maitri
(b) Renal Failure 3. Bharati
(c) Liver malfunction Select the correct answer using the code(s) given
(d) Psychological problems below?
(a) 1 and 2
Q46. Which of the following is the Theme of World (b) 2 and 3
Environment Day 2018? (c) 1 and 3
(a) Beat the plastic pollution (d) 1,2 and 3

FOCUS | July 2018 | R AU’S I AS 142


@civilminds

1. Answer (c) 13. Answer (d) 25. Answer (d) 37. Answer (d)

2. Answer (b) 14. Answer (d) 26. Answer (d) 38. Answer (c)
3. Answer (b) 15. Answer (a) 27. Answer (b) 39. Answer (a)
4. Answer (c) 16. Answer (c) 28. Answer (d) 40. Answer (a)
5. Answer (a) 17. Answer (d) 29. Answer (d) 41. Answer (c)
6. Answer (a) 18. Answer (b) 30. Answer (c) 42. Answer (b)
7. Answer (c) 19. Answer (d) 31. Answer (b) 43. Answer (d)

8. Answer (a) 20. Answer (a) 32. Answer (a) 44. Answer (a)

9. Answer (c) 21. Answer (b) 33. Answer (c) 45. Answer (b)

10. Answer (a) 22. Answer (d) 34. Answer (d) 46. Answer (a)

11. Answer (d) 23. Answer (a) 35. Answer (a) 47. Answer (d)

12. Answer (d) 24. Answer (c) 36. Answer (b) 48. Answer (a)

49. Answer (c)

50. Answer (b)

FOCUS | July 2018 | R AU’S I AS 143


@civilminds

Here, we are sharing Essay Type Questions for self-


practice. These questions have been framed on
important issues covered in editorials drawn from
Essay Type Questions leading newspapers. A reader is expected to read the
editorals, understand & analyse them, and practice
for mains gs papers I, II & III the question given above each editorial in 200 words
to command on issues relevant for General Studies
Papers- I, II, III & Essay.

Q1: Felling of trees for urban development is an outdated approach. Suggest alternative ways for eco-
friendly sustainable urbanisation.

Delhi disaster Times of India | Environment

In an example of callous urban planning, around 14,000 to 16,000 trees are planned to be felled in Delhi to make
way for government complexes and redevelopment projects. This has sparked outrage among residents, many
of whom are protesting the tree felling exercise in the manner of the famous Chipko movement of the 1970s.
As things stand the city suffers from poor air quality throughout the year. In summer it suffers from
extraordinary heat waves due to the ‘heat island’ effect of dense urban clusters, and it is well known that trees
reduce this effect. Delhi’s massive tree felling exercise is thus a classic instance of unsustainable development,
which even our schoolbooks warn against these days.

Authorities are supposed to plant ten new saplings for every tree cut. However, no such compensatory
plantation has taken place at some of the redevelopment sites with the building authority, NBCC, simply stating
that it has transferred the amount for compensatory afforestation. This is becoming a familiar story across the
country. In Bengaluru, the proposed construction of a steel flyover – which would have led to the felling of more
than 800 trees – was scrapped last year after residents opposed the project.

Similarly, in Mumbai last year environmentalists launched a movement to protect over 3,000 trees from a
Mumbai Metro carshed. All of these show growing urbanisation is leading to a clash between development and
environment. This calls for smart urban planning and adoption of modern approaches. To increase housing and
commercial space, authorities can build upwards rather than horizontally. Better utilisation of existing
government land reserves is in order. Modern urban planning preserves trees given their role in curbing air
pollution, mitigating global warming, reducing ambient temperature and recharging ground water.

Key words: 1. Chipko movement, 2. ‘heat island’ effect

Q2: Demand side solutions won’t work when the problem is a supply constraint. In view of this
statement, give your opinions with respect to National Health Protection Mission.

Transform healthcare Times of India | Health and Social Justice

During his ‘mann ki baat’ broadcast, Prime Minister Narendra Modi highlighted the contribution of doctors to
society. It was timely because, among other things, doctors are crucial to the success of the National Health
Protection Mission aiming to cover 500 million people. Narratives around healthcare solutions in India focus on
the demand side, with emphasis on insurance and price controls. However, this hasn’t yielded satisfactory results
because it cannot offset shortages on the supply side, particularly doctors and hospitals. Healthcare must
reorient itself to address supply side challenges.

FOCUS | July 2018 | R AU’S I AS 144


@civilminds

We have seen in recent times a politically promoted populist backlash against doctors and hospitals, as when the
Delhi government closed a hospital for alleged lapses. This was a disproportionately severe move that denied
healthcare to many. The trick here is to develop sophisticated and light touch regulation of healthcare which
doesn’t inhibit innovation and expansion in the name of curbing malpractices. Price controls, for instance, can
create shortages, shortcuts on quality or overcharging in other areas that are equally necessary. They will
exacerbate rather than solve the supply side problem. Encouraging transparency and competition, on the other
hand, will bring down prices holistically while boosting India’s medical tourism sector, a potential job spinner.

Not only is there a shortage of doctors in India, they rarely venture out of urban areas. This dimension influences
India’s disproportionately high out-of-pocket expense on healthcare of more than 60%. The primary cause for
this shortage is the disappointing performance of Medical Council of India which has exercised control over
licensing of doctors and their ambit. Recognising this lacuna, government has introduced a bill to annul MCI, free
up medical education from stifling controls and also introduce an exit exam at the undergraduate level. However,
one supply side dimension which has been neglected is a pathway for experienced nurses to shoulder more
responsibility for providing care. India needs to use its cadre of experienced nurses to offset supply-side
shortages.

Moreover, gaps in rural India can be made good only by government. The national health policy aims to gradually
increase healthcare spending to 2.5% of GDP. Most of the additional spending needs to be directed to raising
healthcare provision in rural areas as well as for underserved sections of the urban populace. Healthcare, like
other areas, needs a lot more options arising from enhanced supply and competition.

Key words: 1. National Health Protection Mission; 2. Medical Council of India

Q3: Reformist changes in Saudi Arabia for creating gender equality will certainly have a broader
trickle-down effect on many Muslim societies across the world. Comment.

Women drivers Times of India | Social Justice

In Saudi Arabia where women are, at best, treated effectively as legal minors and at worst, treated as servants,
revoking the decades-old ban on women drivers is a historical turning point. The country, which punished dozens
of its women who had defied the religious prohibition on women drivers by firing many of them from
government jobs and by calling all of them ‘immoral’ from the pulpits of mosques, has come a long way.

Though the move for invalidating the ban was announced in September 2017, the law came into effect recently.
Saudi Arabia was the only country left in the world where women could not drive. This progressive step, which is
one of the many that Crown Prince Mohammed bin Salman has been taking as part of his Vision 2030
programme to diversify the economy, will have an impact not just domestically but also on global Sunni Muslim
societies.

Ever since the discovery of oil and the subsequent establishment of a petro-dollar economy Saudi Arabia, as the
birthplace of Islam, wields enormous religious influence on Sunni Muslim societies across the world. But Saudi
Arabia practices and propagates Wahhabism, an extremely conservative school of Islam that ordains death
penalty for adultery by beheading or stoning or crucifixion or firing. It also permits Saudi men to enter
‘temporary marriages’ which are synonymous with sexual slavery.

FOCUS | July 2018 | R AU’S I AS 145


@civilminds

Reformist changes in Saudi Arabia will certainly have a trickle-down effect on many Muslim societies, which have
been struggling with the Wahhabi influence on their native and moderate versions of Islam.

However, Saudi Arabia needs to do a lot more for gender equality, given that its women are still not allowed to
take their own decisions in other spheres of life without the permission of their male guardians, go out in public
without all-enveloping head scarves and cloaks, interact with men or compete in sports freely..

Key Words: 1. Vision 2030 programme, 2. petro-dollar economy, 3. Wahhabism

Q4. Lateral entry is fine but other drastic reforms are required to liberate bureaucracy from colonial
hangover. Suggest other reforms.

Lateral entry Economic Times | Polity and Governance

The government’s move to open up 10 joint secretary-level positions in diverse ministries for lateral entry is
welcome. It needs to be accompanied by a strong institutional mechanism to prevent conflict of interest, that is,
to prevent a revolving door between the government and the private sector from compromising the integrity of
governance. It is desirable to make appointments via the Public Service Commission, rather than via some ad-hoc
committee, which is vulnerable to suspicion of political or lobbying bias in the selection process. Widening the
talent pool from which to select personnel for a government machinery that has to deal with the ever-growing
complexity of globalisation is a good idea, if done right. Henceforth, civil servants must be assigned to a common
pool after some years of service in their specific areas.

Those who fail to qualify for more senior positions from within this pool must be offered an exit route, and the
rest given an opportunity to excel regardless of the service cadre in which they happened to find themselves
after the initial selection process. Lateral hiring is common practice in other democracies such as the UK, US and
Canada. Here, too, however, regulating the revolving door between public office and business remains a work in
progress.

The UK’s Advisory Committee on Business Appointments advises former ministers and senior civil servants on
the propriety of accepting particular positions. Cooling periods for specific kinds of jobs are in order. Greater
scrutiny of the post-government jobs of senior civil servants by both government agencies and the media would
be fair. Self-restraint guided by ethics should inform functionaries. When it does not, there should be
consequences—for lateral entry to work.

Key words: 1. lateral entry

Q5: Video graphing atrocities against Dalits is a further attempt to suppress them and dissuade them
from claiming their right to public spaces and an equal social footing with other communities. Comment.

Not new India Times of India | Social Justice

The recurring incidents of violence against Dalits reported from various parts of the country are a direct
challenge thrown by arbiters of caste hegemony to the law of the land. Many of those committing atrocities
against Dalits know very well that they are in violation of the constitutional right to equality and stringent laws
like the SC/ST (Prevention of Atrocities) Act. Yet they are brandishing these atrocities. The surfacing of two
videos last week illustrates the point – one from Maharashtra where two Dalit boys were beaten and paraded

FOCUS | July 2018 | R AU’S I AS 146


@civilminds

naked for swimming in a village well and another from Gujarat where a 13-year-old was assaulted for dressing
like a “kshatriya”.

The videographing of these heinous acts is clearly calculated to humiliate Dalits and dissuade them from claiming
their right to public spaces and an equal social footing with other communities. However, Dalits are also in no
mood to back away from confrontation. The rise of a new generation of assertive Dalit leaders like Jignesh
Mevani and Chandrashekhar Azad Ravan underlines this trend. Upper caste zealots are clearly fighting a losing
battle as more Dalits embrace modernity and these arch-conservatives hark back to feudal memories.

Interestingly, despite extreme polarisation in politics, all mainstream parties condemn caste oppression. But this
political consciousness must seep into society and civil administration especially police personnel who respond
to crime at the grassroots level. Even as PM Narendra Modi attempts this when invoking his idea of a New India
free from casteism and communalism, BJP’s Dalit MPs are essentially pointing to failures of the sabka saath
sabka vikas slogan. Preventing further atrocities requires strong prosecutions and expeditious punishments.
Beyond this New India requires creating more economic opportunities to flatten persistent caste hierarchies..

Key words: 1. SC/ST (Prevention of Atrocities) Act

Q6. Discuss the role, responsibility and challenges of Cauvery Water Management Authority in solving
inter-state river water dispute.

A good beginning: CWMA The Hindu | Polity & Governance

The first meeting of the Cauvery Water Management Authority (CWMA) took place in a cordial atmosphere
augurs well for a sustained phase of constructive cooperation among the States concerned. The CWMA has been
formed by the Centre to implement the water-sharing award of the Cauvery Water Dispute Tribunal as
modified by the Supreme Court earlier this year. At its meeting, it asked Karnataka to release 31.24
tmcft (thousand million cubic feet) of water in July. The quantum is based on the monthly schedule drawn up by
the Tribunal, and excludes the surplus realeased on the Tamil Nadu side in June. For the Authority to successfully
perform its role, it needs the cooperation of the States in gathering data on rainfall, inflows and outflows,
cropping patterns and periodic withdrawals from reservoirs. The CWMA is expected to meet once every 10 days
during the monsoon months. The south-west monsoon has been active for nearly a month, and is forecast to
be normal this year. Therefore, the CWMA may not face any major problem in overseeing the release of water to
Tamil Nadu. As long as the inflows into Karnataka’s major reservoirs are substantial, it has had no problem
releasing its surplus water into the lower riparian areas of the basin. It is only in a distress year that the CWMA
will face a significant challenge, as determining the extent of distress, and dividing the shortfall among the States
on a pro rata basis can be tricky exercises.

Karnataka is planning to challenge in the Supreme Court the Centre’s notification constituting the Authority. It
will be unfortunate if this dispute gets into another round of litigation. The provisions of the Inter-State River
Water Disputes Act, 1956, make it clear that it is the Centre’s duty to notify a scheme to implement the award of
a Tribunal. Parliament has the power to modify the scheme, or leave it as it stands, but Karnataka’s claim that the
scheme requires parliamentary approval before it is implemented is questionable. Further, the Supreme Court
approved the draft scheme only after finding it to be “in consonance with the dictum and directions in the Award
as modified by this Court and also in conformity with Section 6A of the 1956 Act”. Now that the CWMA has
become functional, Karnataka, Tamil Nadu, Kerala and Puducherry should approach the issue of sharing the
waters of the inter-State river in a spirit of cooperation and help the Authority in implementing the verdict. The

FOCUS | July 2018 | R AU’S I AS 147


@civilminds

parties concerned should leave behind the era of litigation. There is now a non-political mechanism available to
make sound professional decisions on water availability and sharing of distress, if any, after discussing the issues
threadbare. After having been locked in a contentious legal dispute for so long, all parties concerned must
embark on a new era of mutually beneficial water-sharing.

Key words: 1. Cauvery Water Management Authority 2. water-sharing award 3. Cauvery Water Dispute Tribunal 4.
south-west monsoon 5. Inter-State River Water Disputes Act, 1956

Q7. It is the responsibility of Election Commission of India to ensure free, fair and transparent election
in India. In light of the above statement discuss the challenges regarding conduct of elections in India.
Suggest suitable measures to overcome the said challenges.

Paper Chase: need for a review of VVPAT The Hindu | Polity & Governance

The high incidence of glitches in the Voter Verifiable Paper Audit Trail (VVPAT) machines in recent by-elections
should be a major cause of concern for the Election Commission of India (ECI). Fresh polling had to be ordered
in dozens of booths in Kairana and Bhandara-Gondiya in Uttar Pradesh and Maharashtra, respectively, as a
consequence. Ever since the implementation of the VVPAT system last year, machine malfunction and
subsequent delays in polling have been recurring issues. Close to 4.2% of the VVPAT machines deployed in the
Karnataka Assembly elections this month developed glitches during the testing as well as polling processes.
The overall fault rate was as high as 11.6% in the by-elections held in four parliamentary and nine Assembly
constituencies. The ECI has suggested that these machines were more prone to malfunctioning due to their
sensitivity to extreme weather conditions and exposure to light. It also blamed the relative inexperience of
polling officers handling them, compared to the ballot and control units for the electronic voting machines
(EVMs) that have been in use for much longer. The technical committee of the ECI is now faced with a challenge
to ensure that the VVPAT machines hold up, with the general election due next year in the hot summer months.
The VVPAT was added to the EVM to audit the voter tallies stored in the machine. Its universal implementation
which began in the Goa Assembly polls in February 2017 was deemed necessary as many political parties
complained about the possible hacking of EVMs. These complaints lacked any basis, but the VVPAT
implementation was hastened to bring back trust in the election process. In all elections where it has been used,
the VVPAT tallies have matched with the EVM counts, but for a stray case or two when the VVPAT machine was
not reset before polling began.

Inadvertently, the use of these machines, which are adjuncts to the ballot and control units of the EVMs, has
added to the complexity of an otherwise simple, single programmable-chip based system, and rendered it prone
to more glitches. There is enough empirical evidence to show that EVMs have eased polling and helped increase
voter turnout since being put to use. But in using VVPAT machines to reassure sceptics about an election’s
integrity, the ECI has introduced a new element, and cost, to the process. Considering these challenges, the ECI
should consider deploying the VVPAT machines in a limited, statistically significant, randomly chosen set of
polling booths. This will reduce the possibility of glitches affecting the polling process as well-tested machines
could be deployed (with enough replacements also handy) to such booths. The current verification process, after
all, only involves the counting of VVPAT slips by randomly choosing one booth from each constituency (or
segment), and this check should not be affected drastically by the new method.

Key words: 1. Voter Verifiable Paper Audit Trail 2. Election Commission of India 3. machine malfunction 4. delays in
polling 5. Karnataka Assembly elections 6. inexperience of polling officers

FOCUS | July 2018 | R AU’S I AS 148


@civilminds

Q8. The quality of higher education in India requires major improvement to make it internationally
competitive. Do you think that replacing University Grant Commission by Higher Education Commission
of India would help improve the quality of technical and higher education in the country. Discuss.

An over regulator Indian Express | Polity & Governance

It wouldn’t be an overstatement to say that India’s higher education sector is in desperate need of reform. A
number of indicators, including the consistently poor performance of Indian universities at the World
University Rankings, testify to the fact that the country’s higher education regulator, the University Grants
Commission (UGC), has not lived up to its mandate of “maintaining standards of teaching, examination and
research in universities”. A new draft law, the Higher Education Commission of India Bill, that proposes to
revamp the governance of higher education in India, could, then, signal better days for the country’s universities.
But the draft that was put up for public comments could, instead, aggravate the failings of the UGC, particularly
its propensity to over-regulate.

The Bill proposes to replace the UGC Act, 1956, and rechristen the UGC as the Higher Education Commission of
India (HECI). The regulator, in its new avatar, will focus on setting, maintaining and improving academic
standards in universities. The Union Ministry of Human Resource Development will take over the grant-giving
functions. The separation between the regulator and the funder is in tune with the first principles of regulatory
governance. The professed goal of the draft law, “autonomy for universities,” is also welcome. However, making
the HRD ministry the fund dispersal agency strikes against this objective. It is true that academic institutions in
the country have never been completely free from government interference. But with the HRD ministry
controlling university funding directly, the dangers of political interference in the running of these institutions
increase manifold. The provision goes against the credo of universities as self-regulating spaces where decisions
are shaped by intellectual debates — not funding or political pressures.

The UGC, a body of academics and experts, was envisioned as a “buffer” between the government and higher
education institutions. However, interference by successive governments stood in the way of the agency fulfilling
this objective. At the same time, the regulator remained a spectator to the falling standards of university
education. The proposed new regulator, the HECI, intends to bridge this lacuna. However, its mandate of
“improving academic standards with a specific focus on learning outcomes, evaluation of academic performance
by institutions and training of teachers,” is bound to raise fears about the regulator micro-managing universities.
And the fact that the proposed law empowers the Centre to remove the HECI’s chairman and vice-chairman for
reasons that include “moral turpitude” — the UGC act did not have such a provision — will raise questions about
the government’s sincerity on giving autonomy to universities. The draft is up for public comments till July 7.
Enforcing this deadline might enable the government to place the Bill before the Parliament in its monsoon
session. But it will be well-advised not to rush through a piece of legislation that aims to fundamentally
restructure higher education.

Key words: 1. higher education 2. poor performance 3. World University Rankings 4. University Grants Commission 5.
Higher Education Commission of India 6. regulatory governance 7. political interference

Q9. The role of individual MPs (Members of Parliament) has diminished over the years and as a result
healthy constructive debates on policy issues are not usually witnessed. How far can this be attributed to
the anti-defection law, which was legislated but with a different intention?

FOCUS | July 2018 | R AU’S I AS 149


@civilminds

Let it play: decline in Parliamentary standards Indian Express | Polity & Governance

Rajya Sabha Deputy Chairman PJ Kurien wants live broadcast of proceedings in Parliament to be stopped
since it could show the House in poor light. Kurien’s concern for the image of the House is understandable. But
with due respect, the causes of falling standards of parliamentary conduct and the cures for it lie elsewhere.
Kurien, who retires on July 2 after two terms in the Rajya Sabha, argues that the live broadcast of proceedings
has resulted in “so many unparliamentary words and unfounded criticisms” beamed into public space while the
Chair may have expunged them from House records. His contention is that the Chair’s efforts to ensure propriety
and order in the conduct of Parliament is thereby rendered meaningless. This may well be the case. At the same
time, however, it sounds suspiciously like the messenger is being blamed.

In the past, people have blamed live broadcast of proceedings more directly for the ungainly conduct of MPs in
the House. It has been argued that cameras induce parliamentarians to engage in rhetoric and showmanship
instead of reasoned debate. But the reasons for the decline of Parliament as a forum of debate are more
complex, go much deeper: They have more to do with the rise of personality-centric politics, the breakdown of
dialogue, and even civility, between the government and the Opposition, which Kurien refers to, for instance.
These changes, caused by the privileging of political mobilisations centered on polarizing issues and
demagoguery, have resulted in a general weakening of institutions, of which Parliament is an important part. The
hardening of exclusivist collective identities — religious, caste, ethnic — have also transformed electoral politics,
in favour of candidates who can mobilise resources, including money and muscle power. Today’s
parliamentarians may be more representative of the country’s class and caste diversity, but they also hold up a
mirror to the general indifference to, and neglect of the task of nurturing robust traditions of debate.

A first step towards restoring the institutional primacy of Parliament could be for the government to reach out, to
build cordial relations with the Opposition and ensure the smooth conduct of House sessions. As Kurien says,
“the government should accept that the Opposition has a right to criticise them” and the “the Opposition should
accept that the government’s job is to run the country”. On the eve of another parliamentary session,
introspection is needed on how to rebuild the House, across the aisle.

Key words: 1. live broadcast 2. proceedings in Parliament 3. falling standards 4. parliamentary conduct 5. personality-
centric politics 6. regulatory governance 7. political interference

Q10. Instances of data security have been a growing concern in India considering the rise in internet
users. In this backdrop do you think giving access of Aadhaar details for police investigations is a logical
idea? Give reasons.

Police and Aadhaar Times of India | Polity & Governance

The National Crime Records Bureau (NCRB) director’s suggestion that police be given limited access to
Aadhaar data to help them trace first-time offenders and unidentified bodies sounds logical. After all nearly
50 lakh cases are registered across the country every year with around 80-85% of the offenders being first-timers
without prior police records. Theoretically access to Aadhaar to verify fingerprints in such cases can aid
investigation and prosecution. Similarly more than 40,000 unidentified bodies are found every year and again
Aadhaar verification could help in handing these over to relatives.

That said, the Supreme Court is hearing a raft of petitions challenging the constitutional validity of Aadhaar,
including on grounds that it violates right to privacy. And this puts expanding the application of Aadhaar to new
areas on a sticky wicket. Plus, while Aadhaar’s original utility was in facilitating social welfare schemes through
direct benefit transfers, it’s increasingly become a security scheme. In this new orientation, ironically the more
Aadhaar is used without adequate safeguards, including strong data protection legislation, the more susceptible

FOCUS | July 2018 | R AU’S I AS 150


@civilminds

it becomes to misuse. And given the high value attached to Aadhaar as an identity document, the scope of
misuse can be catastrophic for victims.

Against this backdrop, before giving police access to Aadhaar there’s a strong case for upgrading investigation
systems. The NCRB director himself has stressed that of the 50 lakh cases registered every year, fingerprint
experts are able to visit only around 55,000 crime scenes. Many states are in crying need of forensic resources.
These are basic systemic issues which need to be fixed first. Without this access to Aadhaar will at best be
meaningless for investigation and at worst it will make a bad situation worse by incentivising shortcuts. A new
technology is only as good as the system operating it.

Key words: 1. National Crime Records Bureau 2. limited access 3. Aadhaar data 4. first-time offenders 5. unidentified
bodies 6. verify fingerprints 7. social welfare schemes 8. direct benefit transfers

Q11: Do you think recent hike in Minimum Support Prices (MSPs) will change the ground situation in
agriculture? Give reasons in favour of your argument.

Old Hobby Horse Times of India | Economy

Ask any politician what must be done to address India’s various socio-economic problems and he is likely to
have a repertoire of stock answers: hike in minimum support prices (MSPs), loan waivers, more reservations,
death penalty. The NDA government didn’t deviate from this tired routine when it fixed MSP for 14 major kharif
season crops at 150% of the cost of cultivation. This translates into a 13% increase in the weighted average of
MSPs, the most generous hike announced by NDA in the last four years. In terms of individual commodities, MSP
of paddy, the kharif season’s main cereal, was enhanced by 11.32%. In relative terms, a coarse cereal bajra
recorded an MSP that provides a return of 96.97%.

Kharif procurement is dominated by paddy, but almost two-third of the procurement is from just five states led
by Punjab. In a significant number of rice growing areas the wholesale price of paddy is often lower than MSP.
Therefore, unless the procurement machinery is able to significantly expand its ambit, the MSP increase may not
really impact a majority of paddy farmers. However, in terms of optics BJP has delivered on a budget promise
after a period when agricultural commodity prices were almost stagnant.

On the economic side, a significant increase in MSP induces uncertainty in India’s inflation trajectory. A lot,
however, will depend on movement of oil prices, rupee’s exchange rate and the determination of the Narendra
Modi government to hold the line on fiscal deficit. A sudden spurt in MSPs also introduces other economic
distortions. It can influence cropping patterns at a time when there is abundance of paddy in government’s
granaries. Linked to it is the adverse impact on water tables in some of the most water stressed parts of India.

MSPs are no longer a potent tool to deal with agrarian challenges and are at best a Band-Aid. A better approach
to tackling agrarian woes is through a combination of pro-market reforms and providing direct investment
support to farmers in a crop neutral way. Telangana is attempting the latter now and BJP-ruled states should
follow suit. This will provide for far more impact per rupee invested than wasteful procurement operations. The
saving could be used to enhance investment in infrastructure and lower risk of farm households which today
fork out almost 80% of investment. More than MSPs, agriculture today is in dire need of reforms.

Key words: 1. Socio-economic problem 2. minimum support price 3. loan waivers4. procurement 5. agrarian
challenges

Q12: The new biofuels policy ticks all the right boxes but execution holds the key. Comment.

FOCUS | July 2018 | R AU’S I AS 151


@civilminds

Green Signal Business Line | Economy

With crude oil prices now hovering around the $80/barrel mark, India’s new National Policy on Biofuel,
announced recently, could not have been timed better. Nobody can argue with the obvious benefits of the
biofuels policy at least in terms of intent. The policy significantly expands the sources from which ethanol can be
produced and also allows for blending ethanol from multiple bio feedstock — and not just ethanol produced
from sugarcane, as was the case earlier — with petrol. The policy also holds significant potential gains for
farmers. Under the new policy, the raw material sourcing net for ethanol is being significantly expanded to
include, besides sugarcane juice, sugar beet, sweet sorghum, corn, cassava, damaged wheat, broken rice
and rotten potatoes. There is also a bonus for civic administrations, since ethanol can now be produced from
municipal solid waste, as well as fuels like bio natural gas. To that extent, the policy can help turn refuse into
gold. The policy also envisages using rice and wheat chaff and other biomasses as feedstock for ethanol
production. This opens up a large and remunerative market for products on which the farmer was bearing the
losses caused by erratic weather and poor storage conditions. Beyond this, used cooking oil can also double up
as a feedstock for biodiesel which is welcome since India is one of the world’s largest consumers of edible oil and
generates a huge quantity of used cooking oil as waste.

While there is no critiquing the intentions, execution will be a challenge. To convert India’s existing biofuel
potential into reality, huge investments need to be made in creating bio refinery capacity. However, this is easier
said than done. While state-owned oil marketing companies are in the process of setting up 12 bio-refineries, this
can only be a base to build on. On the ground, private sector investment in this space has been hampered by
financial constraints and lack of cohesive support from the Central to the local level. Creating the requisite
supply chain infrastructure to source and efficiently transport low value biomass to the refineries is another
challenge. The Centre should ensure that it actively involves the private sector in this exercise especially for
functions like procurement, storage and distribution. It should steer clear of micromanaging the supply chain
but, instead, help in land acquisition for the bio-refineries and working with the stakeholders to fix a reasonable
price for the end product.

Finally, the policy needs to be followed up with coordinated action at the user end to ensure that the larger goal
of the policy — of cleaning up the air, reducing the carbon footprint and shift to more sustainable renewable
fuels — is not lost sight of. From encouraging the use of biofuels in public transport to ensuring that civic bodies
actually realise the potential of municipal waste and sewage the policy needs to be implemented in mission
mode on a nationwide basis.

Key words: 1. National Policy on Biofuel 2. ethanol 3. Investment 4. supply chain infrastructure 5. biomass

Q13: “The Centre’s draft passenger charter will help both passengers and the industry.” In light of the
statement, critically examine the Civil Aviation Ministry’s draft passenger charter.

Welcome move Business Line | Economy

The Civil Aviation Ministry’s draft passenger charter has been a long time coming. Given the NDA government’s
proactive role in trying to bring some order in an industry where chaos is the norm, such a measure was
expected sooner. The charter has several positives especially those relating to compensation for passengers for
delays and cancellations. Earlier, the only way for redress was to make a beeline to the consumer courts. A
cursory reading of the charter may suggest that the Ministry wants to come down hard on delinquent airlines,
but what it primarily does is bring in efficiency in the running of operations. In effect, it forces airlines to become
more responsible, which will eventually benefit them and help in expanding their band of loyal customers. These
draft norms are not out of the ordinary. Most countries, especially in the US and those in Europe have similar

FOCUS | July 2018 | R AU’S I AS 152


@civilminds

ones. The only difference is that the norms laid out by the European Union and the US Department of
Transportation for their respective airlines were brought into force much earlier and since then, these have
been repeatedly tweaked to conform to the changing landscape of the travel industry. Both these agencies
have also ensured that the norms are implemented strictly without which such regulations remain only on paper.

However, there are a few draft rules in the charter which could be a cause for concern. The one on cancellations
could hit the business of travel websites hard. A couple of airlines have pointed out that passengers will receive a
full refund only if they book on airline websites and their apps and not on travel booking platforms. This is
because travel agents use the global distribution system (GDS) to book tickets for their customers and airlines
incur high GDS fee for every booking and cancellation. But the fee is not refunded to the airlines in case of
cancellations made on the travel platforms. The Ministry needs to clarify on this aspect if the travel websites are
to remain in business. Another such proposed rule is on providing affordable food and beverage outlets at
airports. Airlines have little or no control over this, since the licences for setting up such outlets are given out by
the airports based on those who bid the highest. Hence, the food served there is also priced higher.
Nevertheless, it is time all stakeholders realised that flying is no longer a preserve of the elite and that it has
become the common man’s preferred mode of travel.

The only way the airlines can adopt some of these norms is by passing these costs to the passengers and this can
only lead to a higher cost of flying, which was probably not what the Ministry intended. Hopefully, the 30-day
window for consultations among all the stakeholders will iron out these issues before the norms are formally
notified.

Key words: 1. consumer courts 2. Transportation 3. travel industry 4. global distribution system

Q14: What are the factors that contributed to faster pace of poverty reduction in India in the last two
decades? Discuss

Growth rocks Times of India | Economy

India has held, for the longest possible time, the unenviable record of having the largest number of people living
in extreme poverty. Recent research suggests that’s no longer the case, and if current trends persist the
absolute number of Indians in extreme poverty – defined as an income below $1.90 per person per day – will
also drop sharply. That, of course, would be reason to celebrate. The aim should also be to quickly enhance the
living standard of large numbers who remain economically vulnerable. If all this is to be realised, policy makers
need to heed lessons from domestic as well as global experience. Poverty has declined since the 1970s but the
watershed was 1991-92. Since then poverty not only reduced at a faster pace, urban growth has emerged as a
significant influence in reducing rural poverty. A couple of conclusions are inescapable. Economic growth
matters. It influences poverty reduction through two channels. One, it increases the purchasing power of large
swathes of population. Two, it provides governments with additional revenue to make strategic interventions.
The rapid expansion of India’s social safety nets over the last two decades is an outcome of economic growth.

Another positive phenomenon benefiting India has been globalisation. A fact seldom acknowledged is that
globalisation has decreased global inequality, particularly in the last decade. Poverty declines in India and China
have contributed substantially to this trend. In this context, it is in India’s interest to do all that it can to preserve
a system which created the best living standards we have ever experienced. The primary vehicle to raise overall
living standards will remain economic growth. In an evolving situation where globalisation is under attack, policy
has to focus on unshackling wellsprings of growth at home. It is self-defeating, for instance, to shackle the farm
economy with laws restricting cattle trade and slaughter or retain an overbearing regulatory structure for
business. A lesson from China’s success needs to be emulated. Around 40 years ago, China was overwhelmingly

FOCUS | July 2018 | R AU’S I AS 153


@civilminds

poor. If every percentage point of growth in China has lifted more people out of poverty it is partly because
people were better equipped to tap opportunities. Put simply, education will be the key to India’s future
economic growth and well-being. Government needs to get its education policy right.

Key words: 1. poverty 2. living standard 3. urban growth 4. purchasing power 5. social safety6. globalisation 7.
economic growth 8. regulatory structure

Q15: “The focus must shift from compensating farmers to freeing them from rules and structures that
depress farm gate prices.” In light of the statement discuss the need to have long term policy in
agriculture to reduce farm distress.

Unshackle farmers Times of India | Economy

The impact of a 10-day-long farmers’ agitation is being felt in several parts of the country. With farmers
threatening to block dairy and vegetable supplies it is not just governments but residents in cities who will also
feel the heat. The protesters are demanding complete loan waivers, assured income for farmers, and higher
minimum support prices. But while such measures can be a political salve that addresses the immediate
problems of some farmers, these cannot free them from being perpetually at the mercy of governments,
middlemen and moneylenders. It is licence raj era regulations tailored for foodgrain security that have truly
broken the farmers. So the state must bite the bullet and remove regulations that prevent farmers from
accessing larger markets and restrict export. Next, help farmers by facilitating long-term purchase agreements
with agro companies that can set up warehouses and cold chains. It is also time to recognise that MSP and loan
waiver schemes are not reaching those farmers who face repeated price shocks and are in the grip of a trader
mafia.

Dismantling of APMC raj has begun in states like Maharashtra and Bihar but the benefits will become evident
only when all states follow suit and markets are integrated. Many of these measures are politically difficult and
will face resistance from traders and Left groups who will dub it as selling out to corporate interests. Even
Madhya Pradesh’s innovative Bhavantaryojana to pay farmers the differential between MSP and mandi rates
without the state getting into the logistics of procurement has failed during price crashes, a reason for the
renewed protests this year. The focus must shift from compensating farmers to freeing them from rules and
structures that depress farmgate prices.

Key words: 1.loan waivers 2. assured income 3. minimum support price 4. moneylenders5. foodgrain security 6. APMC
7. Bhavantaryojana 8. Procurement 9.farmgate prices

Q16. Uranium contamination of groundwater has emerged as serious issue in many states of India.
What are the reasons for this contamination? Highlight the major gaps in Water- quality monitoring in
India.

Tainted By Uranium: On groundwater contamination The Hindu | Environment

Reports of widespread uranium contamination in groundwater across India demand an urgent response. A
study, published in Environmental Science and Technology Letters, has found over 30 micrograms per litre (mcg/l) of
the heavy metal in parts of northwestern, southern and southeastern India. Drinking such water can damage
one’s kidneys, and the World Health Organization prescribes 30 mcg/l as an upper limit. Unfortunately, the
residents of the regions surveyed were using the contaminated wells as their main source of drinking water.
These findings highlight a major gap in India’s water-quality monitoring. As the Bureau of Indian Standards

FOCUS | July 2018 | R AU’S I AS 154


@civilminds

does not specify a norm for uranium level, water is not tested regularly for it. This is despite the fact that
evidence of uranium contamination has accumulated from across India over the last decade. A 2015 Bangalore
study, for example, found uranium levels of over 2000 mcg/l in the southern part of the city. Other studies found
levels of over 500 mcg/l in Andhra Pradesh and Telangana. The Environmental Science paper adds new data to this
body of evidence by sampling wells in Rajasthan and Gujarat.

The health effects of drinking uranium-tainted water merit special attention. A few small animal and human
studies have found that the heavy metal damages the kidneys. The studies indicate that this is a chemical effect,
rather than a radiological one, even though uranium is radioactive. But the chronic effects of uranium
consumption are still unknown. Could there be, for example, a link between the high rates of chronic kidney
disease (CKD) in India and uranium exposure? In a survey conducted between 2005 and 2010, an Indian registry
found 8,385 CKD cases with no known cause. One cluster of mystery disease, located in Srikakulam district in
Andhra Pradesh, has stumped epidemiologists for years. It is impossible to say if these clusters have anything to
do with groundwater contamination, unless researchers look at it systematically. Another critical area of
research is the mechanism by which uranium enters groundwater. The Environmental Science paper identified
two types of terrains with heavy contamination. In Rajasthan and other northwestern regions, uranium occurs
mostly in alluvial aquifers; while in southern regions such as Telangana, crystalline rocks such as granite seem to
be the source. When groundwater is over-extracted from such soils, the researchers suggest, the uranium is
exposed to air, triggering its release. These hypotheses must be explored, because they will help determine
where to find safer water. This is what happened in West Bengal, where a decade of research revealed why the
contaminant arsenic mainly occurred in shallow aquifers. Researchers found that a combination of geological
and chemical triggers brought arsenic to the Ganga delta in the Holocene era, and then released it into the
sediments from that period. Similar research across India’s uranium hotspots can uncover who is at risk, and
how to protect them.

Key words: 1.Groundwater contamination 2.Health Effects 3.Bureau of Indian Standards4.Pollution 5. Over Extraction

Q17.What are initiatives that have been undertaken to tackle the plastic pollution? Highlight the key
issues with India’s approach in handling plastic waste. Suggest measures to improve the plastic waste
management system.

Reduce, Segregate: On Plastic Ban The Hindu | Environment

Maharashtra’s ban on several consumer articles made of plastic, introduced after a three-month notice period to
industry and users, is an extreme measure. It is naturally disruptive, and Mumbai, famed for its resilience in the
face of urban challenges, is trying to adapt quickly. Today, stemming the plastic tide is a national imperative.
India hosted this year’s World Environment Day and Prime Minister Narendra Modi made a high-profile pledge,
to international acclaim, that it would do away with all single-use plastics by 2022. This goal is not yet backed by
an action plan so that State governments and local bodies can be in sync. Worldwide, the problem has got out of
hand, with only 9% of about nine billion tonnes of plastic produced getting recycled. India has an uninspiring
record when it comes to handling waste. It has patchy data on volumes, and even less on what it recycles. This
lackadaisical approach is at odds with its ambitious goals. Quite simply, if the Centre and the States had got down
to dealing with the existing regulations on plastic waste management and municipal solid waste, a ban would
not even have become necessary. Specifications for the recycling of different types of plastics were issued two
decades ago by the Bureau of Indian Standards.

To address the global concern that the bulk of India’s plastic waste — estimated officially at 26,000 tonnes a day
— is being dumped in the oceans, there has to be an effort on a war footing to segregate it at source. The Urban
Development Secretary in each State, who heads the monitoring committee under the rules, should be

FOCUS | July 2018 | R AU’S I AS 155


@civilminds

mandated to produce a monthly report on how much plastic waste is collected, including details of the types of
chemicals involved, and the disposal methods. Such compulsory disclosure norms will maintain public pressure
on the authorities, including the State Pollution Control Boards. But segregation at source has not taken off, as
there is little awareness, official support or infrastructure. Even bulk generators such as shopping malls, hotels
and offices do not abide by the law. Priority, therefore, should be given to stop the generation of mixed waste,
which prevents recovery of plastics. Companies covered by extended producer responsibility provisions must be
required to take back their waste. In parallel, incentives to reduce the use of plastic carry bags, single-use cups,
plates and cutlery must be in place. Retailers must be required to switch to paper bags. Potentially, carry bag
production using cloth can create more jobs than machines using plastic pellets. What needs to be underscored
is that plastics became popular because they are inexpensive, can be easily produced and offer great
convenience. But, as the UN Environment Programme notes, their wild popularity has turned them into a
scourge. Consumers will be ready to make the switch, but they need good alternatives.

Key words: 1.Plastic Ban 2.World Environment Day3.Single use plastics 4.Plastic Waste Management5. United Nation
Environment Programme.

Q18. Urgent reforms are needed to deal with the acute water stress in most parts of India. In the light
of this statement explain the causes of water stress in various parts of India as highlighted by the
NitiAyog report. Suggest measures to improve the present situation.

Parched or Polluted: on India’s Water crisis The Hindu | Geography-Resources

India’s water crisis is clear and present, with implications for the health of the entire population. According to
the Composite Water Management Index developed by Niti Aayog, 70% of the water resources are identified
as polluted. This is based primarily on data supplied by States for calculating the index. If the water accessible to
millions is contaminated, the problem is infinitely worse than that of availability. The system of ratings for States
is based on their performance in augmenting water resources and watersheds, investing in infrastructure,
providing rural and urban drinking water, and encouraging efficient agricultural use. It presumes that this ‘hall of
fame’ approach will foster “competitive and cooperative federalism”. What emerges from the early
assessment is that States such as Gujarat, Madhya Pradesh, Andhra Pradesh, Karnataka, Maharashtra, Punjab
and Telangana have initiated reforms for judicious water use, while populous ones such as Uttar Pradesh and
Bihar have failed to respond to the challenge. Tamil Nadu, which has a middling score, does well on
augmentation of water sources, but is abysmally poor in ensuring sustainable use for farming. The trends that
the data reflect of high to extreme stress faced by 600 million people call for speedy reforms.

Two areas that need urgent measures are augmentation of watersheds that can store more good water, for use
in agriculture and to serve habitations, and strict pollution control enforcement. In this context, the Committee
on Restructuring the Central Water Commission and the Central Ground Water Board, chaired by Mihir Shah, has
called for a user-centric approach to water management, especially in agriculture. It advocates decentralisation
of irrigation commands, offering higher financial flows to well-performing States through a National Irrigation
Management Fund. Clearly, awarding an index rank should help advance such schemes, making States feel the
need to be competitive. Yet, such approaches may not resolve seemingly intractable inter-State river disputes.
As the Cauvery issue has demonstrated, State governments would rather seek judicial intervention than be
accused of bartering away the rights to a precious resource under a shared, cooperative framework.
Groundwater extraction patterns need to be better understood through robust data collection; less than 5% of
about 12 million wells are now under study. Steady urbanisation calls for a new management paradigm,
augmenting sources of clean drinking water supply and treatment technologies that will encourage reuse.
Pollution can be curbed by levying suitable costs. These forward-looking changes would need revamped national

FOCUS | July 2018 | R AU’S I AS 156


@civilminds

and State institutions, and updated laws. A legal mandate will work better than just competition and cooperation;
it would make governments accountable.

Key words: 1.Water Crisis, 2.Composite Water Management Index, 3.Cooperative and competitive Federalism,
4.National irrigation management fund, 5.Inter-state river disputes. .

Q19. The New Bio-fuels policy tries to address the supply side issues, However it fails to address the
technological and financial feasibility of such initiatives. Comment.

Policy of Biofuels: Green Push ? The Hindu | Geography-Resources

At a time when rising oil prices are putting increasing pressure on the economy, even small steps to encourage
the use of biofuels are welcome. The Cabinet this month approved a National Policy on Biofuels, which
encourages the generation and use of biofuels such as ethanol. It primarily tries to address supply-side issues
that have discouraged the production of biofuels within the country. For one, it allows for a wider variety of raw
materials to be used as inputs to produce ethanol that is blended with petrol. Until now, only ethanol produced
from sugarcane was approved for this purpose. Under the new policy, feedstock for biofuels includes sugar
beet, corn, damaged foodgrain, potatoes, even municipal solid waste. This will likely reduce the cost of producing
biofuels and improve affordability for consumers, particularly during times when oil prices reach discomforting
levels. In India, industrial-scale availability of ethanol so far has been only from sugar factories, which were free
to divert it to other users such as alcohol producers, who would pay more. The oil companies have been floating
tenders for ethanol supply, but availability lags behind their needs, because the price is often not attractive
enough for the sugar industry. The Centre hopes the new policy will also benefit farmers, who will be able to sell
various types of agricultural waste to industry at remunerative prices. But given the technology available, a large
chunk of the biofuel will have to come from the sugar sector for now. Therefore, pricing is the key. The
government estimates that ethanol supply of around 150 crore litres in 2017-18 could save foreign exchange
worth over Rs. 4,000 crore. The production of biofuels from agricultural waste, it is hoped, will also help curb
atmospheric pollution by giving farmers an incentive not to burn it, as is happening in large parts of northern
India.

But policy should not get ahead of technological and financial feasibility — and options should be realistically laid
out for farmers. There is also a need for caution in using surplus foodgrain to produce ethanol. And while
removing the shackles on raw material supply can have definite benefits, it cannot make a significant difference
to biofuel production as long as the supply-chain infrastructure that is required to deliver biofuels to the final
consumer remains inadequate. To address this issue, the new policy envisages investment to the tune of 5,000
crore in building bio-refineries and offering other incentives over the next few years. The government should
also take steps to remove policy barriers that have discouraged private investment in building supply chains.
Until that happens, India’s huge biofuel potential will continue to remain largely untapped.

Key words: 1.Biofuel2.Feedstock for biofuels3.Supply-chain infrastructure4.Bio-refineries 5.Ethanol.

Q20. Protecting Environment should not just be the government’s business alone. Critically examine
the statement in the light of various initiatives taken by the government to tackle Climate change.

Green Good Deed for the day Indian Express | Environment

India has been attracting adverse comments on issues related to the environment — for valid as well as wrong
reasons. The country has received bad press for its failure to check air pollution. It has also been said that

FOCUS | July 2018 | R AU’S I AS 157


@civilminds

environmental degradation costs India $80 billion per year or roughly 5.7 per cent of its economy.
Disadvantaged groups are more vulnerable to climate change.

India has a healthy tradition of environment protection. The country has committed to using 175 GW of
renewable energy by 2022. However, India has development imperatives. While the sustainable development
path chosen by the country shows that economic growth and environment protection can go hand-in-hand, the
world also needs to pay heed to Prime Minister Narendra Modi’s call for “climate justice”.

India’s Nationally Determined Contribution (NDC), finalised before the Paris Climate Agreement, aims to reduce
the emissions intensity of its GDP by 33 to 35 per cent by 2030. India also aims to use 40 per cent of its
cumulative installed electricity capacity from non-fossil fuel based energy resources by 2030. The country has
committed to an additional carbon sink of 2.5-3 billion tonnes of carbon dioxide equivalent. According to several
international evaluations, India is on course to achieving the pre-2020 goal of reducing the emission intensity of
its GDP by 20 to 25 per cent over 2005 levels.

However, building on the gains of the past few years will require a concerted effort from the government and
civil society. Governments may lay down norms but policing and penalising people cannot be the basis of a
robust model of environmental protection. Protecting the environment should not be the government’s business
alone.

The need of the hour is not just for people to act responsibly but also to take responsibility for others, report
misdemeanours not just of individuals but also of institutions and corporates. In other words, the environmental
credo of the country should be: “Do your own green good deeds and ensure others do that as well.” Take, for
example, Delhi’s air pollution. Dust and vehicular emissions are among the major causes for the foul air over
the city. These can only be controlled by a combined effort of civic authorities, resident welfare associations,
individual citizens and the government machinery.

This is not to say that the government has no role in mitigating pollution. The environment ministry has taken up
air pollution on mission mode. In January, the government notified amendments to the Environment (Protection)
Rules 1986 making it mandatory for construction agencies to take dust mitigation measures. The government
has written to major construction agencies like DMRC, NBCC and NHAI to comply with these rules. The
amendments lay down: “No building or infrastructure project requiring environmental clearance shall be
implemented without an approved Environmental Management Plan, inclusive of dust mitigation measures.” The
rules are applicable in all cities where the PM 10 and PM 2.5 levels exceed the limits set in the National Ambient
Air Quality standards. The annual PM 10 limit is 60 micrograms per metre cube. For PM 2.5, the standard is 40
micrograms per metre cube. As I write this article, work on 25 construction sites faces closure. Several
government schemes, including the Pradhan MantriUjjawalaYojana, will also help to improve the country’s air
quality.

The Sustainable Development Goals talk of responsible consumption and production. They also underline the
need for urgent action to combat climate change. That responsibility and urgency has to be shared. Tough
actions means that some will suffer — but the end justifies the means. The end here is the greater common
good. The bottom line is that we are in it together.

Key words: 1.Environment Degradation2.Civil Society3.Climate Change4.Air pollution5.PM-2.5 & PM-10.

Q21. Assess India’s geo-strategic ascent into the Indo-Pacific region.

Securing the littoral Indian Express | India & the World

The Indo-Pacific as a geopolitical construct is a proposition that India is billed as vital to the ascent of India in
world stage Indo-Pacific. India’s new willingness to match the rhetoric with concrete actions. Securing the eastern

FOCUS | July 2018 | R AU’S I AS 158


@civilminds

Indian Ocean in partnership with Southeast Asian littorals like Singapore, Indonesia and Thailand could be one of
the important near-term Indian contributions to peace and stability in the Indo-Pacific.
US has also announced that it was changing the name of the US Pacific Command into the Indo-Pacific
Command. Addressing the conference, Mattis emphasised that this was a recognition of the changing regional
reality and the critical role of India in shaping the future of this region. The region that has long called itself the
“Asia Pacific” was surprised by the American use of the term “Indo-Pacific” Indo-Pacific “a natural region” that
stretches from the east coast of Africa to the west coast of America. He called the building of a “stable, secure
and prosperous Indo-Pacific Region” an “important pillar” of India’s partnership with the United States. India’s
conception of Indo-Pacific is not directed against any other nation such asChina, which has expressed
reservations on the concept). India also rejected the conflation of the Indo-Pacific idea with that of the “Quad” —
the forum for consultation between India, US, Japan and Australia.India does not view the vast Indo-Pacific region
as a “club of limited members”.
Indo-Pacific “stands for a free, open, inclusive region, which embraces us all in a common pursuit of progress and
prosperity. It includes all nations in this geography as also others beyond who have a stake in it”.India’s support
for the Indo-Pacific and the participation in the Quad would undermine the ASEAN. The PM declared that
Southeast Asia is the geographic core of the Indo-Pacific and that the ASEAN must remain central to the
construction of any architecture for the Indo-Pacific. The joint naval exercises with Singapore that have gone on
for a quarter of a century. Of special import was his announcement that India plans to conduct trilateral
exercises in the near future with Singapore and other ASEAN nations.Regular trilateral and plurilateral exercises
in the Andaman Sea and the Malacca Straits between India and the Southeast Asian nations, greater
interoperability between their naval forces, and the joint development of infrastructure across this vital maritime
estate will help develop a security architecture for the eastern Indian Ocean that is at the heart of the Indo-
Pacific.

Keywords: 1. Asia-Pacific 2. ASEAN 3. United States (US) 4. China 5. Quad

Q22. Evaluate India’s growing relationship with Eurasian region.

From Indo-Pacific to Eurasia Indian Express | India & the World

India becomes a full member of the Shanghai Cooperation Organisation (SCO) and expanded India’s idea of
Eurasia at the annual summit in Qingdao, China. India has been following a balancing act between the growing
strategic partnership with the US on the one hand, and its renewed effort at holding onto Russia and advancing
the complex relationship with China is a riveting one.That there is structural tension between these multiple
imperatives should not come as a surprise. Great power relations have entered a turbulent phase in recent
years. For nearly two and a half decades after the end of the Cold War, America, Europe, Russia, China and Japan
all sought to deepen economic cooperation and expand political cooperation with each other.

The Trump Administration has now called its relations with China and Russia “competitive”. Russia is at odds
with Europe and China’s relations with Japan have been under stress for a while. To make matters even more
complicated, Trump is quarreling — on economic and security issues — with his Western partners in G-7 and
NATO.It must also deal with the competing geopolitical frameworks in play. If the Indo-Pacific is a maritime
conception, Eurasia is a continental one.SCO is an effort by China and Russia to consolidate a continental
coalition in the heart of Asia.The obsession of the commentariat with the Quad and its presumed symbolism
misses the fact that India’s engagement with the US and Japan (the triad) has steadily advanced through this
decade. Meanwhile, India continues to sit with the Russians and Chinese in a trilateral forum as well as the BRICS
with Brazil and South Africa.The major powers, despite the new competition between them, have not stopped
finding compromises where possible. The principal question for Indian foreign policy is not whether India should

FOCUS | July 2018 | R AU’S I AS 159


@civilminds

engage both sides. It is about how best it can maximise the gains in both directions. That Delhi must look for
stronger ties with both the maritime and continental powers does not mean the nature and scope of these
possibilities is symmetric.Most of India’s trade, investment, technology and cultural ties are currently with the
maritime powers. The US and the European powers are also increasingly important security partners for India.
When you combine this with India’s natural maritime advantages, the Indo-Pacific comes into bold relief.

The continental construct, in contrast, is about managing India’s intractable problems. Unresolved territorial
disputes with China and Pakistan have resulted in the militarisation of its northern borders and blocked India’s
access to the inner Asian space. Those problems are not going to disappear any time soon.The SCO opens
possibilities for limiting the conflict with China through greater regional cooperation on countering terrorism,
religious extremism and pacifying conflict zones in Afghanistan and Pakistan. We are not there yet, but it is a goal
we must not abandon. Meanwhile, the SCO offers a platform to construct sustained high-level engagement with
Central Asia and strengthen the traditional collaboration with Russia in inner Asia.

Keywords: 1. Shanghai Cooperation Organisation 2. Russia 3. China 4. Inner Asia

Q23. Analyse the India’s contention with the Belt and Road Initiative (BRI).

Falling behind on Digital Silk Road Indian Express | Global Politics

India’s continuing political challenges with China’s Belt and Road Initiative have been matched by Delhi’s enduring
difficulties in advancing its own connectivity initiatives. At the Qingdao summit of the Shanghai Cooperation
Organisation, where India was participating for the first time as a full member, Delhi had to dissociate itself from
the consensus in favour of President Xi Jinping’s Belt and Road Initiative (BRI). China’s BRI will inch closer towards
India this week when Nepal Prime Minister Khadga Prasad Sharma Oli travels to China. Like most other
neighbours of India, Nepal has already endorsed President Xi’s Initiative. But like Pakistan, Sri Lanka and the
Maldives, Nepal is getting ready to sign onto major BRI projects. Many of these projects would be grouped under
the so-called Trans-Himalayan Connectivity Initiative. This is likely to include oil storage terminals, rail and road
links, hydel projects and electricity transmission lines.Although the question of costs relating to China’s projects
has been raised in many parts of the world, including most recently in Malaysia, it is unlikely to dampen the
enthusiasm for the BRI among India’s neighbours. For them, the issues relating to these projects are as much
economic as they are political.

For Pakistan, the participation in China’s BRI is very much part of its ever-deepening strategic partnership that
was forged decades ago to balance India. For other neighbours, the BRI offers, shall we say, “strategic autonomy”
from India.If India had taken for granted the deep geographic interdependence with its neighbours and did little
to modernise it for the 21st century, our neighbours had no choice but to swallow it. Now they have an
alternative, expensive though it might be, in the form of Chinese connectivity initiatives. They are embracing BRI
without any hesitation.The idea of seeking strategic autonomy from very large neighbours is not unique to South
Asia. Many of China’s immediate neighbours in East Asia do much the same — they seek insurance through
diversifying partnerships with many countries, including India. But unlike China, India has not been able to
deliver on the promises it has made to its East Asian partners.

If India has found it hard to develop institutional capabilities to implement infrastructure projects across and
beyond its borders, it has some possibilities in the arena of digital connectivity. They were demonstrated during
PM’s visit to Singapore where he signed a number of agreements to connect the financial markets of the two
countries. These included the launch of India’s RuPay card, the BHIM QR code and SBI’s cross-border remittance
app. Last year, India had launched the South Asia Satellite as part of its neighbourhood first policy.But here
again, China is racing ahead. Beijing has launched a number of ambitious initiatives, now being banded together
as the “Digital Silk Road”. China’s Digital Silk Road agenda is about strengthening internet infrastructure,

FOCUS | July 2018 | R AU’S I AS 160


@civilminds

deepening space cooperation, lowering barriers to e-commerce, developing common technology standards,
promoting cyber security, and improving the efficiency of policing systems among the BRI countries. China wants
to deploy its nationally developed platforms based on artificial intelligence, big data, cloud storage and quantum
computing to pursue these goals.

China and Nepal, for example, have operationalised an optic fibre link between the two countries earlier this
year. The link would eventually reduce Nepal’s dependence on India for internet connectivity. Last year, China’s
Huawei signed an agreement to construct the Pakistan East Africa Cable Express (PEACE) that would connect
Pakistan to Kenya via Djibouti. Huawei may extend this cable to Egypt in the north and South Africa. When
completed, the cable is likely to have a total length of 13,000 km.China’s digital initiative also includes deepening
space cooperation. Besides its long-standing space cooperation with Pakistan, China is discussing plans to
launch a national satellite for Nepal. Last year, Sri Lanka joined China’s Beidou navigation system. China wants
to leverage its earth observation satellite capabilities to deepen cooperation in a number of areas ranging from
environmental monitoring to disaster management. Oli’s visit to Beijing is expected to see the signing of a
bilateral agreement on setting up disaster management centres in Nepal that will be linked to China’s national
remote sensing system.

India has long had significant and growing national capabilities in the digital and space domains. But Delhi has
fallen terribly short in integrating these with larger national economic and security strategies. Delhi’s
bureaucratic bias towards to over-regulation, restrictions on the domestic private sector, constraints on
innovation and suspicion of external collaboration have limited India’s possibilities on digital development and
diplomacy.India paid little attention to China’s internal, cross-border and international infrastructure projects
that eventually came under the rubric of BRI. As a result, Delhi is struggling to cope with the strategic
consequences for the Subcontinent and the Indian Ocean. Unless Delhi quickly sheds its digital defensiveness,
that pattern might well repeat itself with China’s newest version of the Silk Road.

Keywords: 1. Nepal 2. Pakistan 3. Digital Silk Road 4. Sri Lanka

Q24. Evaluate the causes and impact of the US-China tariff wars on global economy?
No one wins: on the US-China trade war The Hindu | Global politics

The trade wars have finally begun with exchanging several threats between the United States and China
implemented a tariff of 25% on imports worth $34 billion. This marks the official beginning of what China dubs
as “the biggest trade war in economic history”. While this trade war is far from the biggest the world has seen, it
has the potential to cause some significant damage to the world economy. U.S. President Donald Trump, who
began the year by imposing tariffs on imported solar panels and washing machines, has vowed to possibly tax all
Chinese imports into the U.S., which last year added up to a little over $500 billion. Mr. Trump’s tariffs against
China will likely resonate with voters who believe in his “America First” campaign and perceive the trade deficit
with China as a loss to the U.S. economy. China, not surprisingly, has responded by targeting American exports
like soybean and automobiles, a move that could cause job losses in American states that accommodate Mr.
Trump’s voter base. Other major U.S. trading partners such as the European Union, Mexico, and Canada have
also slapped retaliatory tariffs on various U.S. goods.

In a globalised world, no country can hope to impose tariffs without affecting its own economic interests. Apart
from disadvantaging its consumers, who will have to pay higher prices for certain goods, tariffs will also disrupt
the supply chain of producers who rely on foreign imports. So both the U.S. and China, which have blamed each
other for the ongoing trade war, are doing no good to their own economic fortunes by engaging in this tit-for-tat
tariff battle. The minutes of the U.S. Federal Reserve June policy meeting show that economic uncertainty due to
the trade war is already affecting private investment in the U.S., with many investors deciding to scale back or

FOCUS | July 2018 | R AU’S I AS 161


@civilminds

delay their investment plans. China, which is fighting an economic slowdown, will be equally affected. The
ongoing trade war also threatens the rules-based global trade order which has managed to amicably handle
trade disputes between countries for decades. It could also isolate the U.S., which has refused to settle
differences through serious negotiations, as other global economies strike trade deals on their own. In March, for
instance, 11 Asia-Pacific countries went ahead to sign a trans-Pacific trade deal while leaving out the U.S., which
had pulled out of the Trans-Pacific Partnership in early 2017. If global trade tensions continue to simmer,
however, it may not be too long before countries resort to other destructive measures such as devaluing their
currencies to support domestic exporters. The world economy, which is on a slow path to recovery, can do
without such unnecessary shocks.

Keywords: 1. Asia-Pacific 2. Tariffs 3. TPP 4. Donald Trump

Q25. Examine the causes of friction in the relationship between India and United States.

Allies, interrupted: on India-U.S. ties The Hindu | #Bilateral Relationship

There are enough signs that relations between India and the United States have suffered, with officials in both
capitals now freely conceding that their interests are diverging. From the U.S. side, policy decisions by President
Donald Trump to walk out of the multilateral nuclear deal with Iran, and the U.S. Congress’s CAATSA law
sanctioning Iran and Russia have set up an inevitable conflict. Mr. Trump’s insistence on tough sanctions against
all those continuing to engage with Iran and Russia limits India’s options on energy security and defence
procurement. During her visit last week, Nikki Haley, the U.S. envoy to the UN, told India to “revise” its
relationship with Iran; this line is expected to be reiterated by U.S. interlocutors in the coming days. Added to this
confrontation is the U.S.’s tough policy on trade tariffs, applied to ally and adversary alike, including India. For its
part, India has taken a policy turn away from four years of a pro-U.S. tilt. India has reached out to the Chinese
and Russian Presidents in informal summit. At variance with the U.S. position on limiting engagement with these
very countries, India promised to raise oil imports from Iran this year, committed to far greater engagement on
the Chabahar port project and oilfields in Iran, while negotiating a $5.5 billion deal with Russia for the S-400
Triumf missile systems. These will trigger U.S. sanctions unless the two countries reach a compromise.

What is more troubling for bilateral ties is that despite the obvious problems, the political will to address these
issues is now considerably diminished. In contrast to his meetings with the Russian and Chinese leaderships,
Mr. Modi has had little contact with Mr. Trump since their meeting in Manila last November, which by all
accounts did not go well. Now, the postponement of the Indian Foreign and Defence Ministers’ “2+2” dialogue
with their U.S. counterparts has denied the governments a chance to gather together the fraying bilateral
threads. It is imperative that the dialogue be quickly rescheduled. While the U.S. has traditionally applied
pressure on its allies to limit their engagement with countries it considers to be threats to the international order,
the manner in which deadlines have been publicly issued by US twice this week will only make its demands more
difficult for India to even consider. India must now decide how best to deal with the ultimatums, with U.S.
sanctions kicking in by November. The clock is ticking on the relationship.

Keywords: 1. 2+2 Dialogue 2. CAATSA law 3. China 4. Iran

FOCUS | July 2018 | R AU’S I AS 162


@civilminds

Here, we are sharing case studies for self- practice. These


Case Studies case studies are drawn from various aspects of professional
and personal experiences, and are helpful for General
For mains Gs paper IV Studies- Paper IV.
Do practice them!

Case 1: XYZ Ltd. is a large transnational company having diversified business activities with a huge shareholder
base. The company is continuously expanding and generating employment. The company, in its expansion and
diversification programme, decides to establish a new plant at Vikaspuri, an area which is underdeveloped. The
new plant is designed to use energy efficient technology that will help the company to save production cost by
20%. The company’s decision goes well with the Government policy of attracting investment to develop such
underdeveloped regions. The Government has also announced tax holiday for five years for the companies that
invest in underdeveloped areas. However, the new plant may bring chaos for the inhabitants of Vikaspuri region
which is otherwise tranquil. The new plant may result in increased cost of living, aliens migrating to the region,
disturbing the social and economic order. The company sensing the possible protest tried to educate the people
of Vikaspuri region and public in general that how its Corporate Social Responsibility (CSR) policy would help
overcome the likely difficulties of the residents of Vikaspuri region. In spite of this the protests begin and some of
the residents decided to approach the judiciary as their plea before the Government did not yield any result.

(a) Identify the issues involved in the case.

(b) What can be suggested to satisfy Company’s goal and to address the resident’s concern.

Case 2: A private company is known for its efficiency, transparency and employee welfare. The company though
owned by a private individual has a cooperative character where employees feel a sense of ownership. The
company employs nearly 700 personnel and they have voluntarily decided not to form a union.

One day suddenly in the morning, about 40 men belonging to a political party gatecrashed into the factory
demanding jobs in the factory. They threatened the management and employees, and also used foul language.
The employees feel demoralized. It was clear that those people who gate-crashed wanted to be on the payroll of
the company as well as continue as the volunteers/members of the party. The company maintains high
standards in integrity and does not extend favours to civil administration that also includes law enforcement
agency. Such incidents occur in public sector also.

1. Assume you are the CEO of the company. What would you do to diffuse the volatile situation on the date of
gate-crashing with the violent mob sitting inside the company premises?

2. What could be the long-term solution to the issue discussed in the case?

3. Every solution/action that you suggest will have a positive and a negative impact on you (as CEO), the
employees and the performance of the employees. Analyze the consequences of each of your suggested
actions.

FOCUS | July 2018 | R AU’S I AS 163


@civilminds

Case 3: One of the scientists working in the R & D laboratory of a major pharmaceutical company discovers that
one of the company’s best-selling veterinary drugs, B has the potential to cure a currently incurable liver disease
that is prevalent in tribal areas. However, developing a variant of the drug suitable for human beings entailed a
lot of research and development having a huge expenditure to the extent of 50 crores rupees. It was unlikely that
the company would recover the costs as the disease was rampant only in poverty-stricken area having very little
market otherwise.

If you were the CEO, then—

(a) Identify the various actions that you could take;

(b) Evaluate the pros and cons of each of your actions.

FOCUS | July 2018 | R AU’S I AS 164


@civilminds
@civilminds

You might also like